Kaplan Real Property Flashcards

1
Q

A farm and an orchard are adjoining tracts of land located in a county. In 2006, a farmer purchased the farm, a 10-acre tract, in fee simple absolute. The orchard, a 20-acre tract situated to the north of the farm, was owned by a rancher in fee simple absolute. A remote predecessor of the farmer had granted to a shepherd a way for egress and ingress across the farm under such terms and circumstances that an easement appurtenant to the orchard was created. This right-of-way was executed by deed and properly recorded. The shepherd, however, never made any actual use of the right-of-way.In 2010, the rancher conveyed the orchard to the farmer. The next year, the farmer conveyed the orchard by deed to an investor for a consideration of $250,000, receipt of which was acknowledged. Neither the rancher—farmer deed nor the farmer— investor deed contained any reference to the easement for right-of-way. The investor has now claimed that she has a right-of-way across the farm. The farmer, on the other hand, has informed the investor that no such easement exists.Assume that both the farm and the orchard abut a public highway and that ingress and egress are afforded the investor by that highway. In an appropriate action by the investor to determine her right to use the right-of-way across the farm, she should(A) lose, because the easement was extinguished by merger when the farmer acquired the orchard from the rancher.(B) lose, because the right-of-way was abandoned inasmuch as there never was any actual use made.(C) win, because the farmer had constructive notice of the easement.(D) win, because the investor acquired an easement by implication.

A
  1. (A) This Multistate questions deals with extinguishment of on easement by merger. When an easement appurtenant exists and both the dominant and servient tenements come under the ownership of the same person, the easement is terminated by operation of law. The apparent rationale is that one cannot have an easement in his own property. Therefore, when the farmer (who was already the owner of the farm) acquired the orchard from the rancher, the easement in effect merged in his fee simple ownership of both properties. Choice (B) is incorrect. When an easement is extinguished or terminated due to abandonment, there needs to be a clear showing by the dominant tenant that s/he intends to abandon the use. This is evidenced by the conduct of the dominant tenant. Non-use coupled with an intent to abandon will generally suffice to show affirmative abandonment by the dominant tenant. Here, there was no such abandonment shown by the shepherd, who was the original dominant tenant. The shepherd’s lack of use was not enough to create an abandonment of the easement. Choice (B) is incorrect because the reason why the easement was lost was due to the merger of the dominant and servient estates when the rancher sold the orchard to the farmer. Choice (C) is incorrect. While it is true that the shepherd had recorded the right-of-way in the past (thus putting the farmer on constructive notice of it), that does not change the fact that the easement was still lost by merger. Because the servient and dominant tenements had merged due to the rancher’s conveyance to the farmer, the easement was lost despite the fact that there was a recordation of it. Because this choice focuses only on the recordation without taking into account the merger of the estates, it is incorrect. Choice (D) is incorrect. An easement may also be created, though not expressly in a writing, by implication. An easement by implication generally arises when the owner of two or more adjacent parcels sells one or more of them and it is clear (although no easement was mentioned in the instrument of conveyance) that one was intended. In order to establish an easement by implication, one of the requirements is that the easement be reasonably or strictly necessary to the use and enjoyment of the quasi- dominant estate. Even though the farmer had gained common ownership and sold the orchard portion to the investor (which would be the quasi-dominant estate), the highway provides sufficient ingress and egress such that the easement across the farm portion is neither reasonably nor strictly necessary. Thus, the investor did not acquire an easement by implication when the farmer conveyed the northern half to her.
How well did you know this?
1
Not at all
2
3
4
5
Perfectly
2
Q
  1. An orange grove and a vineyard are adjoining tracts of land located in a county. In 2006, a farmer purchased the orange grove, a 10-acre tract, in fee simple absolute. The vineyard, a 20-acre tract situated to the north of the orange grove, was owned by a rancher in fee simple absolute. A remote predecessor of the farmer had granted to a shepherd a way for egress and ingress across the orange grove under such terms and circumstances that an easement appurtenant to the vineyard was created. This right-of-way was executed by deed and properly recorded. The shepherd, however, never made any actual use of the right-of-way.In 2010, the rancher conveyed the vineyard to the farmer. The next year, the farmer conveyed the vineyard by deed to an investor for a consideration of $200,000, receipt of which was acknowledged. Neither the rancher—farmer deed nor the farmer— investor deed contained any reference to the easement for right-of-way. The investor has now claimed that she has a right-of-way across the orange grove. The farmer, on the other hand, has informed the investor that no such easement exists.Assume that the orange grove abuts a public highway. The only access to that public highway for the vineyard is across the orange grove. If the investor initiates an appropriate action to determine her right to use the right-of-way across the orange grove, she should(A) prevail, because an easement by implication arose from the farmer’s conveyance to the investor.(B) prevail, because once an easement is properly recorded, it remains in effect ad finem until expressly released.(C) not prevail, because any easements were extinguished by merger when the farmer acquired a fee simple in the vineyard.(D) not prevail, because the deed of conveyance from the farmer to the investor failed to contain any mention of the right-of-way.
A
  1. (A) This is an example of an easement by implication. An implied easement is created and proved not by the words of the conveyance (because there is no such express language), but by all the circumstances surrounding the execution of the conveyance. It is based upon the intent of the parties. There are five prerequisites to the creation of an implied easement: (1) There must be a quasi-easement; (2) there must be a conveyance of one part of the property and a retention of another part of the property by the grantor; (3) the quasi-easement must be apparent at the time of the easement; (4) the quasi-servient tenement must be permanently adapted to serve the quasi-dominant tenement; and (5) the quasi-easement must be (a) reasonably necessary to the enjoyment of the quasi-dominant tenement if that tract is conveyed, and (b) strictly necessary if the quasi-dominant tenement is retained by the grantor. Therefore, because the investor’s only access to the highway was across the orange grove, she would have an easement by implication. Choice (B) is incorrect. This choice suggests that because the easement was recorded by the shepherd, that it remains in effect indefinitely until it is expressly released. However, an easement can be extinguished by merger, and that is what occurred when the rancher conveyed the vineyard to the farmer. Because this choice focuses too much on only one way that an easement can be extinguished (by express release), it is too limiting and therefore incorrect. Choice (C) is incorrect. It is true that the easement granted to the shepherd was extinguished by merger when the farmer acquired a fee simpLe in the vineyard, thus extinguishing that easement given to the shepherd by express grant. However, when the farmer sold the former the vineyard portion to the investor and created a parcel that had no access to the public highway, an implied easement arose due to strict necessity. Because this choice ignores the creation of an easement by implication, it is incorrect. Choice (D) is incorrect. An easement can be created by express grant and can be mentioned in a deed of conveyance. That is but one way in which an easement can be created. Even though the deed faiLed to contain any mention of a right-of-way, an easement by implication may stiLl arise. Because this choice fails to account for other ways in which an easement can be created, it is incorrect.
How well did you know this?
1
Not at all
2
3
4
5
Perfectly
3
Q
  1. A landowner, being owner in fee simple of a farm conveyed the property by warranty deed to an investor. The investor gave a farmer a mortgage on the farm to secure a loan from the farmer to the investor in the amount of $50,000. The mortgage was recorded immediately. Two years later, the investor conveyed the farm to his friend by quitclaim deed. According to the deed instrument, the friend “assumed the mortgage.” The investor then defaulted on the mortgage, and the farmer brought an in personam action against the friend to recover the amount of the mortgage due.The farmer will most likely(A) prevail, because the friend acquired title to the farm by quitclaim deed.(B) prevail, because the farmer was a third-party beneficiary under the conveyance between the investor and the friend.(C) not prevail, because the conveyance between the investor and the friend did not effectuate a delegation of duties.(D) not prevail, unless the land was subject to the mortgage.
A
  1. (B) A key testing area on the MuLtistate deals with mortgages. The difference between a transfer of the mortgage property “subject to” and “assuming” the mortgage is the difference between personal liability and the lack thereof on behalf of the grantee. In a conveyance of Land “subject to” a mortgage, the grantee is not personally liable for the debt that the mortgage secures. ConverseLy, in a sale of land in which the purchaser “assumes the mortgage,” the purchaser or grantee Is personally liable for the mortgage debt. Because the friend “assumed the mortgage,” he is personally liable; therefore, choice (B) is correct. Choice (A) is incorrect. A quitclaim deed is a deed that contains none of the six covenants of title. By quitclaiming title to the land, the grantor only transfers whatever title s/he holds in the land, but gives no covenants. The acquisition of title by quitclaim deed does not address the issue of whether or not the friend assumed any personal liability for the mortgage on the property. Because this choice does not address the friend’s liability and focuses only on deed covenants, it is incorrect. Choice (C) is incorrect. The friend did assume the mortgage underthe facts. That alone would make the friend personally responsible for the mortgage debt if the investor defaulted on the mortgage. An actual delegation of duties is not required because the deed itself stated that the friend “assumed the mortgage.” Because this answer choice falsely suggests that a delegation of duties was necessary in order to make the friend liable, it is incorrect. Choice (D) is incorrect. This choice suggests that the farmer will only prevail if the land was subject to the mortgage. However, the friend’s assuming of the mortgage creates liability and will provide a means by which the farmer can prevail. Because this answer choice fails to take into account the friend’s assuming of the mortgage, it is incorrect.
How well did you know this?
1
Not at all
2
3
4
5
Perfectly
4
Q
  1. A rancher, being owner in fee simpleconveyed the property by warranty deed to a woman. The woman gave her niece a mortgage on the ranch to secure a loan from the niece to the woman in the amount of $500,000. The mortgage was recorded immediately. Two years later, the woman conveyed the ranch to a farmer by quitclaim deed. The woman then defaulted on the mortgage, and the niece brought an in personam action against the farmer to recover the amount of the mortgage due.Assume that the woman’s quitclaim deed to the farmer made no reference to the mortgage. The woman then defaulted on the mortgage, and the niece brought an in personam action against the farmer to recover the amount of the mortgage due.The mortgagee will probably(A) succeed, because an implied delegation of duties resulted from the woman’s conveyance to the farmer.(B) succeed, because the niece was a third-party beneficiary under the conveyance between the woman and the farmer.(C) not succeed, because the farmer did not promise to pay the mortgage debt.(D) not succeed, unless the farmer had constructive notice of the existence of the mortgage.
A
  1. (C) In order for the grantee-farmer to be personally liable to the mortgagee, the farmer must “assume the mortgage.” Thus, in this situation where the deed instrument makes no reference to the mortgage, the grantee does not assume the mortgage and is not personally liable to the mortgagee. Choice (A) is incorrect. Where the deed language is silent, the grantee is considered to take “subject to” the mortgage. This means that the grantee is not personally liabLe for the mortgage debt. A deed’s silence on this matter does not imply a delegation of duties such that the farmer must now pay just because she was the woman’s grantee. An affirmative assumption of the mortgage must be stated in the deed and no delegation of the duty to pay will be implied by the conveyance. Choice (B) is incorrect. Because the deed was silent, the farmer took the property “subject to” the mortgage. Because the farmer took the property in this fashion, there is no agreement between the farmer and the woman that the farmer would pay the mortgage for the benefit of the niece. Thus, there is no third.party beneficiary situation like there was in Question 3. Choice (D) is incorrect. This choice suggests that if the farmer had constructive notice of the existence of the mortgage, then that alone would have made her liable for the debt. Even though the mortgage was recorded, that will not be enough to make the farmer liable for the mortgage debt because the woman—farmer deed never affirmatively stated that the farmer would assume the mortgage.
How well did you know this?
1
Not at all
2
3
4
5
Perfectly
5
Q
  1. For many months, a buyer had been negotiating with a man for the purchase of a tract of land. Finally, on August 18, 2009, the buyer and the man entered into a real estate sales contract that provided in part:“I, (the man), agree to convey good and marketable title to (the buyer) 60 days from the date of this contract.” The stated purchase price for the tract was$175,000.On October 11, 2009, the buyer phoned the man and told him that his title search indicated that a third party, not the man, was the owner of record of the property. The man responded that, notwithstanding the state of the record, he had been in adverse possession for 21 years. The statutory period of adverse possession in this jurisdiction is 20 years.The next day the buyer conducted an investigation that revealed that the man had, in fact, been in adverse possession of the tract for 21 years. At the time set for closing, the man tendered a deed in the form agreed in the sales contract. The buyer, however, refused to pay the purchase price or take possession of the tract because of the man’s inability to convey “good and marketable title.”In an appropriate action by the man against the buyer for specific performance, the vendor will(A) prevail, because he has obtained “good and marketable title” by adverse possession.(B) prevail, because the man’s action for specific performance is an action in rem to which the third party is not a necessary party.(C) not prevail, because an adverse possessor takes title subject to an equitable lien from the dispossessed owner.(D) not prevail, because the buyer cannot be required to buy a lawsuit even if the probability is great that the buyer would prevail against the man.
A
  1. (D) Title established through adverse possession is free from encumbrance and of a character to assure quiet and peaceful enjoyment of the property by the vendee. However, it is not a “marketable” title of record until there has been a judicial determination of such title. To show a record title by adverse possession requires a suit and the recording of a decree. Even though a court may determine that the vendor had title by adverse possession, the vendee did not bargain for that kind of title when the contract required a “marketable” title of record. Therefore, in accordance with the prevailing view, choice (D) is correct. This specific problem is referred to in Smith and Boyer, Law of Property, pg, 264. Choice (A) is in correct. The vendor! seller will not have “marketable” title of record untilthere has been a judicial determination of such title. Until that happens, the seller does not have the “good and marketable title” as recited in the original sales contract. Choice (B) is incorrect. The man is suing the buyer in an in personam action because the man is asking the buyer to tender the purchase price. This choice is also incorrect in that it fails to address the fact that without a judicial decree, the man did not have marketable title of record and cannot prevail in his action against the buyer. Choice (C) is incorrect. An equitable lien is a charge placed upon property that is imposed by law to prevent unjust enrichment. This choice suggests that the dispossessed owner would be able to place an equitable lien on the property that the man (as the would-be adverse possessor) is taking by adverse possession. However, if title were vested in the man by a judicial decree, the man would take title free from encumbrances. In other words, if the man were to prevail, the actual owner would not be able to have an equitable lien placed on the property.
How well did you know this?
1
Not at all
2
3
4
5
Perfectly
6
Q
  1. An uncle owns his property in fee simple. The uncle executes and delivers a deed to the property with the following granting clause:“To my nephew and my niece for the life of my nephew, then to my gardener.”As a result of the grant, which of the following is the most accurate statement regarding the ownership rights of the property?(A) The nephew and niece are joint tenants in fee simple defeasible, and the gardener has an executory interest.(B) The nephew has a life estate, the niece has a life estate pur autre vie, and the gardener has a vested remainder.(C) The nephew and niece are tenants in common for the life of the first of them to die, the nephew and niece have contingent remainders in fee simple defeasible, and the gardener has an executory interest.(D) The nephew has a life estate, the niece also has a life estate, and the gardener has a vested remainder.
A
  1. (B) First of all, the conveyance created a life estate in the nephew, and the niece had a life estate pur autre vie for the life of the nephew. The gardener received a vested remainder in fee simple absolute. A vested remainder is a remainder created in an ascertained and existing person that is not subject to any condition precedent except the normal termination of the preceding estate. Because the gardener would take as soon as the nephew died, the gardener received a vested remainder. Choice (A) is incorrect. A fee simple defeasible is a fee simple interest that could last for an infinite duration but can be terminated upon the happening of a specific event. The nephew and niece had, at most, a life estate in the property. Moreover, the gar. dener did not have an executory interest; the gardener received a vested remainder. Choice (C) is incorrect. The nephew and niece did not receive contingent remainders in the property. In fact, they received no future interest. The nephew and niece received life estates, which are a present possessory interest in Land that are to take effect immediateLy. Moreover, this answer choice is incorrect because the gardener received a vested remainder upon the death of the nephew. The gardener did not receive an executory interest. Choice (D) is incorrect. As seen in the other answer explanations, the niece has a life estate purautre vie, not a life estate measured by her own life.
How well did you know this?
1
Not at all
2
3
4
5
Perfectly
7
Q
  1. A woman was the fee simple owner of a 20-acre tract of land. When the woman moved to another state, a man took possession of the tract. The man’s possession has at all times complied with the requirements of the applicable adverse possession statute in effect.Twelve years after the man took possession, the woman died intestate, leaving her six-year-old son as her only surviving heir. Nine years after the woman’s death, when the son was 15, the son’s guardian discovered that the man was in possession of the tract. Assume that the statutory period of adverse possession is 20 years and the age of majority is 18.Which of the following correctly describes the state of title to the tract?(A) The man has acquired title by adverse possession.(B) The man will not acquire title unless he continues in adverse possession for an additional three years, or until the son reaches the age of 18.(C) The man will not acquire title unless he continues in adverse possession for an additional eight years, making a total of 12 years after the woman’s death.(D) The man will not acquire title unless he continues in adverse possession for an additional 12years, or nine years after the son attains the ageof 18.
A
  1. (A) In this adverse possession problem, the facts indicate that the statutory period of adverse possession in this jurisdiction is 20 years. Looking back, the facts tell you that 12 years after the man took possession (of the tract), the woman died. Next, nine years after her death, the son’s guardian discovered the man in (adverse) possession of the tract. Now, adding the 12 years (before the woman’s death) to the nine years (following death) would create 21 years. Thus, because the man’s adverse possession is beyond the statutory period, choice (A) is correct. Choice (B) is incorrect . The disability of the successor, the son, would have no effect because the man is already in possession of the tract due to his fulfillment of the statutory requirements. Therefore, the man does not have to continue on for an additional three years. Choice (C) is incorrect. The disability of the successor, the son, would have no effect because the man is already in possession due to his fulfillment of the statutory requirements. Therefore, the man does not have to continue on for an additional eight years. Choice (D) is incorrect. Because the disability of the successor, the son, would have no effect here, the man is already in possession of the tract due to his fulfillment of the statutory requirements. Therefore, the man does not have to continue on for an additional 12 years.
How well did you know this?
1
Not at all
2
3
4
5
Perfectly
8
Q
  1. A sister and brother own a ranch as joint tenants. If the sister transfers her interest by quitclaim deed to her friend without the brother’s knowledge, what interest, if any, does the friend have in the ranch?(A) No interest.(B) An undivided one-half interest with right of survivorship.(C) An undivided one-half interest without right of survivorship.(D) A lien against the entire property.
A
  1. (C) If one joint tenant conveys his interest in the property to another, even if the conveyance is done secretly, severance occurs, whereby the right of survivorship is severed and a tenancy in common results. Based on the facts presented, the friend would take an undivided one-half interest in the ranch without right of survivorship. Choice (A) is incorrect. When the sister conveyed to her friend by quitclaim deed, she transferred to the friend whatever interest she happened to own at the time. The sister’s unilateral severance of the joint tenancy does not change the fact that she still transferred some interest in the ranch to her friend. Choice (B) is incorrect. When the sister (as a joint tenant) conveyed the property to the friend, that act alone severed the joint tenancy and destroyed any right of survivorship that once existed. The friend will take an undivided one-half interest, but it will not be with any right of survivorship. Choice (D) is incorrect. The friend was the sister’s grantee and owns an undivided one-half interest in the ranch. The friend does not have a lien against the property because he is not a creditor.
How well did you know this?
1
Not at all
2
3
4
5
Perfectly
9
Q
  1. A wife and husband are married and own a dairy as tenants by the entirety. If the wife transfers her interest in the dairy by quitclaim deed to her aunt without the husband’s knowledge, what interest, if any, does the aunt have?(A) No interest.(B) An undivided one-half interest with right of survivorship.(C) An undivided one-half interest without right of survivorship.(D) A lien against the entire property.
A
  1. (A) A tenancy by the entirety, unlike a joint tenancy, does not allow either party to convey away his or her interest in the property without the other’s consent. As a result, the aunt would not have any interest in the dairy. Choice (B) is incorrect. With a tenancy by the entirety, neither spouse can individually dispose of any interest in the estate; rather, both must join in the conveyance. Because of this, the aunt does not have an undivided one-half interest. Choice (C) is incorrect. With a tenancy by the entirety, neither spouse can individualLy dispose of any interest in the estate; rather, both must join in the conveyance. Because of this, the aunt does nothave an undivided one-half interest. Choice (D) is incorrect. With a tenancy by the entirety, neither spouse can individually dispose of any interest in the estate; rather, both must join in the conveyance. Because of this, the aunt does not have a lien or any other interest in the dairy.
How well did you know this?
1
Not at all
2
3
4
5
Perfectly
10
Q
  1. A man, his nephew, and his son are the owners of three contiguous lots in the city. A downward slope exists from the man’s land to the son’s land.The man’s and the nephew’s lots were in an unimproved natural state. The son, however, had lived for 10 years in a house that he had built on his property.In 2009, the man planted trees and shrubbery on his land along the boundary of the nephew’s lot.In 2010, the nephew, in preparation for building a house on his lot, carefully excavated an area eight feet deep for the purpose of building a basement. The side of the excavation closest to the man— nephew boundary suddenly collapsed, and a quantity of the man’s soil, trees, and shrubbery fell into the hole. The nephew hauled away the debris.In an appropriate action by the man against the nephew to recover for the damage to his land, judgment should be for whom?(A) The nephew, if he was conducting the excavation work non-negligently.(B) The nephew, because he was under no duty to support the man’s land in its improved state.(C) The man, because a landowner is entitled to support of his land in its natural condition.(D) The man, because a landowner has an absolute right to have his land supported by the neighboring land.
A
  1. (C) One who by excavation, or other methods, withdraws lateral support from her neighbor’s land is liable for the injury done to such land in its natural condition, regardless of negligence. The common law right to lateral support is a right to the support of land in substantially its natural condition. It does not include the right to have the additional weight of artificial structures supported by the neighboring land. However, a defendant may be held liable for damage caused to structures if there is proof of negligence on his part. In the present example, the man’s land was essentially in its natural condition. As a result, choice (C) states the correct rule. Choice (A) is incorrect. Because the man’s land was in a natural condition, he has an absolute right to lateral support. So when the nephew withdrew lateral support from his neighbor’s land, the nephew became absolutelyandstrictly liable (regardless of negligence) for the damage to the man’s land. Because this choice falsely suggests that lack of negLigence would somehow be a defense to the nephew in this situation, it is incorrect. Choice (B) is incorrect. Because the man’s land was in a natural condition, he has an absolute right to lateral support. So, when the nephew withdrew lateral support from his neighbor’s land, the nephew had an absolute duty to the man not to cause any damage to the land from a withdrawal of lateral support. Choice (D) is incorrect. Although this choice is correct in stating that the man had an absolute right to have his land supported, it fails to explain why the man has that right in this situation. It is only because the land was in its natural condition with no artificial structures on it that the man has this absolute right to lateral support. Because choice (C) is more complete in giving the reason why the man is entitled to this right (the land stayed in its natural condition), it is preferred over choice (D).
How well did you know this?
1
Not at all
2
3
4
5
Perfectly
11
Q
  1. A homeowner, his daughter, and his sister are the owners of three contiguous lots in the city. A downward slope exists from the homeowner’s land to the sister’s land.The homeowner’s and daughter’s lots were in an unimproved natural state. The sister, however, had lived for 10 years in a house that she had built on her property.The daughter, in preparation for building a house on her lot, carefully excavated an area eight feet deep for the purpose of building a basement. The daughter completed construction of her house and macadamized an area for use as a driveway without changing the former contours of the land. Shortly thereafter, the sister began to make complaints to the daughter about the flooding of her basement, which she claimed had been previously free of water.The sister then built a concrete wall three feet along her border with the daughter to prevent the flow of rain water running onto her land from the daughter’s property. This caused the surface water to stand and become stagnant on the daughter’s land. The daughter demanded that the sister remove the wall, and upon the sister’s refusal, the daughter brought an appropriate action to compel removal.The most likely result is(A) the sister must remove the wall because she has no right to obstruct the flow of such surface water.(B) the sister must remove the wall at the daughter’s expense.(C) the sister may leave the wall without being liable to the daughter for money damages.(D) the sister may leave the wall, but she will be liable to the daughter for money damages.
A
  1. (C) This Multistate question deals with surface waters. Surface waters are those that come from rain, springs, and melting snow and ice, and simply follow the contours of the land and have not yet reached a natural water course or basin with well- defined bed and banks. Under the common law rule (still followed in a majority of jurisdictions), sometimes called the “common enemy rule” because surface water is considered a common enemy, the lower tract is not burdened with any servitude in favor of the higher land, and the owner of the lower tract has the right to protect his lower tract from “the common enemy” or the flow of surface water by making any improvements that are suitable for the purpose. Choice (A) is incorrect. Under the “common enemy rule” for surface waters, the sister had a right to build the wall to protect her downhill home from the extra water running down into it. Because she does have the right to obstruct the flow of the excess surface water caused by the daughter’s driveway, this choice is incorrect. Choice (B) is incorrect. Under the “common enemy rule” for surface waters, the sister had a right to build the wall to protect her downhill home from the extra water running down into it. Because she does have the right to obstruct the flow of the excess surface water caused by the daughter’s driveway, this choice is incorrect. Choice (D) is incorrect. Not only does the sister have the right to build the wall to protect herself from the excess water caused by the daughter’s new driveway, but the sister would also not be liable to the daughter for the damages caused by the sister’s new wall.
How well did you know this?
1
Not at all
2
3
4
5
Perfectly
12
Q
  1. A landowner, her aunt, and her brother are the owners of three contiguous lots. A downward slope exists from the landowner’s land to the brother’sland.The aunt, the owner in the middle of the three lots, was an amateur scientist. She began experimentation with electricity, using a lightning rod to conduct her experiments. Frequently, the electrical storms apparently originated in and over the area within which the brother’s land is located. Because of this, the aunt would often angle her rod over the fence between her property and the brother’s property to maximize her chances of success. After one entire rainy and windy season of experiments, the brother had grown tired of the aunt’s lightning-rod intrusions because they interfered with his ham radio operations in his garage. The brother brought suit to enjoin such lightning-rod experiments. At trial, it was found that the aunt had been conducting her activities in the airspace directly above the brother’s land.The court should(A) enjoin the aunt’s experiments because they constitute an unreasonable interference with the space above the brother’s land.(B) enjoin the aunt’s experiments because one does not have the right to engage in such scientific experiment without permission.(C) not enjoin the aunt’s lightning rod experiments if they were necessary to protect her property from the electrical storms.(D) not enjoin the aunt’s experiments because the brother does not own the space above his land.
A
  1. (A) The possessor of real property has the right to the exclusive possession of the surface of the ground, the airspace above, and the soil underneath, the extent of which is determined by the exterior boundaries extended vertically upward and downward. Choice (A) is correct because any use of the space above one’s land that is unreasonable, improper, or interferes with the use and enjoyment of the surface can constitute a trespass. Choice (B) is incorrect. The aunt will be enjoined not because she did not have permission; rather, she will be enjoined for the simple reason that her conduct created an interference with the airspace directly above the brother’s home. These Lightning rod experiments directly affected the brother’s ham radio activities and constituted a trespass. Choice (A) more directly addresses the aunt’s conduct and is a better explanation of why she will be enjoined. Choice (C) is incorrect. Even if the experiments were necessary to protect her property from electrical storms, the aunt cannot use this as a defense to her intentionaL trespass into the airspace directly above the brother’s land. If the aunt were that concerned about her property, she could have just as easily angled her lightning rod above her own property. Choice (D) is incorrect. This statement is not true on the facts. The brother does own the airspace directLy above his land and home. Therefore, he is allowed to enjoin the aunt from conducting experiments in the airspace above that interfere with the use and enjoyment of the surface.
How well did you know this?
1
Not at all
2
3
4
5
Perfectly
13
Q
  1. An investor owned a four-story office building located in downtown. The building was old and badly in need of renovation. To finance the improvements, the investor borrowed $125,000 from his friend. As consideration for the loan, the investor executed a promissory note for $125,000 payable to the friend in one year and secured by a mortgage on the building. The mortgage was dated January 1, and was recorded January 2. Thereafter, on February 1, the investor executed a deed absolute on the building and named the friend as grantee. This deed, although absolute in form, was intended only as additional security for the payment of the debt. In order to make judicial foreclosure unnecessary and to eliminate the right to redeem, the investor then delivered the deed to his nephew in escrow with instructions to deliver the deed to the friend if the investor failed to pay his promissory note at maturity.On January 1 of the following year, the investor failed to pay the note when it came due. Thereupon, the nephew, in accordance with the escrow instructions, delivered the investor’s deed on the office building to the friend, which the friend promptly and properly recorded. Two weeks later, the investor tendered the $125,000 indebtedness to the friend. When the friend refused to accept it, the investor brought an appropriate action to set aside and cancel the deed absolute and to permit the redemption of the building from the friend. Conversely, the friend counterclaimed to quiet title and argued that the deed absolute was intended as an outright conveyance upon default.The court should enter a judgment that will grant the relief sought by(A) the investor, but only if the investor can establish that the mortgage takes precedence over the deed absolute because it was executed earlier in time.(B) the investor, because the deed absolute did not extinguish his right of redemption.(C) the friend, because the deed absolute effectuated an outright conveyance that extinguished the redemption interest sought to be retained by the investor.(D) the friend, because the investor is estopped to deny the effect of the deed absolute in conjunction with the escrow arrangement.
A
  1. (B) Mortgages is an area on the MBE that is heavily tested. In this particular question, it has long been recognized in equitythata deed absolute intended forsecurity will, in fact, be construed as a mortgage. This is not really surprising when it is remembered that the traditional form of the mortgage was a conveyance subject to defeasance, and that the equity of redemption was created by the equity court to protect the mortgagor after default. In order to preserve this equity of redemption, various rules were formulated to prevent mortgages from limiting (or clogging) the equity of redemption. The most common example of such rules is the principle “once a mortgage always a mortgage.” This, in effect, means that a mortgagee cannot circumscribe the mortgagor’s right to redeem by disguising the transaction as an outright conveyance. In this example, the facts indicate that the investor executed the deed absolute to the friend as additionaL security. Therefore, the deed will not extinguish the investor’s right of redemption because it (the deed absolute) will be construed as a mortgage and not an outright conveyance. Choice (A) is incorrect. This choice falsely suggests that the only way that the investor can prevaiL is by showing that mortgage takes priority because it was executed earlier in time. However, the investor does not need to go to this much trouble just to prove that the deed was not an absolute or outright conveyance. Because the investor executed the deed absolute to the friend only as additional security, there was no intention to make an outright conveyance. The investor prevails because the delivery of the deed was not intended as an absolute or outright conveyance, and the investor does not need to go to the extra effort of showing which was executed first because the deed will be construed as only a mortgage. Choice (C) is incorrect. At common law, the equity of redemption was created by the equity court to protect the mortgagor after default. In orderto preserve this equity of redemption, various rules were formulated to prevent mortgages from limiting (or clogging) the equity of redemption. Because the deed will be treated as a mortgage (and this is especially true given that the facts state that the deed was delivered only for the limited purpose of providing additional security), the right of redemption will not be extinguished. The act of delivering a deed will not be enough to extinguish the right of redemption. Choice (D) is incorrect. The delivery of the deed to the nephew with escrow instructions will still not be enough to overcome the presumption that, at most, this was to be treated as a mortgage with the right of redemption. Never was this arrangement designed to be an outright conveyance. As such, to say that the investor is estopped to deny the actuaL nature of the transaction is to effectively extinguish the right of redemption. Because the right will still exist, the investor will not be estopped to deny the effect of the deed absolute and escrow instructions.
How well did you know this?
1
Not at all
2
3
4
5
Perfectly
14
Q
  1. A man owned a four-story apartment building. The man borrowed $125,000 from his friend to make improvements. As consideration for the loan, the man executed a promissory note for $125,000 payable to the friend in one year and secured by a mortgage on the apartment building. The mortgage was dated January 1, 2008, and was recorded January 2, 2008. Thereafter, on February 1, 2008, the man executed a deed absolute on the apartment building and named the friend as grantee. This deed, although absolute in form, was intended only as additional security for the payment of the debt. In order to make judicial foreclosure unnecessary and to eliminate the right to redeem, the man then delivered the deed to an escrow agent in escrow with instructions to deliver the deed to the friend if the man failed to pay his promissory note at maturity.On January 1, 2009, the man failed to pay the note when it came due. The next day, the escrow agent delivered the deed to the apartment building to the friend. The friend then properly recorded this deed on January 3. One week later, on January 10, the friend conveyed the apartment building by warranty deed to an investor for the purchase price of $200,000. On January 12, the man tendered the $125,000 balance due to the friend, which he refused to accept. The man now brings an appropriate action against the friend and the investor to set aside the conveyance and to permit the redemption of the property by the man.Which of the following best states the man’s legal rights, if any, in his action against the friend and the investor?(A) The man has no rights against the investor, but the man does have an action for redemption against the friend for the value of the property.(B) The man has no rights against the friend, but the man does have an action for redemption against the investor for the value of the property.(C) The man has the option of seeking redemption against either the friend or the investor for the value of the property, but the man cannot set aside the conveyance.(D) The man has no rights against either the friend or the investor because he defaulted on the promissory note.
A
  1. (A) When the “mortgagee” under a deed absolute mortgage transfers to a bona fide purchaser, the mortgagorhas no rights against the bona fide purchaser, but he does have an action for redemption against the “mortgagee for the value of the land, or, at his election, the proceeds of the sale.” The theory is that the mortgagee now has the value of the Land in his hands as a separate fund, and such fund (as a substitute for the land) may be redeemed by the mortgagor. Applying this rule to our given set of facts, the man has no right against the investor, the bona fide purchaser, but he does have an action for redemption against the friend, the mortgagee. Choice (B) is incorrect. When the “mortgagee” under a deed absolute mortgage transfers to a bona fide purchaser, the mortgagor has no rights against the bona fide purchaser, but he does have an action for redemption against the “mortgagee for the value of the land, or, at his election, the proceeds of the sale.” The theory is that the mortgagee now has the value of the land in his hands as a separate fund, and such fund (as a substitute for the land) may be redeemed by the mortgagor. Applying this rule to our given set of facts, the man has no right against the investor, the bona fide purchaser, but he does have an action for redemption against the friend, the mortgagee. Because this choice states that the man has no rights against the friend for the proceeds that the friend is holding, it is incorrect. Choice (C) is incorrect. When the “mortgagee” under a deed absolute mortgage transfers to a bona flde purchaser, the mortgagor has no rights against the bona fide purchaser, but he does have an action for redemption against the “mortgagee for the value of the land, or, at his election, the proceeds of the sale.”The theory is that the mortgagee now has the value of the Land in his hands as a separate fund, and such fund (as a substitute forthe land) maybe redeemed by the mortgagor. Applying this rule to our given set of facts, the man has no right against the investor, the bona fide purchaser, but he does have an action for redemption against the friend, the mortgagee. However, the man does not have an election of rights with regard to the right of redemption; the man may only seek redemption against the friend and not the investor, the bona fide purchaser. Choice (D) is incorrect. Even though the man may have defaulted on the promissory note, he will still have an equitable right of redemption to reclaim the property or, in this case, the proceeds of the sale. However, the man may only seek redemption against the friend and not the investor, who took as a bona fide purchaser under the facts.
How well did you know this?
1
Not at all
2
3
4
5
Perfectly
15
Q
  1. A millionaire owned two adjacent 10-story commercial buildings. One building housed medical offices, and the other building housed dental offices. The first floors of both buildings were occupied by various retail establishments. The buildings’ other floors were rented to professionals and used as offices. There was an enclosed walkway that connected the second floor of each building. Thus, shoppers and office staff could walk across the common walkway and gain access to each building.While the buildings were being used in this manner, the millionaire sold the dental building to an investor by warranty deed, which made no mention of any rights concerning the walkway. The walkway continued to be used by the occupants of both buildings. Thereafter, the walkway became unsafe as a consequence of wear and tear.As a result, the investor hired a contractor to repair the walkway area. When the millionaire saw the contractor removing the carpeting along the walkway, he demanded that the investor discontinue the repair work. After the investor refused, the millionaire brought an action to enjoin the investor from continuing the work.The most likely result will be a decision for(A) the millionaire, because the investor does not have rights in the walkway.(B) the millionaire, because the investor’s rights in the dental building do not extend to the walkway.(C) the investor, because the investor has an easement in the walkway and an implied right to keep the walkway in repair.(D) the investor, because he has a right to take whatever action is necessary to protect himself from possible tort liability from persons using the walkway.
A
  1. (C) The investor would have an implied easement in the walkway. An implied easement is created and proved, not by the words of the conveyance, but by all the circumstances surrounding the execution of the conveyance. It is based on the intention of the parties as inferred from the surrounding circumstances. There are five distinct requirements for the existence of an implied easement, all of which are present in the facts of this example. First, there must be two properties owned by one person who uses one of the pieces of property to serve the other piece of Land. Second, there must be a conveyance of one part of the property to another person, the other part being retained by the conveyor. Third, the quasi-easement must be apparent at the time of the conveyance. Fourth, the quasi-easement must be continuous, which means that the use of the quasi-servient tenement must be permanently adapted to serve the needs of the quasi-dominant tenement. Fifth, the quasi-easement must be (a) “reasonably necessary” to the convenient enjoyment of the quasi-dominant land if that tract is the property conveyed to the grantee, and (b) “strictly necessary” to the enjoyment of the quasi-dominant tenement if that tract is retained by the grantor. By virtue of the implied easement, the investor has the right to enter the walkway for the purpose of repairing, maintaining, and improving the means by which the easement is enjoyed. Choice (A) is incorrect. The investor has an implied easement in the walkway. Since all of the five requirements of an implied easement are present in this question, the investor does have rights in the walkway. One of those rights is the right to enter the walkway to maintain and improve it. Choice (B) is incorrect. One of the rights that the investor does have is the right to use the walkway that connects his building to the other building. Because use of the walkway is reasonably necessary to the convenient enjoyment of the investor’s building, it is one of the investor’s rights. Choice (D) is incorrect. This choice suggests that the investor has unfettered discretion to do whatever is necessary to protect himself from tort liability. While that is something that the investor would want to be concerned with, he is not in a position to take any action he deems necessary. Because both the investor and the millionaire share the walkway, the investor cannot do anything to the walkway that would encroach upon the millionaire’s rights in this walkway because both parties are allowed reasonable use and enjoyment of the walkway.
How well did you know this?
1
Not at all
2
3
4
5
Perfectly
16
Q
  1. A grantor conveyed his property to his son “for life, remainder after (the son’s) death to his heirs.” Two years later, the son entered into a real estate agreement for the sale of the property to his friend, whereby the son agreed to convey the premises to his friend in fee simple absolute. Prior to the settlement date, the friend contacted the son, telling him that he would not perform his part of the agreement because the son could not convey a fee simple.If the Rule in Shelley’s Case is followed in this jurisdiction, the outcome of a suit by the son for specific performance of the real estate contract would result in(A) the son’s not succeeding, because he could not convey marketable title.(B) the son’s succeeding, because he had a fee simple to convey.(C) the son’s not succeeding, because his heirs have to join in the transaction in order to convey marketable title.(D) the son’s succeeding, because the son’s conveyance of his life estate to the friend divested the contingent remainder of his heirs.
A
  1. (B) The son would succeed because the Rule in Shelley’s Case operates as follows: If a life estate is conveyed to Ann and in the same instrument, a remainder is given to Ann’s heirs, then Ann will take a remainder in fee simple. In other words, Ann’s life estate merges with the remainder to her heirs, thus giving Ann a fee simple absolute. Choice (A) is incorrect. After application of the Rule in Shelley’s Case, the son will take a fee simple absolute, which would allow him to convey it to his friend. This would give the son marketable title to the property. Choice (C) is incorrect. After application of the Rule in Shelley’s Case, the son will take a fee simple absolute. Because the son has outright fee ownership, his heirs do not need to be joined in the transaction in order for the son to be able to convey marketable title. The son has marketable title all on his own because he takes all of the property in fee simple absolute. Choice (D) is incorrect. The son will succeed, but not for the reasons given in this choice. This choice suggests that the son’s conveyance of his own life estate to the friend was the reason that his heirs were divested of their remainder. The remainder was divested by application of the Rule in Shelley’s Case because it is followed in this jurisdiction, not from anything the son himself did. Also, because the Rule in Shelley’s Case operated to give the son a fee simple absolute, he was not conveying just a life estate to the friend; the son was conveying a fee simple interest.
How well did you know this?
1
Not at all
2
3
4
5
Perfectly
17
Q
  1. A grantor conveyed her mansion and surrounding property to her nephew “for life, remainder after (the nephew’s) death to his heirs.” Two years later, the nephew entered into a real estate agreement for the sale of the property to his brother, whereby the nephew agreed to convey the premises to the brother in fee simple absolute. Prior to the settlement date, the brother contacted the nephew, telling him that he would not perform his part of the agreement because the nephew could not convey a fee simple.The Rule in Shelley’s Case has been abolished by statute in this jurisdiction. Thus, the nephew’s prayer for specific performance would be(A) denied, because the Rule would not be triggered, thus creating only a life estate in the nephew.(B) granted, because the remainder in his heirs would become vested into a full fee in those heirs.(C) granted, because the nephew’s heirs receive a vested indefeasible interest in the property.(D) denied, because under the Doctrine of Worthier Title, at the termination of the nephew’s life estate, the grantor has a reversionary interest.
A
  1. (A) This choice is correct because since the Rule was abolished, the nephew would acquire only a life estate. As a result, in order to convey marketable title, the nephew’s heirs must join in the conveyance of the property. The remainder to the nephew’s heirs is treated as a contingent remainder that does not merge with the nephew’s life estate. Choice (B) is incorrect. This choice suggests that the remainder in the nephew’s heirs would become vested into a full fee in those heirs. However, that would not occur until the nephew’s death. Because the nephew is still alive on these facts, the heirs do not own a full fee. Choice (C) is incorrect. The nephew’s heirs received a contingent remainder in the property. A contingent remainder is any remainder created in favor of an existing but unascertained person. Because it is not clear who the nephew’s heirs will be upon his death, the “heirs” are unascertamed and the remainder is contingent, not vested. Moreover, this choice is incorrect because the nephew’s prayer for specific performance will not be granted; the nephew will not be able to compel the brotherto purchase the property because the nephew did not have a fee simple to grant. Choice (D) is incorrect. The Doctrine of Worthier Title (or the Rule against Remainders in the grantor’s heirs) is not at issue here. That rule arises in an example such as this: Al conveys to Bea for life, remainder to Al’s heirs. Under the Doctrine of Worthier Title, Bea would still have her life estate, and the remainder to Al’s heirs would become a reversion in Al himself.However, in this question, the remainder is going to the heirs of the life tenant, not the grantor. So, the Doctrine of Worthier Title is inapplicable here.
How well did you know this?
1
Not at all
2
3
4
5
Perfectly
18
Q
  1. A homeowner conveyed his property to his cousin “for life, with remainder to (the homeowner’s) heirs.” Later, the cousin conveyed the property to a farmer “for as long as he would continue to farm the property.” After the cousin’s death, the homeowner’s heirs brought suit against the farmer to quiet title. Which common law doctrine would be most applicable in the court’s determination of the ownership of the property?(A) Rule in Wild’s Case.(B) Doctrine of Destructibility of Contingent Remainders.(C) Doctrine of Worthier Title.(D) Rule against Remainders in the Grantees’ Heirs.
A
  1. (C) Choice (C) is correct because the old common law Doctrine of WorthierTitle is construed today as a rule of construction whereby the grantor presumes not to create a remainder in his heirs, but rather intends to retain a reversion in himself. Refer to Justice Cardozo’s opinion in the leading case of Doctorv. Hughes, 22 N.E. 211,1919. Choice (A) is incorrect. The Rule in Wild’s Case, when applied to a devise such as “to B and his children” will be construed to mean a life estate to B and a remainder to B’s children if B has no living children at the time of the devise. If B had children at the time of the devise, B and B’s children would be tenants in common. Note that the rule only applies to devises. In this question, the homeowner conveyed to the cousin so the Rule is inapplicable on these facts. Choice (B) is incorrect. This common law doctrine was applicable when a life estate had terminated and the holder of a contingent remainder had not met the condition, for whatever reason, If the remainder holder did not fulfill the condition by the time the life tenant died, the remainder was destroyed and the estate returned to the grantor permanently, even if that person eventually fulfilled the condition. The destructibility rule has been abolished in most but not all states. However, that is inapplicable here because the life tenant has not died, and reversion back to the grantor is not in issue here. Choice (D) is incorrect. The Rule against Remainders in the grantees’ heirs is more commonly known as the Rule in Shelley’s case. The Rule in Shelley’s Case operates as follows: If a life estate is conveyed to A and in the same instrument, a remainder is given to A’s heirs, then A will take a remainder in fee simple. In other words, A’s life estate merges with the remainder to his heirs, thus giving A a fee simple absolute. However, that Rule is inapplicable here because the original grant was “to (the cousin) for life, remainder to (the homeowner’s) heirs.” Because the remainder was given to the grantor’s heirs and not the life tenant’s heirs, the Rule in Shelley’s Case is not applicable.
How well did you know this?
1
Not at all
2
3
4
5
Perfectly
19
Q
  1. A farmer owned a 40-acre tract of farmland located in a small southern town. The farmer leased the property and building thereon to a tenant for a term of seven years commencing on February 15, 2000 and terminating at 12:00 noon on February 15, 2007. The lease contained the following provision:“Lessee covenants to pay the rent of $5,000 per month on the 15th day of each month and to keep the building situated upon said leased premises in as good repair as it was at the time of said lease until the expiration thereof.” The lease also contained a provision giving the tenant the option to purchase 10 acres of the tract for $150,000 at the expiration of the lease term. Before the lease was executed, the farmer orally promised the tenant that he (the farmer) would have the 10-acre tract surveyed.During the last year of the lease, the tenant decided to exercise the option to purchase the 10 acres of the tract. Without the farmer’s knowledge, the tenant began to build an irrigation ditch across the northern section of the property. When the tenant notified the farmer that he planned to exercise the option, the farmer refused to perform. The farmer also informed the tenant that he never had the 10-acre tract surveyed.If the tenant brings suit for specific performance, which of the following is the farmer’s best defense?(A) The option agreement was unenforceable under the parol evidence rule.(B) The farmer’s failure to survey the 10-acre tract excused him from further obligations under the contract.(C) The description of the property was too indefinite to permit the remedy sought.(D) The option was unenforceable because it lacked separate consideration.
A
  1. (C) Smith and Boyer note that no conveyance is valid unless the description of the land sought to be conveyed is sufficient to identify the land. The facts indicate that the tract is a 40-acre tract of farmland. The leasehold agreement provided that the tenant would have an option to purchase 10 acres of the tract. Because the lease failed to identify or describe a distinct piece of the tract, the farmer’s best argument is that the option should fail for lack of description. Law of Property, pg. 300. Choice (A) is incorrect. Under the parol evidence rule, evidence of a prior or contemporaneous agreement is inadmissible if it would vary or contradict the terms of a totally integrated writing. There is no need to consider extrinsic evidence here because the original lease agreement already contained a provision giving the tenant the option to purchase 10 acres of the tract. As a result, the parol evidence rule is inapplicable. Choice (B) is incorrect. The better defense here is that the description is so insufficient as to render the prior agreement unenforceable. It is no defense for the farmer to argue that something he himself orally promised (to have the tract surveyed) and then failed to do can excuse his performance under the contract. Because this choice fails to take into account the best reason why the lease fails to satisfy the statute of frauds (inadequate description), it is not correct. Choice (D) is incorrect. Even if the option to purchase had been fully supported by its own consideration, it still would have failed due to its lack of specificity. Choice (0) would be a poor defense because it fails to address the strongest reason why the suit for specific performance will fail—that the description was so insufficient as to render the agreement unenforceable.
How well did you know this?
1
Not at all
2
3
4
5
Perfectly
20
Q
  1. A landlord was the owner of a large, high-rise apartment building in a Midwestern city. On June 1, 2007, two tenants took possession of a three- bedroom apartment in the landlord’s building under a three-year lease at a rental of $1,200 per month. Their lease (as all other leases given by the landlord) contained the following provisions:“The term of this lease shall be three years from the date hereof as long as all the agreements herein shall be faithfully performed.”The two tenants lived in the apartment for two years. On June 10, 2009, however, a fire destroyed the apartment building. As a result, all the apartments in the building were rendered uninhabitable. After the two tenants were dispossessed from their apartment, the landlord brought suit against them to recover the rent due for the balance of the lease. The two tenants claim that they are no longer liable for rent or any other obligations under the lease. The landlord— tenants leasehold contract contained no provision regarding liability for fire.If the decision is in favor of the two tenants, it will most likely be because(A) there was nothing in the lease regarding liability for fire.(B) the two tenants did not own an interest in the property.(C) the jurisdiction has rejected the common law view on the tenant’s duty to pay rent.(D) the landlord did not contract to convey the property to the two tenants.
A
  1. (C) At common law, a tenant remains liable to pay rent even though because of fire, floods, storms, or other action of the eLements or otherwise, the property is rendered totally uninhabitable unless the lease otherwise provides. So, if the decision is in favor of the two tenants, it will most Likely be because the strict common law view has been rejected by this jurisdiction and another view has been adopted regarding tenant liability for rent. Though it is not clear that a court would come to this conclusion, the call of the question requires you to presume that the tenants would win and to find the best reason why the court could come to this conclusion. Choice (A) is incorrect. At common law, a tenant remains liable to pay rent even though because of fire, floods, storms, or other action of the elements or otherwise, the property is rendered totally uninhabitable unLess the lease otherwise provides. So, a lease provision allocating risk would be the most helpful to the tenants. However, the lease’s silence on liability does not automatically help the tenants either because if the jurisdiction still retained the common law view, the tenants would still be liable for rent. Because choice (C) explicitly states that the jurisdiction has rejected the common law view, it provides a stronger reason than choice (A). Choice (B) is incorrect. First, this choice is incorrect because a leasehold is considered an interest in land. Moreover, because the common law duty to pay rent was absolute, if the jurisdiction retained this rule, this argument still doesn’t explain why the tenants should be excused from paying their rent. Choice (D) is incorrect. The doctrine of equitable conversion is inapplicable. Note that the equitable conversion doctrine applies only when there is an enforceable obligation to sell land. So, even though this is a true statement on the facts, this argument still doesn’t explain why the tenants should be excused from paying their rent.
How well did you know this?
1
Not at all
2
3
4
5
Perfectly
21
Q
  1. A homeowner executed a deed by which he conveyed his home and surround property for a consideration of one dollar, receipt of which was acknowledged, “to my daughter for life, then to my aunt for life, but if my aunt moves to another state, to my sister for the life of my aunt, then to the heirs of my aunt if my aunt does not move to another state, and to the heirs of my sister if my aunt does move to another state.” This deed was promptly recorded.During the daughter’s lifetime, the aunt’s interest may best be described as a(A) contingent remainder.(B) shifting executory interest.(C) vested remainder subject to complete divestiture.(D) vested remainder subject to partial divestiture.
A
  1. (C) The aunt’s interest in the property would be a vested interest subject to complete divestiture. At the expiration of the daughter’s life estate, the aunt would immediately be entitled to take possession of the property. However, the aunt’s vested remainder (i.e., life estate in futuro in the property) would be subject to complete divestiture upon the contingency of her redomiciling. Although it may seem Like the aunt is only getting a life estate because of the language “then to my aunt for life,” the aunt is receiving a vested remainder because if she never moves, it goes to the aunt (stays with the aunt) and then passes to the aunt’s heirs. Choice (A) is incorrect. A contingent remainder is a remainder that is limited because it will depend on an event or condition that may not happen or may not be performed until after the termination of the preceding estate. ConverseLy, we are certain here that the daughter’s life estate will eventually terminate. Thus, although the aunt may predecease the daughter, thereby terminating the aunt’s own actual enjoyment, the aunt’s right to such enjoyment (i.e., vested remainder) is not uncertain. Because the property is designed to pass to the aunt (as long as the aunt abides by the one condition) the remainder is not contingent. Choice (B) is incorrect. A shifting executory interest “cuts short” or terminates a preceding estate in favor of another grantee, and it shifts the right of possession from one grantee to another upon the happening (or non-happening) of a particular contingency. In order for the aunt to take, she must wait for the daughter to pass away. There is no contingency that can allow the aunt to take away the daughter’s Life estate any earlier than the daughter’s death. Because of this lack of a contingency, the aunt’s future interest is a remainder, not an executory interest. Choice (D) is incorrect. A remainder is vested subject to being partly divested when the remainderman is in existence and ascertained, but the amount of her estate is subject to diminution in favor of other members of a class. This type of remainder, frequently called a remainder vested subject to open, is often illustrated by a class gift. Because the aunt is not part of a class that could dilute the aunt’s property interest, this is not the name of the estate the aunt was given.
How well did you know this?
1
Not at all
2
3
4
5
Perfectly
22
Q
  1. A grantor executed a deed by which he conveyed his apartment building for a consideration of one dollar, receipt of which was acknowledged, “to my son for life, then to my uncle for life, but if my uncle moves to another state, to my brother for the life of my uncle, then to the heirs of my uncle if my uncle does not move to another state, and to the heirs of my brother if my uncle does move to another state.” This deed was promptly recorded.During the son’s lifetime, the brother’s interest may best be described as a (an)(A) estate pur autre vie.(B) contingent remainder pur autre vie.(C) vested remainder pur autre vie.(D) shifting executory interest pur autre vie.
A
  1. (D) The brother’s interest would be a shifting executory interest purautre vie. An executory interest is an interest that divests the interest of another transferee (shifting executory interest) or that “follows a gap” or divests the interest of the transferor (springing executory interest). Because the brother’s interest could divest the uncle of the uncle’s interest in the property (if the uncle redomiciles), the interest is an executory interest of a shifting type. Another way to put this is that the brother can only take if the uncle meets (or fails to meet) a particular contingency, and that categorizes the brother’s future interest as an executory interest. Furthermore, his shifting executory interest would be pur autre vie, because it would be an estate for the life of another (the uncle). Choice (A) is incorrect. This choice is incomplete. All it states is that the brother received an estate pur autre vie. An estate pur autre vie means that the duration of the estate is measured by the life of someone other than the person receiving it. However, this choice still doesn’t tell us what type of estate or future interest was given to the grantee (i.e., life estate, shifting executory interest). Because this choice only states the duration without explaining the type of interest granted, it is incorrect. Choice (B) is incorrect. A remainder is a future interest created in a third person that is intended to take after the natural termination of a preceding estate. A contingent remainder is any remainder that is created in favor of an ascertained person but (a) is subject to a condition precedent, or (b) is created in favor of an unborn person, or (c) is created in favor of an existing but unascertained person. Choice (B) is incorrect because the brother’s interest in the property will not come after the natural termination of a preceding estate (like a life estate). Rather, the brother will have to wait to see if the uncle redomkiles in order for the brother to take the property. Also, when this contingency occurs, it will divest another grantee of the estate. Because the brother will not take after the natural termination of a preceding estate, his interest cannot be categorized as a remainder. Choice (C) is incorrect. A remainder is a future interest created in a third person that is intended to take after the natural termination of a preceding estate. A vested remainder is a remainder created in an ascertained and existing person that is not subject to any condition precedent except the normal termination of the preceding estate. Choice (C) is inco rrect because the brother’s interest in the property will not come after the natural termination of a preceding estate (like a life estate). Rather, the brother will have to wait to see if the uncle redomiciles in order for the brother to take the property. Also, when this contingency occurs, it will divest another grantee of the estate, Because the brother will not take after the natural termination of a preceding estate, his interest cannot be categorized as a remainder.
How well did you know this?
1
Not at all
2
3
4
5
Perfectly
23
Q
  1. In 1998, a landowner owned a 30-acre tract located just inside the city. The tract included the family home, a decaying antebellum mansion complete with tennis courts, stables and a smaller second house that was once occupied by tenants who farmed the city. The second house, however, had long been vacant as a result of the economic decay of the surrounding area.Prosperity burst upon the city in 1999, and the landowner began selling acre lots in the tract. By 2006, the landowner had sold 25 acres, retaining five acres that included the antebellum mansion, tennis courts, stables, and the former tenants’ house.On May, 19, 2007, the landowner entered into a valid written contract with a buyer. According to the terms of their agreement, the landowner agreed to sell and convey his remaining interest in the tract for a consideration of $500,000. The land sale contract provided a closing date of November 19, 2007 and stipulated that “time was of the essence.”On July 2, 2007, a fire destroyed the antebellum mansion. The landowner had the mansion insured for $450,000 against fire loss and collected that amount from the insurance company. At the closing on November 19, the buyer tendered a cashier’s check for $50,000 and demanded a deed conveying a fee simple interest in the property. Conversely, the landowner tendered a deed of conveyance and demanded the full purchase price of $500,000. The buyer refused the landowner’s demand.In an appropriate action for specific performance against the buyer, the landowner demanded $500,000. If the landowner prevails, which of the following is the best rationale for the outcome?(A) The fact that the antebellum mansion was insured for $450,000 is irrelevant.(B) The landowner and the buyer each had an insurable interest in the property.(C) The doctrine of equitable conversion has been abolished.(D) The doctrine of equitable conversion requires such a result.
A
  1. (D) This is exactly the same situation that Smith and Boyer discuss in their hornbook. An executory contract for the sale of land requiring the seller to execute a deed conveying legal title upon payment of the full purchase price works an equitable conversion in order to make the purchaser the equitable owner of the land and the seller the equitable owner of the purchase price. The result is that the purchaser, as equitable owner of the land, takes the benefit of all subsequent increases in value and, at the same time, becomes subject to all losses not occasioned by the fault of the seller. Thus, the purchaser, to protect himself, either must procure his own insurance, or by appropriate provision in the contract, cast the risk upon the seller. He is not, however, entitled to recover insurance payments payable to the vendor. Choice (D) is a better answer than (A) because the equitable conversion doctrine (which places the risk on the buyer) requires such a result. Choice (A) is incorrect. Though it could be argued that the court awarded the landowner the $500,000 because the court chose to ignore the insurance policy, this choice still does not explain why this particular result was required. The question of why the court still granted the landowner his $500,000 still remains. It is because the court followed the common Law default doctrine of equitable conversion that the landowner was stiLL awarded the purchase price. Though choice (A) may be factually correct and the court may have chosen to ignore the insurance policy, choice (D) is preferred because it states the correct rule of law that the court followed to reach this result. Students wiLl want to choose a correct statement of law over one of fact because a correct rule of law is more universal and can apply to many factual situations. A correct statement of fact, however, may only be applicable to one factual scenario. Choice (B) is incorrect. Even if both the landowner and the buyer each had an insurable interest in the property, this still does not explain why the landowner was awarded the fulL $500,000 by the court. It is because the court followed the common law default doctrine of equitable conversion that the Landowner was still awarded the purchase price. Though (B) may be factuaLLy correct and both parties had an insurable interest in the property, choice (D) is preferred because it states the correct rule of law that the court foLlowed to reach this result. Choice (C) is incorrect. This is the exact opposite of the correct answer and would be a reason NOT to award the landowner the $500,000.
How well did you know this?
1
Not at all
2
3
4
5
Perfectly
24
Q
  1. On November 1, Beeson contracted to purchase from Sloan for $250,000 certain property located in the City of La Mirada. In the contract of sale, Beeson and Sloan agreed that the property was to be used for the purpose of building a commercial shopping mall. The contract required Beeson to pay Sloan a deposit of $12,500, with the balance of the purchase price payable at closing a month later on December 1. On November 24, the city council rezoned the property so that it could be used only for single-family residential purposes.As a consequence, Beeson refused to honor the contract. Sloan now brings an action for specific performance arguing that the doctrine of equitable conversion places the loss on the buyer. Beeson argues that to enforce the contract would be harsh and oppressive to him.If judgment is for Beeson, it will most likely be because(A) Sloan assumed the risk.(B) Sloan would be unjustly enriched.(C) legal title remained in Sloan.(D) equity will relieve Beeson of a bad bargain.
A
  1. (B) Under the doctrine of equitable conversion, the risk of loss from casualty and other fortuitous events is normally placed on the purchaser in the absence of controlling provisions in the contract. Equity thus considers the vendee as the owner of the land and the vendor as the owner of the purchase money. Smith and Boyer point out, however, that this rule is limited in its application to cases where the intention of the parties will not produce an inequitable result. For exampLe, assume that A contracts to seLl to B a certain piece of land that was to be used for the purpose of erecting a hotel. However, between the time the contract of sale was made and the time for delivery of the deed, the city council rezones the Lot so that it could only be used for residential purposes. A now brings suit for specific performance. In this situation, Smith and Boyer note that the granting of specific performance would be unduly harsh and oppressive to B. Because the intent of the parties was defeated by the supervening event, specific performance should be denied. However, the denial of specific performance does not end the matter, but, instead, the vendor may proceed against the purchaser in a suit at Law for any damages that have been incurred, but the purchaser does not have to be required to tender the purchase price of the property. By analogy, if the court rules in favor of Beeson, it will be to avoid unjust enrichment. Choice (A) is incorrect. Under the doctrine of equitable conversion, the risk of loss is normally pLaced on the buyer of the property, absent any controlling provisions in the contract. Here, Sloan (as the seller) did not assume the risk of the supervening event that the city council would rezone the property. If anyone assumed the risk of any unforeseen events, it would have been Beeson (as the buyer). Choice (C) is incorrect. Under the doctrine of equitable conversion, legal title to the property remains with the seller, and the buyer is treated as the equitable owner of the land. Given that this is the traditional definition, this is the reason why Beeson should be required to buy the property. Stating that Sloan retains legal title only enforces the idea that Beeson should be forced to retain a bad bargain. Because this choice fails to explain why Beeson should prevail, it is incorrect. Choice CD) is incorrect. This choice falsely suggests that the court will inquire into the imprudence of the bargain struck between the two parties. It is not for the court to decide whether this was a good bargain or one formed from bad judgment. However, the court (to do equity) may avoid unjust enrichment by refusing to grant specific performance to the seller and rescind the contract of sale. However, it would be due to the supervening illegality of the zoning ordinance, not simply because itwas a bad bargain looking backwards.
How well did you know this?
1
Not at all
2
3
4
5
Perfectly
25
Q
  1. A state has the following recording statute in effect:“No conveyance is good against a subsequent purchaser for a valuable consideration and without notice, unless the same be recorded prior to subsequent purchase.”An owner is the fee simple the owner of a 20-acre tract of unimproved land, situated in the state. On May 1, the owner sold the tract to a buyer for the purchase price of $40,000 under a quitclaim deed. The owner delivered the deed to the buyer who did not record the deed. After the sale to the buyer, the owner found himself in desperate need of money because he lost his job. So the owner, in consideration of the sum of $75,000, conveyed the tract to an investor by warranty deed. This transaction took place on August 1. When the investor acquired title to the tract, he had no actual knowledge of the buyer’s deed (which was still unrecorded). On August 10, the buyer recorded his deed to the tract. The investor did not record the deed he received from the owner until August 15.In an appropriate action to quiet title to the tract, in which all interested parties have been joined, title will be found to be in(A) the buyer, because his deed preceded the investor’s deed.(B) the buyer, because his deed was recorded prior to the investor’s deed.(C) the investor, because he is protected by the recording statute.(D) the investor, because he took title by warranty deed and the buyer took title by quitclaim deed.
A
  1. (C) The keyto this question is carefully interpretingthe wording of the recording statute. According to the statutory Language, the state’s recording act protects subsequent bona fide purchasers for value and without notice. As such, this is an example of a pure “notice” type recording statute, which generally provides that an unrecorded conveyance or other instrument is invalid as against a subsequent bona fide purchaser forvalue and without notice. Under this type of recording statute, the subsequent bona fide purchaser prevails over the prior interest whether the subsequent purchaser records ornot. As a consequence, the investor will prevail over the buyer because the investor was a subsequent bona fide purchaser without notice of the buyer’s deed. Choice (A) is incorrect. Even though the buyer’s deed preceded the investor’s deed, that fact will not be enough for the buyer to prevail. Because the state has a notice recording statute, a subsequent purchaser for valuable consideration and without notice can still prevail, even though someone else received a prior deed. Choice (B) is incorrect. While this is true, the test isn’t who recorded first; rather, the test is whether the subsequent purchaser for valuable consideration purchased without notice. Because the buyer’s deed remained unrecorded when the investor purchased the property and because the investor had no actual notice of the grant to the buyer, the investor will prevail in a notice jurisdiction, even though the buyer ultimately recorded before the investor. Choice (D) is incorrect. A quitclaim deed is a deed that makes none of the six covenants of title. Whether a grantee took title via a quitclaim or warranty deed is only relevant when a grantee is suing a grantor under a deed covenant. Because the competition here is between two grantees, the only relevant inquiry is whether or not the investor had notice when he purchased from the owner. The fact that the investor has covenants in his deed will not help him if he had either actual or constructive notice of the grant to the buyer. Because this choice falsely suggests that deed covenants would be the reason why the investor prevails over the buyer, it is in correct.
How well did you know this?
1
Not at all
2
3
4
5
Perfectly
26
Q
  1. A deed executed by a grantor conveyed his property for a consideration of one dollar, receipt of which was acknowledged, “to my brother for life, then to the heirs of my brother.” A life interest in the property for the life of the brother is worth $20,000 on the date of the conveyance. The total worth of the property is $50,000. The brother accepted but didn’t record the deed. The recording statute in this jurisdiction provided “unless recorded, all written instruments affecting title to land are void as to subsequent purchasers who paid value and without notice.”Four years later, the grantor purported to convey his property in fee simple absolute to his two sons, by a warranty deed, as a gift. The two sons recorded the deed. Shortly thereafter, the brother ascertained that the grantor’s sons were about to take possession of the property. As a consequence, the brother promptly recorded his deed.In a dispute between the brother and the grantor’s children as to the ownership of the property, if the brother prevails it will be because(A) the brother paid valuable consideration for his deed.(B) the brother recorded his deed before the grantor’s children sought to oust him from the land.(C) the grantor’s sons are not protected by the recording statute.(D) the grantor’s knowledge is imputed to his children.
A
  1. (C) In order to be a bona fide purchaser protected under the recording act, one must (a) be subsequent; (b) pay value; (c) be without notice (the value must have actually been paid before notice); and (d) be of good faith. Recording statutes do not protect a subsequent claimant who has not paid more than a nominal consideration because he is nota purchaser. Therefore, in our case, the grantor’s children are not protected by the recording statute because they are not purchasers (as the grantor purportedly conveyed the property to them as a gift). The brother will ultimately prevail because he was the first in time to take the property from the grantor. Choice (A) is incorrect. In order to be a bona fide purchaser protected under the recording act, one must (a) be subsequent; (b) pay value; (c) be without notice (the value must have actually been paid before notice); and (d) be of good faith. In this type of situation, part of the inquiry would be whether or not the subsequent grantees paid valuable consideration because if a subsequent grantee paid valuable consideration and took without notice, then that person would prevail even though someone else had already been granted the property. However, this type of inquiry only matters to subsequent grantees. Because the brother was the first grantee, the manner in which the brother took the property is irrelevant. It makes no difference whether the brother paid or it was given to him as a gift, because if the subsequent grantees took without nottce and paid valuable consideration, the brother would lose even if he paid. Because this answer choice falsely suggests that the brother’s payment of consideration is the reason he prevails, it is incorrect. Choice (B) is incorrect. An ouster occurs where one co-tenant manages to wrongfully exclude her co-tenants from possession of the property. When one co-tenant ousts her cotenant from possession, the ousted tenant has a cause of action against the possessor to regain possession for herself with the possessor. In this fact pattern, there is no co-tenancy relationship between the brother and the grantor’s children where ouster would be a viable issue to raise. The brother prevails because the sons cannot claim protection under the recording statute, not because the brother recorded before there was any ouster. Choice (D) is incorrect. If the sons had actual knowledge of the transaction to the brother or if there were record notice (assuming the brother did record first), then the children would possess their own knowledge of the prior transaction. However, the grantor’s knowledge of the prior transaction to the brother is not necessarily imputed to the grantor’s children simply because they are the grantor’s children; the children have to know about it in their own right.
How well did you know this?
1
Not at all
2
3
4
5
Perfectly
27
Q
  1. A father died leaving a will by which he devised a 100-acre tract to his daughter. At the time of the father’s death, he believed that he owned all of the 100-acre tract. However, the father actually owned 95 acres of the tract. The other five acres were owned by a farmer. After taking possession of the tract, the daughter executed and delivered a warranty deed to a buyer, purporting to convey all 100 acres of the tract. The agreed purchase price was $100,000. According to the terms of the sale, the buyer paid the daughter a down payment of $25,000, with the unpaid portion of the purchase price (i.e., $75,000) secured by a mortgage. The mortgage instrument described the property interest as covering all 100 acres of the tract.After the daughter—buyer transaction was completed, the farmer came forward and informed the parties that she was the true record owner of five acres of the 100-acre tract. Upon the farmer’s threat to sue to quiet title, the daughter negotiated a deal whereby she paid the farmer $5,000 for the five-acre tract. As part of their agreement, the farmer executed and delivered a quitclaim deed quitclaiming to the buyer all of her interest in the five-acre parcel.Thereafter, the buyer defaulted on the mortgage debt, and the daughter properly consummated foreclosure proceedings, becoming the purchaser of the tract at her own foreclosure sale. The description of the deed in the foreclosure sale referred to the tract as consisting of all 100 acres. After the foreclosure sale was finalized, the buyer brought suit claiming title in fee simple to the five-acre tract, formerly owned by the farmer.In an appropriate action to determine the title to the said five-acre tract, the buyer will(A) lose, because the daughter did not have good and marketable title at the time she purported to convey the 100-acre tract to the buyer.(B) lose, because the doctrine of after-acquired title controls.(C) win, because the deed from the farmer to the buyer was a quitclaim deed.(D) win, because the quitclaim deed from the farmer to the buyer was subsequent to the deed from the daughter to the buyer and to the mortgage.
A
  1. (B) Under the doctrine of estoppel by deed (sometimes referred to as the “after-acquired title” doctrine), when a person executes a deed purporting to convey an estate in Land that he does not have (orwhich is largerthan he has), and such person at a later date acquires such estate in such land, then the subsequently acquired estate will, by estoppel, pass to the grantee. In this example, once the daughter purchased the five-acre tract from the farmer, title inured to the benefit of the buyer as evidenced by the farmer’s quitclaim deed to the buyer. Most importantly, the daughter held a mortgage on the 100-acre tract. The mortgage instrument described the property interest as covering all 100 acres of the tract. Thus, when the buyer defaulted and the daughter foreclosed, she purchased back the entire tract of land (including the five acres previously owned by the farmer). Choice (A) is incorrect. This choice is wrong because even though the daughter did not have good and marketable title to all of the tract initially, this title defect was subsequently cured by operation of the daughter—farmer transaction. Choice (C) is incorrect. At the moment the daughter purchased and acquired title to the five acres from the farmer, title automatically inured to the benefit of the buyer by virtue of the estoppel-by-deed doctrine. It is irrelevant whether the farmer quitclalmed the property to the buyer. This is tricky because in most states estoppel by deed will not be applied where the conveyance is by a quitclaim deed. In this question, however, the daughter—buyer conveyance was by warranty deed. For this reason, the doctrine of estoppel by deed will apply. In other words, title would nonetheless inure to the buyer’s benefit regardless of the form of deed used. Choice (D) is incorrect. All this choice does is mention some of the facts that occurred subsequent to the deed from the daughter to the buyer and the mortgage. Certainly, the quitcLaim deed from the farmer to the buyer came after the daughter—buyer deed and the mortgage. However, this choice does not explain why those facts are significant to either party. Moreover, this choice states that the buyer will prevail when, in fact, the buyer will not prevail due to the doctrine of estoppel by deed.
How well did you know this?
1
Not at all
2
3
4
5
Perfectly
28
Q
  1. A landlord owns a three-story building. The landlord leased the building and the surrounding property to a tenant for a term of six years, commencing on December 1, 2003 and terminating on November 30, 2009.On April 15, 2004, a neighbor, who owned an adjoining parcel of land, was burning leaves and underbrush on his land. There was a high wind blowing in the direction of the land that the landlord leased to the tenant, and the wind carried the burning leaves onto the tenant’s property. The building caught fire and caused $150,000 in fire damages. Because of the fire, the tenant has not occupied the leased premises since the date of the fire because the building was rendered uninhabitable.This state is a common law jurisdiction. In addition, the controlling Statute of Limitations for initiating an action for damages due to negligence is five years from the time the cause of damage occurs.On May 1, 2004, the landlord brings suit against the tenant asserting breach of contract for failing to pay the rent. Judgment should be for(A) the landlord, because a tenant remains liable to pay rent, even though as a result of the fire, the property was rendered uninhabitable.(B) the landlord, because the covenant to pay rent runs with the land.(C) the tenant, because it would be inequitable to enforce the rental provision in the lease because the premises was rendered uninhabitable.(D) the tenant, because an increasing number of states have enacted statutes relieving a tenant of his obligation to pay rent where the premises are rendered uninhabitable by fire or other acts of God.
A
  1. (A) The tenant is still liable to the landlord for the monthly rental despite the fact that he can no longer reside in the building. At common law, the general rule is that a tenant remains liable to pay rent even though because of fire, floods, or other unforeseen action, the property is rendered totally uninhabitable. Please note that it is possible for the parties to provide in a lease for certain excuses for non-payment of rental; but in our hypothetical, no such provision was made. Choice (B) is incorrect. The covenant to pay rent does run with the land, making this a correct statement. However, this choice does not explain why the tenant is still required to pay rent even though the premises were so destroyed by fire as to cause the tenant to move out. Because this choice provides less of an explanation of why the landlord would prevail, it is a weaker choice, and choice (A) is a stronger reason why the landlord would prevail. Choice (C) is incorrect. Even though the premises were rendered uninhabitable, the tenant’s duty to pay rent is still absolute in spite of the destruction to the premises. Though it may be inequitable to compel the tenant to pay, that will not be part of the inquiry here. In this breach of contract situation, the tenant will be required to pay the rent and will not prevail. Choice (D) is incorrect. Even assuming it were true that an increasing number of states have enacted such statutes, this jurisdiction is not one of them. The facts state that this is a common law jurisdiction.
How well did you know this?
1
Not at all
2
3
4
5
Perfectly
29
Q
  1. A landlord owns a two-story building. The landlord leased the building and the surrounding property to a tenant for a term commencing on December 1, 2006 and terminating on November 30, 2008. Their leasehold agreement contained the following provisions: “The tenant covenants to pay the rent of $500 per month on the first day of each month and to keep the building situated upon leased premises in as good repair as it was at the time of said lease until the expiration thereof.”On April 15, 2007, the tenant had her annual “Tax Time Party” where she and her friends would all walk to the post office together to mail their taxes and then have drinks and dinner at the tenant’s apartment to celebrate. After a few drinks, one of the tenant’s friends became belligerent toward one of the partygoers who challenged some of the friend’s deductions. The friend charged at the other partygoer and crashed into the wall after the partygoer got out of the way. The crash caused extensive damage to one of the walls in the apartment.This state is a common law jurisdiction. In addition, the controlling Statute of Limitations for initiating an action for damages due to negligence is five years from the time the cause of damage occurs.In an appropriate action to construe the tenant’s obligation under the covenant to keep the premises in repair, which of the following is the most accurate statement?(A) The tenant is liable for normal wear and tear under the covenant to repair.(B) The tenant is liable under such a covenant for all defects, including the damage to the wall.(C) The tenant’s covenant to repair is void as against public policy, because the landlord is under an affirmative duty to make repairs on the demised premises.(D) The tenant’s duty to keep the premises in good repair relieves the landlord of his obligation to disclose the existence of any latent defects.
A
  1. (B) The general rule provides that a tenant who covenants to keep the leasehold premises in good repair is liable under such a covenant for all defects regardless of their cause. As a result, the tenant remains liable for any defects that she caused or that were caused by third persons. Choice (A) is incorrect. At common law, such a covenant in a lease required the tenant to repair more than just the ordinary wear and tear of the premises. Because the covenant includes the promise to keep the premises in good repair, this would include the duty to make the premises the same as they were the day that the landlord delivered them to the tenant. This includes fixing the wall and not just ordinary wear and tear. Choice (C) is incorrect. At common law, the tenant is responsible for such damage when the tenant makes such a covenant. It would then be irrelevant for the tenant to raise the issue of the covenant beingvoid as against public policy because the landlord was never under any such duty to fix the wall. Harsh as the common law rule might be, this argument will still be unpersuasive. Choice (D) is incorrect. A landlord may be liable in tort to the tenant, her guests, licensees, and invitees if, at the commencement of the Lease, there is a dangerous condition that the landlord knows or should know about and the discovery of which would not likely occur by the tenant exercising due care. The tenant’s taking on this covenant does not relieve the landlord of this obligation, so this statement is incorrect. Moreover, this choice focuses on the wrong issue, making it incorrect.
How well did you know this?
1
Not at all
2
3
4
5
Perfectly
30
Q
  1. A farmer, being fee simple owner of a farm, devised it to his niece and her husband as tenants by the entirety. The niece and her husband took immediate possession of the farm and lived there with their daughter and son. Thereafter, the husband died in an automobile accident. In 1972, two years after her father’s death, the daughter moved to another state.The son lived with his mother on the farm until her death intestate in 1980. The son continued in exclusive possession of the farm until his death in 2001. In his will, the son devised the farm to a local charity. When he was alive, the son was unaware that his sister was still alive and that title to the farm had descended to the two of them as their mother’s sole surviving heirs. Since his mother’s death in 1980, the son has held himself out as the owner of the farm, maintaining it and paying all of the taxes on the property. The sister had not communicated with either her mother or her brother since her redomiciling in 1972. The jurisdiction in which the farm is located has a 20-year limitation period for the acquisition of property by adverse possession.What interest, if any, does the sister have in the property?(A) None, because of her own laches.(B) None, because the brother acquired title to the farm by adverse possession.(C) An undivided one-half interest because the brother’s possession was not adverse to her title.(D) An undivided one-half interest because the 20-year limitation period did not run against her because she was unaware of the brother’s exclusive possession.
A
  1. (C) Here, the brother and sister acquired title to the farm as tenants in common (by descent from their mother). In a tenancy in common, each tenant owns an undivided fractional part of the property and none owns the whole (as in joint tenancy). The brother will not own the property outright because the brother’s conduct never amounted to an ouster of his sister. An ouster occurs where one co-tenant manages to wrongfully exclude her co-tenants from possession of the property. When one co-tenant ousts from possession her co-tenant, the ousted tenant has a cause of action against the possessor, not to put her out, but to regain possession for herself with the possessor. Where one co-tenant stakes a claim for exclusive possession of any part of the property, the claim alone may amount to an ouster. The brother’s living on the property alone will not be enough to oust the sister of her undivided one-half interest because the brother’s possession was not adverse to his sister’s title. Choice (A) is incorrect. Laches is a defense to any action brought in equity. If a party had waited an unreasonably long period of time in which to bring a claim, the other party may argue that he has relied upon that lack of assertion of rights to his detriment. However, nothing in the facts suggests that the brother has, in fact, relied upon his sister’s failure to enforce her rights to the farm, which has led to his detriment. Choice (B) is incorrect. The brother’s possession of the farm was not hostile and adverse because he never, in fact, ousted his sister. Choice (D) is incorrect. This falsely suggests that the 20-year period would have started running if the sister became aware of her brother’s exclusive possession or somehow learned of it. However, the brother’s exclusive possession would still not start the clock running if there had been no affirmative ouster of the sister. Because this choice focuses too much on the sister’s awareness and not on the brother’s conduct, it is incorrect.
How well did you know this?
1
Not at all
2
3
4
5
Perfectly
31
Q
  1. A landlord is the owner in fee simple of a tract of land on which is situated a large office building. The landlord leases the land and building thereon to a tenant for a term of seven years, commencing on August 1, 2001, and terminating at midnight on July 31, 2008. The lease contains the following provisions“(1) The tenant covenants to keep the building on the leased premises insured against fire in the amount of $100,000.”After three years of the lease had expired, the tenant assigned the entire balance of the lease period to a doctor, who took immediate possession of the leased property. Then in 2005, the doctor assigned his leasehold interest to a dentist. The dentist went into possession, but failed to pay any rent for two months. Soon afterwards, a fire spread from an adjoining building to the leased property, completely destroying the building thereon.While the tenant was in possession of the leased property, he carried a fire insurance policy on the premises in the amount of $100,000. However, the tenant allowed the policy to lapse after his assignment to the doctor. The doctor did carry insurance on the leased building, but only in the amount of $50,000. When the dentist took possession of the building, he did not obtain any fire insurance.The landlord learned that the building was not insured at the time of the fire as per the original agreement. In an action by the landlord against the tenant to recover for the fire loss, the landlord will most probably(A) recover, because in accordance with the Rule in Spencer’s Case, the covenant to maintain fire insurance would “touch and concern” the land.(B) recover, because the tenant’s obligation to maintain fire insurance did not terminate after his assignment to the doctor.(C) not recover, because the covenant to provide fire insurance did not run with the land.(D) not recover, because the dentist, as assignee, would be liable for the fire loss.
A
  1. (B) A covenant in a lease to pay (fire) insurance is held to “run with the land.” In this regard, a covenant to pay insurance is capable of running if and only if the landlord is bound to use the proceeds for repair or replacement. It is important to understand that the tenant’s assignment of his leasehold interest to the doctor did not relieve or extinguish the tenant’s contractual obligations under his lease with the landlord. In short, an assignment does not release the tenant from his contract obligations to the landlord under the terms of this original leasehold agreement. To be sure, a lease is a contract as well as a conveyance. Choice (A) is incorrect. The Rule in Spencer’s Case provides that an assignee of the leasehold estate cannot be held liable for breach of covenant if the covenant is of a type that “does not touch and concern the land.” However, the tenant still has a contractual obligation to the landlord to buy proper fire insurance based on the original covenant in the lease. So, even though this covenant will run to successive assignees, the tenant can still be liable. This choice focuses more on why successive assignees would be liable, but does not explain why the tenant is still liable. Thus, it is not as complete as choice (B). Choice (C) is incorrect. Covenants to insure the buildings on leased premises do, in fact, run with the land. Because this choice states otherwise, it is incorrect. Choice (D) is incorrect. The dentist may be liable in his own right because the covenant does run with the land. However, in stating that the landlord will not recover against the tenant, this choice suggests that the tenant is not liable for the failure to insure properly. Because the tenant is still liable under the original lease, the landlord will be able to recover against the tenant.
How well did you know this?
1
Not at all
2
3
4
5
Perfectly
32
Q
  1. A homeowner gave his friend a mortgage on his property to secure a loan from the friend to the homeowner in the amount of $100,000. The homeowner then gives an investor a mortgage on the property to secure a loan from the investor to the homeowner in the amount of $80,000. The friend then records his mortgage. The homeowner then gives a bank a mortgage on the property to secure a loan from the bank to the homeowner in the amount of $50,000. The investor then records. The bank then records.The homeowner lost his job and was unable to pay the mortgages on the property. In a foreclosure proceeding in which the friend, the investor, and the bank were parties, the property sold for $160,000. The recording statute in this jurisdiction provided “unless recorded, all written instruments affecting title to land are void as to subsequent purchasers who paid value and without notice.”What dollar amount, if any, should go to the friend?(A) $0.(B) $30,000.(C) $80,000.(D) $100,000.
A
  1. (C) A mortgage is defined as an interest in land created by a written instrument providing security for the performance of a duty or the payment of a debt. A mortgage should be and is generally recorded. A foreclosure proceeding is the method by which the security (i.e., the property), or proceeds for the sale thereof, is applied to the satisfaction of the debt. A mortgagee has both an in personam claim against the mortgagor and an in rem action against the security. In these facts, all three mortgagees (the friend, the investor, and the bank) are involved in a foreclosure proceeding for the proceeds of the sale. Priority as to these proceeds will be determined by such factors as order of recordjng and notice of prior mortgages, depending on the applicable recording statutes in the jurisdiction. In a pure notice jurisdiction, a subsequent bonafide purchaser is protected if the mortgagee of a prior mortgage has not recorded at the time when the subsequent bonafide purchaser takes his mortgage, regardless of who records first. Here, the bank took its mortgage after the friend had already recorded. Therefore, the bank has notice and will not prevail. The investor will prevail, however, because she was the last BFP without notice of any prior recordation, because at the time the investor took her mortgage, the friend had not yet recorded. The investor will then get $80,000 from the proceeds to satisfy her debt, leaving $80,000 from the sale price of $160,000. Although $80,000 will not fully satisfy the friend’s interest, he will be entitled to receive the rest of proceeds and the bank takes nothing. Choice (A) is therefore incorrect, because it is the bank that will take nothing. Choice (B) is incorrect. The only apparent significance of this dollar amount is that it represents the difference between the investor’s mortgage (of $80?000) and the bank’s mortgage (of $50,000). The friend would be entitled to the rest of the purchase price after the investor is fully compensated. There is no reason why the friend would be limited to only $30,000 of the remaining $80,000. Choice (D) is incorrect. The property sold for $160,000. Because the investor is the last BFP to take without notice, she will be paid first from the sale proceeds. Because this leaves only $80,000 remaining, that is the most that the friend can receive. The friend is not entitled to receive the full $100,000 as stated in this choice because the friend will not take first. As a result, his debt will not be fully satisfied.
How well did you know this?
1
Not at all
2
3
4
5
Perfectly
33
Q
  1. A landlord is the owner in fee simple of a tract of land on which is situated a large office building. The landlord leases the land and building thereon to a tenant for a term of seven years, commencing on August 1, 2001, and terminating at midnight on July 31, 2008. The lease contains the following provisions:“(1) The tenant covenants to pay the rent of $750 per month on the first day of each month.”After three years of the lease had expired, the tenant assigned the entire balance of the lease period to an attorney, who took immediate possession of the leased property. Then in 2005, the attorney assigned his leasehold interest to a doctor. The doctor went into possession, but failed to pay any rent for two months.After the doctor failed to make his rental payments for the first two months of his lease, the landlord brought suit against the tenant to recover for the unpaid rent.Judgment should be for(A) the landlord, because the tenant’s contractual obligation under the lease survived the assignments.(B) the landlord, because he did not object to the assignments.(C) the tenant, because the doctor, as assignee, would only be held liable.(D) the tenant, because his assignment to the attorney constituted a novation, thereby extinguishing his obligation to pay rent.
A
  1. (A) Where a tenant—assignor transfers all of his leasehold interest to an assignee, such assignment does not release the original tenant from his contract obligations to the landlord under the terms of the lease. This is true even when the assignee, thereafter, assigns/transfers his leasehold interest to a sub-assignee. Choice (B) is incorrect. The landlord prevails here not because he didn’t object to the assignments, but rather because the tenant is still primarily liable under the terms of the original lease. Because privity of contract still exists between the tenant and the landlord under the original lease, the landlord can bring suit against the tenant. Choice (C) is incorrect. Where a tenant—assignor transfers all of his leasehold interest to an assignee, such assignment does not release the original tenant from his contract obligations to the landlord under the terms of the lease. Choice (D) is incorrect. The assignment to the attorney did not constitute a novation, but rather an assignment of the balance of the leasehold interest. A novation is a complete substitution of one party for another in a contract. An assignment alone will not effectuate a complete substitution of parties. Without express language that the substitution of the attorney would completely discharge the tenant from all prior duties, the tenant still remains liable for the rent under the original lease agreement.
How well did you know this?
1
Not at all
2
3
4
5
Perfectly
34
Q
  1. A landlord is the owner in fee simple of a tract ofland on which is situated a studio. The landlordleases the land and building thereon to a tenant fora term of seven years, commencing on August 1,2001, and terminating at midnight on July 31, 2008.The lease contains the following provisions:“(1) The tenant covenants not to assign the leased premises without the consent of the landlord.”After three years of the lease had expired, the tenant, without the consent of the landlord, assigned the entire balance of the lease period to a dance instructor, who took immediate possession of the leased property. The landlord accepted rental payments from the dance instructor. Then in 2005, the dance instructor assigned his leasehold interest to a yoga instructor who went into possession.Assume for the purposes of this question only that the landlord, after learning of the dance instructor’s assignment to the yoga instructor, brings suit against the dance instructor to have the assignment declaredvoid.The landlord will most likely(A) succeed, because the original leasehold agreement prohibited assignments.(B) succeed, because the covenant prohibiting assignments did not run with the land.(C) not succeed, because in accordance with the Rule in Dumpor’s Case, where the landlord consents to one transfer, he waives his right to avoid future transfers.(D) not succeed, because the privity of estate between the dance instructor and the landlord terminated when he assigned the leasehold to the yoga instructor.
A
  1. (C) In accordance with the Rule in Dumpor’s Case, if a landlord grants consent to one transfer (e.g., the tenant’s assignment to the dance instructor), he waives his right to avoid future transfers, assigns, or subleases in violation of a prohibition in the lease against such transfers. By the same token, if the lessee/assignee pays rent to the landlord and such rent is accepted by him, in spite of the fact that the transfer was in violation of the lease, the landlord will be deemed to have waived his right to avoid the transfer. Choice (A) is incorrect. Certainly, the original provision limited the ability to assign. However, when the landlord accepted rent from the dance instructor, the assignee, he waived any right he had to enforce it against any subsequent assignments. Because this choice fails to account forthe fact that this prohibition was waived, it is incorrect. Choice (B) is incorrect. Covenants not to assign or sublease the premises have been held to run with the land. Because this choice states that such covenants do not, it is incorrect. Choice (D) is incorrect. While this is a true statement, this does not explain why the landlord is no longer in a position to contest the transfer to the yoga instructor. The landlord Loses because he waived the prohibition on assignments by accepting rent from the dance instructor, not because there is no Longer privity of estate between the two parties.
How well did you know this?
1
Not at all
2
3
4
5
Perfectly
35
Q
  1. A landlord, the owner of a two-story dwelling house, leased it completely furnished to a tenant for a 1 0-year period. Toward the end of the seventh year of the term, a violent storm blew off several of the roof shingles. The tenant noticed a leak through the roof where the shingles had blown off, but didn’t make any repairs. A month later, a severe rain storm occurred, and water leaked through the roof, causing damage to the valuable parquet floors in the two rooms below. Before the term of his lease ended, the tenant discovered that the landlord had not paid his taxes on the property, so he purchased the property through a sheriff’s sale. The tenant refuses to make any further rental payments on the property.If the landlord brings suit against the tenant for payment of rent in arrears, he would most likely(A) succeed, because the tenant is estopped to deny the landlord’s title.(B) succeed, because of his security interest in the property.(C) fail, because the purchase by the tenant vitiated any further contractual obligations.(D) fail, because the landlord was under a duty to keep the demised premises in reasonably good repair.
A
  1. (C) After purchasing the property at the sheriff’s sale, the tenant, as record title owner, would no longer remain obligated under the leasehold contract. As such, choice (C) is correct because the sheriff’s sale would vitiate the tenant’s obligation to make any further rental payments to the Landlord. Choice (A) is incorrect. The landLord cannot successfully argue that the tenant is estopped to deny the landlord’s title because the landlord no Longer has title. Furthermore, this choice is incorrect in stating that the landlord will prevail. Choice (B) is incorrect. The landlord will not be able to claim a security interest in the property because the landlord does not hold a lien on the property. Choice (D) is incorrect. This choice falsely suggests that the landlord’s failure to keep the premises in good repair was the reason the tenant no longer owes rent. This choice disregards the sheriff’s sale to the tenant, which is the best reason why the tenant is no longer obligated to pay rent.
How well did you know this?
1
Not at all
2
3
4
5
Perfectly
36
Q
  1. A landlord was the owner of a two-story dwelling house and leased it completely furnished to a tenant for a 10-year period. Toward the end of the seventh year of the term, a violent storm blew off several of the roof shingles. The tenant noticed a leak through the roof where the shingles had blown off, but didn’t make any repairs. A month later, a severe rain storm occurred and water leaked through the roof, causing damage to the valuable parquet floors in the two rooms below.In a subsequent proceeding by the landlord against the tenant to recover for the damage to the parquet floors resulting from the leak, liability would most likely be imposed upon(A) the landlord, because he was under an implied obligation to keep the premises in a habitable condition.(B) the landlord, because he was under an affirmative obligation to deliver the premises in a reasonable state of repair.(C) the landlord, because of the contractual obligation under the lease to make all necessary repairs during the term of the lease.(D) the tenant, because a tenant for years is obligated to make such ordinary repairs on the leased property.
A
  1. (D) The tenant is liable for the damage to the floors caused by his failure to replace the shingles because a tenant for years is legally bound to make such ordinary repairs on the leased property as will avoid serious injury to the property. Thus, the tenant’s failure to make the necessary repairs would make him liable for such permissive waste. Choice (A) is incorrect. A landlord impliedly covenants in the lease not to interfere with the quiet enjoyment of the premises. One way that that quiet enjoyment can be interfered with is if the landlord engages in behavior (or fails to engage in certain behavior) that amounts to a constructive eviction. However, the living arrangement must be so severe as to cause the premises to be uninhabitable such that the tenant must move out. Here, the roof leaked from a hole caused by several of the shingles falling away. This caused water damage to the parquet floors. However, nothing in the facts suggests that the damage was so severe as to cause heavy flooding or some other condition that would make the premises uninhabitable. So, although the landlord may have an obligation to keep the premises in a habitable condition, nothing about the damage here suggests that the Landlord is somehow failing to provide habitable premises. Choice (B) is incorrect. Even though the landlord was required to deliver the possession of the premises and was required to not interfere with the tenant’ quiet enjoyment, the tenant would be under a duty to make ordinary repairs. Choice (C) is incorrect. Nothing in the facts suggests that there was an express contractual obligation for the landlord to make all necessary repairs during the term of the lease. Absent those provisions, it is the tenant who has the duty not to commit permissive waste on the premises.
How well did you know this?
1
Not at all
2
3
4
5
Perfectly
37
Q
  1. A landlord was the owner of a two-story dwelling house and leased it completely furnished to a tenant for a 10-year period. Two years after the tenant entered into the leasehold agreement with the landlord, he subleases the house to his cousin. Thereafter, the cousin defaults on the rental payments.If the landlord brings an action to recover the past rent due from the cousin, the landlord will:(A) recover, because pnvity of estate exists between a landlord and sublessee.(B) recover, because there is privity of contract between a landlord and sublessee.(C) recover, because there is both privity of estate and privity of contract between a landlord and sublessee.(D) not recover, because privity of estate does not exist between the landlord and sublessee.
A
  1. (D) The landlord may not recover for rent due from sublessee because there is no privity of estate between landlord and sublessee. The landlord’s only cause of action remains against the tenant under their original leasehold agreement. Choice (A) is incorrect. This is an incorrect statement. There is no privity of estate between a landlord and a sublessee. Choice (B) is incorrect. This is an incorrect statement. There is no privity of contract between a landlord and a sublessee. The landlord is in privity of contract with the tenant. Choice (C) is incorrect. This is an incorrect statement. There is neither privity of estate nor privity of contract between a landlord and a sublessee.
How well did you know this?
1
Not at all
2
3
4
5
Perfectly
38
Q
  1. A corporation, under the authority of a statute of the state, sued to have condemned 1,000 acres of forested land owned by a timber company, which it planned to develop for use as a state recreational area and state game lands. After a hearing, the state court ordered possession of the land surrendered to the corporation, prior to determination of compensation, upon deposit in court of a sum deemed adequate to cover damages that might be awarded. The timber company immediately commenced an action to enjoin the court-ordered sale of its property.Which of the following would be the best ground for upholding the state court’s order?(A) The power of eminent domain may only be delegated directly to a private enterprise for a public related use or activity.(B) The power of eminent domain may only be delegated to a public authority through a legislative determination.(C) The injured party has not proved such irreparable injury to use as amounts to a “taking.”(D) The Fifth Amendment’s power of eminent domain, incorporated by the Fourteenth Amendment as applicable to the states, does not require that payment be made prior to condemnation of the property.
A
  1. (A) The power of eminent domain may be delegated directly or indirectly to a private person or enterprise subject to the requirements that the taking be (a) for a public use, and (b) just compensation be given. Choice (B) is incorrect. The court will not want to rely on this argument because it is not on point with the facts addressed in the fact pattern. One of the issues before the court would be whether or not a private enterprise can be delegated the power of eminent domain. Choice (B) addresses the issue of how power can be delegated to a public authority. However, the corporation is the party trying to condemn the forestland, not a public authority. Thus, the argument in (B) isa less persuasive reason for allowing the land to be surrendered to the corporation. Choice (C) is incorrect. It is a weak argument for the court to argue that the party was not irreparably injured enough and thus no taking occurred. One way that a taking ocurs is when the government confiscates or physically occupies property. When there is this kind of conduct, there is a taking and it is not necessary for the dispossessed ownerto show that she was irreparably injured. Because this is not part of the court’s inquiry in deciding whether or not the land should be surrendered to the corporation, it cannot be the basis or best grounds for upholding the order. Choice (D) is incorrect. As long as just compensation is given to the timber company, it will not matter whether that compensation was given prior to condemnation or afterwards. This will not be the best reason to uphold the court’s decision here.
How well did you know this?
1
Not at all
2
3
4
5
Perfectly
39
Q
  1. A state on the eastern seaboard has a long stretch of scenic, ecologically unique, and relatively unspoiled seacoast. Most of the coastline is privately owned by persons who have permanent or vacation residences on their coastal lands. A man purchased a two-acre tract of littoral (i.e., coastal) land in the state. When the man bought the property, there was a narrow concrete walkway (about 20 feet in length and 5 feet in width) adjoining the land. The walkway, which was designed to provide pedestrian access to the beach, was owned by the State Coastal Commission. When the man moved into his beach house, he noticed that the walkway was rarely used. As a result, the man decided to enclose the walkway and construct a paddle ball court over the entire area. He did so without notifying the State Coastal Commission. He has continued to use this area in excess of the period required for adverse possession.In an appropriate action brought by the man to establish title to the walkway area, which of the following must he establish if he is to prevail?(A) Real property interests can be abandoned by a governmental entity without an official vote.(B) Lack of use of the walkway by the state created an irrevocable license in him.(C) Real property interests can be lost by a state by adverse possession.(D) Due to a defect in the chain of title, the state does not own this piece of land.
A
  1. (D) Because title togovernment-owned property cannot be gained through adverse possession, the man’s best argument would be that someone other than the state actually owns the property. Choice (A) is incorrect. This is a weak choice because it only explains that the government has abandoned the property interest without giving a reason as to why or how that interest could have been abandoned. The man would not want to only argue that the property interest was abandoned; he would want to argue why he feels that interest was lost, and this choice does not supply the reasons. Choice (B) is incorrect. The man is trying to establish title to the walkway, i.e., ownership in the strip of land. This choice discusses licenses, which at most give the man the right to use the walkway, and this is not what the man is trying to establish. Choice (C) is incorrect. This statement goes against the gen eral rule that title to government-owned property cannot be gained through adverse possession, making it a very weak argument.
How well did you know this?
1
Not at all
2
3
4
5
Perfectly
40
Q
  1. A landlord, an owner in fee simple, leased a three- story house to a tenant for a term of 10 years. By the terms of the lease, the tenant expressly covenanted to pay a monthly rental of $300 and to pay the taxes on the premises during the term of the lease. The lease also stipulated that the tenant, as lessee, may not assign or sublease the said premises.The tenant and his family lived in the house for two years. Then the tenant, still owing the landlord three months back rent, assigned his leasehold interest in the property to a student, who was unaware of the prohibition against assignments. This written assignment expressly provided that the student would pay the landlord the monthly rental of $300, but was silent concerning the payment of taxes. The landlord never objected to this assignment. Six months after the assignment, the landlord conveyed his property interest in the premises to a new owner.The student failed to pay the taxes on the property. The new owner, after paying the taxes, brought suit against the student for the amount paid. Judgment should be for(A) the new owner.(B) the student, because he did not contract with the tenant to pay the taxes on the property.(C) the student, because the covenant to pay taxes will not “run with the land,” unless the intent is clearly expressed in the original lease.(D) the student, because the covenant to pay taxes is merely collateral and does not “run with the land.”
A
  1. (A) By virtue of the assignment, the assignee (i.e., the student) falls into privity of estate with the landlord, and he is liable on all covenants in the original lease that “run with the land.” Because the covenant to pay taxes (under the original leasehold agreement between landlord and tenant) “runs with the land,” the student, as assignee, would be held liable. Choice (B) is incorrect. Even though the student did not specifically contract with the tenant in the facts, the fact remains that the covenant did run with the land. Because the student is an assignee of the tenant’s property interest, the student is now in privity of estate with the new owner. Because of this, the student is bound by the covenant, even though there was no separate contract with the tenant to pay the taxes. Choice (C) is incorrect. Even though the Lease does not in express language say that the covenant will run with the land, the lease does say that the taxes will be paid during the term of lease. So, there is an intention that the covenant to pay rent would run with land in spite of a lack of express language. Because this answer choice falsely suggests that a Lack of express language leads to the student not being liable, it is incorrect. Choice (D) is incorrect. This type of covenant is not collateral and would run with the land. Because this answer choice states the opposite, it is incorrect.
How well did you know this?
1
Not at all
2
3
4
5
Perfectly
41
Q
  1. A homeowner owns a home in fee simple and leased it to his cousin for a term of 10 years. By the terms of the lease, the cousin expressly covenanted to pay a monthly rental of $300 and to pay the taxes on the premises during the term of the lease. The lease also stipulated that the cousin, as lessee, may not assign or sublease the said premises.The cousin and his family lived in the house for two years. Then the cousin, still owing the homeowner three months back rent, assigned his leasehold interest in the property to a friend, who was unaware of the prohibition against assignments. This written assignment expressly provided that the friend would pay the homeowner the monthly rental of $300, but was silent concerning the payment of taxes.After the cousin’s assignment to the friend, the homeowner sues the cousin to recover the rent in arrears. The homeowner will most likely(A) succeed, because the homeowner and the cousin stood in privity of contract.(B) succeed, even though the homeowner had notice of the assignment.(C) not succeed, because the cousin had assigned his interest in the premises before the homeowner brought suit.(D) not succeed, because he did not object to the assignment.
A
  1. (A) Under the terms of their original leasehold agreement, the cousin would remain Liable for the rent in arrears. The cousin’s assignment to the friend would not operate as an extinguishment of his duty to pay rent to the homeowner under the terms of their original agreement. An assignment does not release the tenant from his contractual obligation to the Landlord under the lease. To be sure, a lease is a contract as well as a conveyance. Choice B) is incorrect. This choice is erroneous because it falsely suggests that the homeowner’s notice (of the assignment) is somehow relevant to the cousin’s obligation to pay. The cousin’s obligation to pay is based upon the privity of contract that arose from the original lease agreement, and the homeowner’s notice (or lack thereoO will have no effect upon this original obligation. Choice (C) is incorrect. Even though the cousin had already assigned before the homeowner brought suit, the cousin is still liable for the rent under the original agreement based on privity of contract. The homeowner’s right to collect the rent is not extinguished because he brings suit after the assignment. Choice (D) is incorrect. Whether the homeowner objected to the assignment or not, the fact remains that the cousin is still liable for the rent. This obligation to pay the rent is not dependent upon the homeowner’s consent or objection to the assignment.
How well did you know this?
1
Not at all
2
3
4
5
Perfectly
42
Q
  1. A landlord owned a three-story home in fee simple and leased it to his friend for a term of 10 years. By the terms of the lease, the friend expressly covenanted to pay a monthly rental of $300 and to pay the taxes on the premises during the term of the lease. The lease also stipulated that the friend, as lessee, may not assign or sublease the said premises.The friend and his family lived in the house for two years. Then the friend, still owing the landlord three months back rent, assigned his leasehold interest in the property to a professor, who was unaware of the prohibition against assignments. This written assignment expressly provided that the professor would pay the landlord the monthly rental of $300, but was silent concerning the payment of taxes. The landlord never objected to this assignment. Six months after the assignment, the landlord conveyed his property interest in the premises to a new owner.After residing in the house for a year, the professor subleased the third floor to a student for a term of two years; the agreement provided that the professor would receive a monthly rental payment of $100.After the student had made regular rental payments to the professor for the first six months of the sublease, he defaulted. The following month, the professor sent the new owner $200 as payment for rent and notified him that he should collect the remaining $100 from the student, who refused to vacate the premises. In an action by the new owner against the student to recover the additional $100 for rent due, he will most probably(A) recover, because the landlord and sublessee are in privity of estate.(B) recover, even though he never objected to the sublease.(C) not recover, because the professor would only be held liable.(D) not recover, because the professor remains liable.
A
  1. (D) Because the sublessee (the student) is not in privity of estate with the landlord, the sublessee is not subject to the covenants or terms of the leasehold agreement. Therefore, the professor will remain liable for the entire $300 rental. Choice (A) is incorrect. The new owner will not recover because the landlord and sublessee are neither in privity of contract nor privity of estate. Choice (B) is incorrect. Whether the new owner objected to the sublease or not, the new owner still cannot recover from the student because there is neither privity of estate nor privity of contract here. Choice (C) is incorrect. The professor will be liable as he is in privity of estate with the landlord as an assignee. The friend will still remain in privity of contract with the landlord based upon the original lease agreement. Because this answer choice states that only the professor is liable, it is incorrect.
How well did you know this?
1
Not at all
2
3
4
5
Perfectly
43
Q
  1. A property owner who owner her property in fee simple leased a three-story house and the 21 -acre tract upon which it stood to a friend for a term of 10 years. By the terms of the lease, the friend expressly covenanted to pay a monthly rental of $300 and to pay the taxes on the premises during the term of the lease. The lease also stipulated that the friend, as lessee, may not assign or sublease the said premises.The friend and his family lived in the house for two years. Then the friend assigned his leasehold interest in the property to a cousin, who was unaware of the prohibition against assignments. This written assignment expressly provided that the cousin would pay the owner the monthly rental of $300, but was silent concerning the payment of taxes. The owner never objected to this assignment. Six months after the assignment, the owner conveyed her property interest in the premises to a developer.During the cousin’s tenancy, the State Highway Authority filed proceedings to condemn two acres of the tract for the purpose of highway construction. As a result, the cousin contacted the developer and informed him that he should be entitled to an apportioned reduction in the rental. After the developer refused to reduce the rent, the cousin brings an action to have his rent apportioned pro tanto.Judgment should be for(A) the developer, although the cousin would be entitled to terminate the lease.(B) the developer, because the cousin would be held to the original leasehold contract.(C) the cousin, because the value of his leasehold interest was reduced pro tanto.(D) the cousin, because eminent domain operates as an apportionment of rent.
A
  1. (B) The question of law presented here is, namely, when there is a partial taking of a leasehold estate by condemnation and the remaining portion is susceptible for occupation, shall the contractual monthly rental be abated pro tanto for the part taken during the remainder of the time of the lease? The majority rule is that rentals are not abated in this situation; rather, the tenant is obligated to continue the payment of the rentals provided in the lease contract and must look to an apportionment of the damages assessed against the condemning authority based on the reduced value of his lease. Choice (A) is incorrect. If only a portion of the Leased property is condemned, the tenant is not discharged from her obligation to pay rent. Because the facts state that only two acres of the 21-acre tract are being condemned, the cousin will not be allowed to terminate the lease. Choice (C) is incorrect. The majority rule is that rentals are not abated in this situation; rather, the tenant is obligated to continue the payment of the rentals provided in the lease contract and must look to an apportionment of the damages assessed against the condemning authority based on the reduced value of his lease. Choice (D) is incorrect. Eminent domain itself does not necessarily operate as an apportionment of rent, but instead allows the tenant to a share of the condemnation award. Where, as here, a portion of the land is condemned and the other portion is suitable for occupation, the tenant is obligated to continue the payment of the rent as provided in the lease and is entitled to a share of the condemnation award.
How well did you know this?
1
Not at all
2
3
4
5
Perfectly
44
Q
  1. A landowner owned a large building in the city. On January 15, the landowner leased the building to a tenant for a period of 20 years at a rental of $10,000 per month. The leasehold agreement between the landowner and tenant provided that the latter was not permitted “to assign this lease to anyone except a corporation with an ‘A’ credit rating from a well- known credit rating company.” On February 1 of the following year, the tenant leased the premises to a corporation that did not have the required credit rating. The tenant—corporation lease was for a period of five years, with a rental of $15,000 per month, payable by the corporation to the tenant. In addition, the corporation agreed to abide “by all of the terms and conditions of the lease between the landowner and tenant.”Which of the following accurately states the legal effect of the non-assignability clause contained in the landowner—tenant leasehold contract?(A) The non-assignability provision had no legal effect.(B) The non-assignability provision made the assignment from the tenant to the corporation ineffective.(C) The tenant-corporation lease did not effectuate a breach of the landowner—tenant contract.(D) Although the tenant-corporation lease constituted a breach of the landowner—tenant contract, the landowner would nevertheless be required to recognize the validity of the transfer (of the premises) to the corporation.
A
  1. (A) Choice (A) is the most accurate statement. The non-assignability provision had no legal effect. Because the tenant’s transfer (of the premises) to the corporation constituted a subLease, not an assignment, the said provision would have no legal effect. Students must know the distinction between an assignment and a sublease. An assignment is a transfer of a tenant’s entire leasehold estate to another; whereas a sublease is a transfer of anything less than a tenant’s entire leasehold estate. Choice (B) is incorrect. Because the tenant’s transfer (of the premises) to the corporation constituted a sublease, not an assignment, the said provision would have no legal effect. This answer choice is incorrect in referring to the transfer as an assignment when it was a sublease. Also, the transfer is, in fact, effective because the provision is inapplicable. Choice (C) is less correct because, although factually true, it does not answer the question asked. The question asks about whether the assignment clause has any effect on the validity of the transfer between the tenant and the corporation, not whether or not there was a breach of the original contract. Therefore, choice (C) is not the most accurate sttement and thus incorrect. Choice (D) is incorrect. This choice falsely states that the transfer was a breach of the original contract. Because the contract said nothing about subleases, there was never a breach of the original agreement.
How well did you know this?
1
Not at all
2
3
4
5
Perfectly
45
Q
  1. A landlord owned a large building in the city. On January iS, the landlord leased the building to atenant for a period of 20 years at a rental of $10,000 per month. The leasehold agreement between the landlord and tenant provided that the latter was not permitted “to assign this lease to anyone except a corporation with an ‘A’ credit rating from a well- known credit rating company.” On February 1 of the following year, the tenant leased the premises to a corporation that did not have the required credit rating. The tenant—corporation lease was for a period of five years, with a rental of $15,000 per month, payable by the corporation to the tenant. In addition, the corporation agreed to abide “by all of the terms and conditions of the lease between the landlord and the tenant.”One year later, the corporation leased the premises to a health club for the balance of the term of the corporation—tenant lease. The health club owner took possession of the said premises on February 1, the same day that the corporation vacated its occupancy. Pursuant to the corporation—health club leasehold agreement, the latter was obligated to pay a monthly rental of $17,500 directly to the corporation. The health club has a ‘B’ credit rating with the well-known credit rating company. For one year, the health club paid $17,500, each month directly to the corporation. During that same period, the corporation continued to pay $15,000 each month to the tenant, while the latter paid $10,000 (each month) to the landlord. The landlord knew about the leases to the corporation and the health club and protested promptly, but took no further action, apparently satisfied as long as he received his $10,000 per month from the tenant.Thereafter, the health club abandoned the premises and stopped paying rent to the corporation. After the health club discontinued paying rent, the corporation stopped paying rent to the tenant. When the tenant failed to receive his rent, he, too, stopped paying rent to the landlord.If the landlord brings suit to recover for past rent due, which of the following is most accurate?(A) The landlord may recover against the tenant for past rent due.(B) The landlord may recover against the corporation for past rent due.(C) The landlord may recover against the health club for past rent due.(D) The landlord may recover against the corporation and the health club, jointly and severally,for past rent due.
A
  1. (A) In the case of a subLease, privity of estate and privity of contract exist between the original landlord and tenant. Similarly, privity of estate and privity of contract exist between the tenant and subtenant. However, neither privity of contract nor privity of estate exists between the original landlord and subtenant. Therefore, the Landlord cannot sue the subtenant. As a consequence, the Landlord may not recover against subtenants (the corporation and the heaLth club owner); he may recover only against the tenant under the terms of their original leasehold contract. Choice (B) is incorrect. Neither privity of contract nor privity of estate exist between the original landlord and subtenant. Therefore, the landlord cannot sue the corporation subtenant. Choice (C) is incorrect. Neither privity of contract nor privity of estate exist between the original landlord and subtenant. Therefore, the landlord cannot sue the subtenant health club owner. Choice (D) is incorrect. Neither privity of contract nor privity of estate exists between the originaL landlord and subtenant. Therefore, the landlord cannot sue the subtenants jointly and severally.
How well did you know this?
1
Not at all
2
3
4
5
Perfectly
46
Q
  1. A property owner owned a large building in the city. The owner leased the building to a shopkeeper for a period of 20 years at a rental of $10,000 per month. The leasehold agreement between property owner and shopkeeper provided that the latter was not permitted “to assign this lease to anyone except a corporation with an ‘A’ credit rating from a well- known credit rating company.” One month later, the shopkeeper leased the premises to a corporation that did not have the required credit rating. The shopkeeper-corporation lease was for a period of five years, with a rental of $15,000 per month, payable by the corporation to the shopkeeper. In addition, the corporation agreed to abide “by all of the terms and conditions of the lease between the property owner and the shopkeeper.”One year later, the corporation leased the premises to a non-profit organization for the balance of the term of the corporation—shopkeeper lease. The non-profit organization took possession of the said premises the same day that the corporation vacated its occupancy. Pursuant to the corporation—nonprofit leasehold agreement, the latter was obligated to pay a monthly rental of $17,500 directly to the corporation. The non-profit organization has a ‘B’ credit rating with the well-known credit rating company.Which of the following is not correct regarding the legal relationship(s) of the various parties?(A) Privity of estate and privity of contract exist between the property owner and the shopkeeper.(B) Privily of estate and privity of contract exist between the shopkeeper and the corporation.(C) Privily of estate, but not privily of contract, exists between property owner and the corporation.(D) Neither privily of estate nor privily of contract exists between the property owner and the nonprofit organization.
A
  1. (C) This is a false statement and thus the correct choice. The statement is incorrect in suggesting that the property owner and the corporation are in privity of estate, which is false; the landlord and subtenant are never in privity of estate. Choice (A) is incorrect. This is a true statement because there is privity of estate and privity of contract between the landlord and the tenant. Likewise, privity of estate and privity of contract exist between the tenant and subtenant. However, neither privity of contract nor privity of estate exists between the landlord and the subtenant. Choice (B) is incorrect. This is a true statement because privity of estate and privity of contract exist between the tenant and the subtenant. Choice (D) is incorrect. This is a true statement because as between the property owner and the nonprofit organization, there is neither privity of estate nor privity of contract.
How well did you know this?
1
Not at all
2
3
4
5
Perfectly
47
Q
  1. A landlord was the owner of a vacant warehouse. On May 1, the landlord leased the warehouse to the tenant for a term of five years with a renewable option. The leasehold agreement provided in part:“The tenant hereby acknowledges that certain areas of the leased premises are in a state of disrepair and unsafe for the conduct of business. Nonetheless, the tenant agrees to assume full responsibility for the necessary repairs. Furthermore, the tenant agrees to indemnify the landlord for any loss resulting from the condition of the said premises.”Under the terms of the lease, the landlord delivered possession of the warehouse to the tenant on May 2. On that date, an employee of the tenant was working in the warehouse. He was moving some equipment into the warehouse when a section of the ceiling collapsed and fell on him. At the time the landlord and the tenant entered into their lease, only the landlord was aware that the ceiling was defective and needed to be repaired. Although the landlord could have remedied the defect before delivering possession to the tenant, the landlord didn’t perform any repair work on the ceiling. The employee initiated suit against the landlord to recover damages resulting from his injury. There are no applicable statutes.If the employee prevails it will be because a(A) landowner is strictly liable for injuries occurring on his property.(B) landowner’s duty of care to third parties cannot be shifted to a tenant by the tenns of a lease.(C) tenant cannot waive the implied warranty of commercial habitability.(D) covenant to indemnify by a tenant in favor of a land owner is against public policy.
A
  1. (B) At common law, a landlord generally does not impliedly warrant that the leased premises are suitable for any particular purpose, and he is not liable for a dangerous condition existing on the leased premises. Normally, the doctrine of caveat emptor prevails, but there are two notable exceptions. First, Smith and Boyer point out that “a landlord may be liable in tort to the tenant, his guests, licensees and invitees, if at the commencement of the lease there is a dangerous condition that the landlord knows or should know about and the discovery of which would not likely occur by the tenant exercising due care.” Property, pg. 138. Based on this exception, choice (B) is correct because the dangerous condition existed at the commencement of the leasehold period. (Note: The facts indicate that the employee was injured on the first day the tenant took possession of the warehouse.) The second exception is that a landlord in the lease of a completely furnished dwelling for a short period of time impliedly warrants the fitness of the premises and the furnishings. Choice (A) is incorrect. If the landowner is liable, it is based on her/his failure to warn of a dangerous condition that the landlord knew about or should have known about. The landowner’s liability arises from her/his negligence and any claim the employee brings would be based in negligence, not strict liability. Choice (C) is incorrect. This argument, though it sounds similar to choice (B), does not address the fact that the landlord did in fact owe a duty to third parties. This choice generally states that the tenant cannot waive the warranty, but makes no mention of any negligence by the landlord. Because choice (B) does at least mention that the landlord owed a duty of care to third parties, it states a better reason why the landlord is at fault. Choice (C) makes less mention of fault and is a weaker choice than choice (B). Choice (D) is incorrect. This is a weak argument because it gets away from the best theory of liability for the employee. The employee would sue the landlord for the landlord’s own negligence in breaching the duty of care. Though it is not clear that a court would necessarily hold that this type of indemnification provision is against public policy, it is not necessary to engage in this inquiry because this choice does little to explain that the landlord owed a duty to someone in the employee’s position. This choice does nothing to show that the landlord was at all negligent in failing to disclose the defects in the property. Because choice (B) at least mentions that the landlord owed a duty of care to third parties, that choice would be a better argument because it addresses the argument that the landlord was negligent.
How well did you know this?
1
Not at all
2
3
4
5
Perfectly
48
Q
  1. A famous environmentalist was the owner of a large tract of virgin forest. The environmentalist conveyed the tract “to the people of the state in fee simple; provided, however, that if any portion of said tract shall ever be used or developed for any commercial purpose or otherwise converted from its natural state (with exception for recreational, scientific, or educational purposes), then the grantor or his successors in interest may re-enter as of the grantor’s former estate.”The interest of the state in the tract would be best described as a (an)(A) fee simple determinable.(B) fee simple subject to condition subsequent.(C) easement appurtenant.(D) determinable fee subject to an executory interest.
A
  1. (B) The interest in the tract would be described as a fee simple subject to condition subsequent, which means fee simple subject to being terminated by exercise of the power of termination or right of re-entry for condition broken. The important characteristic that distinguishes this type of estate from a fee simple determinable is that the estate will continue in the grantee, or his successors, unless and until the power of termination is exercised. Although no particular words are essential to create an estate on condition subsequent, the use in the conveyance of the traditional words of condition, “upon condition that,”“provided that,” or “but if,” coupled with a provision for re-entry by the transferor or the occurrence of the stated event will normally be construed to manifest an intention to create an estate on condition subsequent. Choice (A) is incorrect. A fee simple determinable is a fee that automatically expires by force of the special limitation contained in the instrument creating the estate and the fee terminates when the stated contingency occurs. By contrast, the fee simple subject to condition subsequent continues despite the breach of the specified condition until it is divested or cut short by the exercise by the grantor of his power to terminate. Although no particular words are essential to create an estate on condition subsequent, the use in the conveyance of the traditional words of condition, “upon condition that,”“provided that,” and “but if,” coupled with a provision for re-entry bythe grantor will be construed to create an estate on condition subsequent. Choice (C) is incorrect. An easement is the right of one person to go onto the land in possession of another and make a limited use thereof. An easement is appurtenant when it is attached to a piece of land and benefits the owner of such land in his use and enjoyment thereof. Every easement appurtenant requires two pieces of land that are owned by two different persons. The property interest given to the state was a fee simple subject to condition subsequent, which is an actual ownership interest in the land, not merely the right to use the land. Thus, the state was not given an easement. Choice (D) is incorrect. A determinable fee subject to an executory interest is a fee simple estate, where ownership is to pass from the grantee to one other than the grantor upon the happening (or non-occurrence) of a specified event. Because the ownership interest is to pass to the grantor or his successors, this is not an executory interest and this answer choice is incorrect.
How well did you know this?
1
Not at all
2
3
4
5
Perfectly
49
Q
  1. A resident of a state owned a large tract of virgin forest. The resident conveyed this tract “to the people of the state in fee simple so long as said tract shall remain in its natural state (with exception for recreational, scientific, or educational purposes).”Under state law, conveyances of real property to “the people” of the state are deemed to vest title in the state regarded as a legal entity, and custody over such property resides in an appointed official known as the director of environmental resources. The director granted a logging company the privilege to cut timber on a remote portion of the tract, together with incidental easements of way.The right of the logging company to cut and sell the timber that was to be cleared from the tract would be an example of a (an)(A) license.(B) easement appurtenant.(C) easement in gross.(D) profit-a-prendre.
A
  1. (D) A profit-a-pendre is similar to an easement in that it is a non-possessory interest. The holder of the profit is entitled to enter upon the servient tenement and take the substance of the land (e.g., minerals, trees, oil, or game) subject to the priviLege. In this regard, a profit, like an easement, may be appurtenant or in gross. Choice (A) is incorrect. A license simply permits one person to come onto the land in the possession of another without being a trespasser. Whereas a profit is an interest in land, a License is not. This is not the best description of the logging company’s rights because the logging company was given more than mere permission to come onto the land. The logging company was aLLowed to remove timber from the property, so their interest is best described as a profit. Choice (B) is incorrect. An easement is the right of one person to go onto the land in possession of another and make a limited use thereof. An easement is appurtenant when it is attached to a piece of rand and benefits the owner of such land in his use and enjoyment thereof. Every easement appurtenant requires two pieces of land that are owned by two different persons. The logging company’s use of the tract does not benefit them as adjacent landowners; rather, their benefit comes from the timber they are allowed to remove. Therefore, it is incorrect to term their interest as an easement appurtenant. Choice (C) is incorrect. An easement is in gross when it is intended to benefit the holder of the easement personally, rather than in connection with any Land the holder owns. In other words, every easement in gross requires only one piece of land (i.e., the servient tenement) that is owned by a person other than the owner of the easement in gross. Because there is no dominant tenement and inasmuch as the logging company has the right to remove timber from the property, a profit is a more accurate description of what the logging company received.
How well did you know this?
1
Not at all
2
3
4
5
Perfectly
50
Q
  1. A resident of a state was the owner of a large tract of virgin forest. The resident conveyed the tract “to the people of the state in fee simple; provided, however, that if any portion of said tract shall ever be used or developed for any commercial purpose or otherwise converted from its natural state (with exception for recreational, scientific, or educational purposes), then the grantor or his successors in interest may reenter as of the grantor’s former estate.”Under state law, conveyances of real property to “the people” of the state are deemed to vest title in the state regarded as a legal entity, and custody over such property resides in an appointed official known as the director of environmental resources. The director granted a logging company the privilege to cut timber on a remote portion of the tract, together with incidental easements of way. The section that the logging company was to clear would be used for the development of a state recreational area.After the privilege was granted, the logging company proceeded to clear the timber, which he later sold for $10,000. When the resident learned that the logging company was cutting the timber and selling it, he notified the director to desist further exploitation of the land.In an ejectment action instituted by the resident against the state to recover the tract, what result would be expected?(A) The resident would succeed, because the logging company’s selling of the timber would entitle the resident to re-enter and terminate the grant to the state.(B) The resident would succeed, because the logging company’s selling of the timber would constitute an automatic termination of the state’s fee interest.(C) The resident would not succeed, because the director approved the clearing of the timber in order to develop the area for recreational purposes.(D) The resident would not succeed, because the reservation of the resident’s right to terminate would be violative of the rule against perpetuities.
A
  1. (C) The resident will not succeed because the director’s objective in the clearing of the timber was for the purpose of developing a recreational area. Although the logging company was not privileged (under the terms of the resident’s deed over to the state) to use the tract for his own commercial advantage, the court will look to the ultimate purpose for which the land was to be used (i.e., recreational park). Thus, the court will conclude that the resident’s original intention and expectation in conveying the tract to the state was not materially or adversely affected. Choice (A) is incorrect. The resident will not succeed because the director’s objective in the clearing of the timber was for the purpose of developing a recreational area. Although the logging company was not privileged (under the terms of the resident’s deed over to the state) to use the tract for his own commercial advantage, the court will look to the ultimate purpose for which the land was to be used (i.e., recreational park). Thus, the court will conclude that the resident’s original intention and expectation in conveying the tract to the state was not materially or adversely affected. For this reason, the resident will not be able to re-enter and terminate the grant to the state. Choice (B) is incorrect. Assuming that the state did violate the condition, that behaviorwould not automaticaLlyterminate the state’s fee interest because the resident granted the state a fee simple subject to condition subsequent. The important characteristic that distinguishes this type of estate from a fee simple determinable is that the estate will continue in the grantee, or his successors, unless and untiL the power of termination is exercised. Because this choice falsely suggests that the estate would automatically be terminated without the resident having to exercise any rights, it is incorrect. This choice is also incorrect because the resident would not prevail because the director’s objective in the clearing of the timber was for the purpose of developing a recreational area. Choice (D) is incorrect. The resident’s right of re-entry is not the type of future interest that requires an application of the Rule against Perpetuities. The Rule against Perpetuities applies to the following future interests: vested remainder subject to open, contingent remainder, and executory interest.
How well did you know this?
1
Not at all
2
3
4
5
Perfectly
51
Q
  1. A farmer was the fee-simple owner of a tract of land inthe country. The farmer moved to the city in 1986. Aman took possession of the land in 1987 as an adversepossessor and remained on the land from 1987 to 1997;he then left the property to care for his sick mother forone year, returning the following year, 1999.Assume that the statutory period for adverse possession in this state is 20 years.In 2010, the farmer wanted to move back to the tract because he desired to return to the country. Finding the man in possession, the farmer initiated a suit in ejectment against the man. The most probable judicial determination would be(A) the farmer would lose, because the man’s possession was open, hostile, and continuous.(B) the farmer would lose, because the man had fulfilled the required statutory period to become an adverse possessor.(C) the farmer would win, because the man’s possession was not continuous.(D) the farmer would win, because he is the titleholder of record.
A
  1. (C) The farmer would win because the man’s possession was not continuous. For one to hold property adversely, his possession must be (1) actual and exclusive; (2) open and notorious; (3) continuous and peaceable (meaning without any interruption); and (4) hostile and adverse. Because the man’s possession was interrupted, he didn’t fulfill the statutory requirement of continuous possession for 20 years. Because the man never fulfilled an unbroken 20-year period, the man cannot claim ownership through adverse possession. Choice (A) is incorrect. The man’s possession was not continuous for the 20-year statutory period because he left for one year to care for his sick mother. When the man returned in 1999, that reset the statutory period and the man needed to fulfill a new 20-year period. Choice (B) is incorrect. The man did not fulfill the required statutory period to become an adverse possessor because he left for one year to care for his sick mother. When the man returned in 1999, that reset the statutory period and the man needed to fulfihla new 20-year period. Choice (D) is incorrect. The farmer does not win simply because he is the title holder of record because if the man had been in continuous possession, then the man would be allowed to claim the tract. Rather, the better reason why the farmer wins is because the man cannot fulfill all of the requirements of adverse possession.
How well did you know this?
1
Not at all
2
3
4
5
Perfectly
52
Q
  1. A woman was the owner of a farm in fee simple. Thewoman left the farm in 1986 to move to the city. Aman took possession of the land in 1987 as an adversepossessor and remained on the land from 1987 to 1997;he then left the property to care for his sick mother forone year, returning the following year, 1999.The man’s nephew agreed to live on the farm and take care of it during his uncle’s absence. When the man returned from his mother’s home in 1999, he continued in possession until 2010 when the womanbrought an ejectment action.Assume that the statutory period for adverse possession in this state is 20 years. The court would most likely hold that(A) the woman was the rightful owner and thus entitled to possess the farm.(B) the woman was entitled to possession, because the man’s intermittent possession was insufficient to make him an adverse possessor.(C) the man was entitled to possession, because his possession was open, notorious, continuous, and hostile as required for the statutory period.(D) the man was entitled to possession, because of the privity between the man and his nephew, which aided him in fulfilling the required statutory period.
A
  1. (D) Choice (D) is correct because the period of adverse possession of one possessor can be tacked on to the period of adverse possession of another possessor when there is privity (e.g., blood or contract) between the two. Choice (A) is incorrect. Because the nephew’s time spent can be tacked on to both segments of the man’s occupation, the man will be able to show that his possession was continuous forthe requisite 20 years. Thus, the woman wiLl not be declared the rightful owner. Choice (B) is incorrect. Although the man’s possession was intermittent and broken, the man will be allowed to tack his two segments of possession onto the nephew’s possession and create a continuous period. Choice (C) is incorrect. The man is, in fact, entitled to possession, but only because he was allowed to tack his time onto the nephew’s stay. Because this choice fails to take this factor into account, it is not as complete as choice (D), which does mention that it is the man’s tacking on that is the reason he will prevail.
How well did you know this?
1
Not at all
2
3
4
5
Perfectly
53
Q
  1. A rancher was the owner of a parcel of land in fee simple. The rancher left the parcel in 1977 to move to the city. A man took possession of the land in 1978 as an adverse possessor and remained on the land from 1978 to 1988; he then left the property to care for his sick mother for one year, returning the following year, 1990. Meanwhile, during the man’s possession, the rancher devised the parcel to his son in 1994. The man remained on the property until2010.Assume that the statutory period for adverse possession in this state is 20 years.Upon the rancher’s death in 2010, his son, as his sole heir, brings an ejectment suit to recover possession of the property from the man. In his action, the son will most probably(A) lose, because the man has acquired title by adverse possession.(B) win, because the statutory period for adverse possession does not run against a remainder- man until his interest becomes possessory.(C) win, because the man left the property to take care of his mother for one year.(D) win, because his title was perfected at the time of conveyance.
A
  1. (A) Since the man was in continuous possession of the parcel from 1990 to 2010, he would acquire title by adverse possession. As a result, choice (A) is correct. Choice (B) is incorrect because the facts do not disclose that the son was a remainderman or had a remainder interest in the parcel. The rancher simply devised the property to his son in 1994. As a consequence, the son did not have a remainder interest but a present possessory interest as of 1994. Choice (C) is incorrect. Certainly, the man left the property to take care of his mother and this created a gap in the continuous nature of the man’s possession. But when the man returned in 1990, he fulfilled a full statutory period of 20 years because the son is bringing suit in 2010. Choice (D) is incorrect. The son’s title was not perfected in 1994 simply because it was conveyed to him. In fact, there was no attempt to perfect title until the son sued for ejectment and tried to quiet title. That attempt to perfect title did not occur until 2010 and by that point, the man had fuLfilled the statutory period for adverse possession.
How well did you know this?
1
Not at all
2
3
4
5
Perfectly
54
Q
  1. A homeowner died in 1985. His will devised his estate in a southern state to his uncle and his best friend “to share and share alike as tenants in common.”At the time of the homeowner’s death, the uncle lived in a different part of the southern state (in which the estate was located), while the best friend resided in a northern state. After the homeowner’s funeral, the uncle returned to his own residence, but the best friend decided to occupy the estate. He put his name on the mailbox and has paid the taxes and maintenance expenses. To earn extra money, the best friend rented a small house on the property to a teacher and received a monthly rental payment from her. The best friend also grew fruits on the property and sold them at a stand on Fridays. The uncle has been generally aware of this, but because he cared little about the estate, the uncle has never pressed the best friend about the property. Since 1985 the uncle has not paid any rent or other compensation to the best friend, nor has the best friend requested such payment.In January 2010, a series of disputes arose between the uncle and the best friend for the first time concerning their respective rights to the estate. The state in which the property is located recognizes the usual common law types of cotenancies and follows majority rules on rents and profits. There is no applicable legislation on the subject.The uncle brings an appropriate action for a portion of the proceeds that the best friend received from his fruit stand and a portion of the rent that the teacher paid. If the best friend contests the apportionment of the monies he received, judgment should be for whom?(A) As a cotenant in possession, the best friend retains the profits from his crops and the rents paid by the teacher.(B) As a cotenant in possession, the best friend retains the profits from his crops, and the uncle is entitled to a share of the rent paid by the teacher.(C) The uncle is entitled to a share of the rent that the teacher paid and the profits from the best friend’s crops.(D) The uncle is entitled to no share of any of the monies raised because the uncle’s lack of contact with the best friend will be deemed a waiver.
A
  1. (B) In a majority of states, a tenant in possession has the right to retain profits gained by his use of the property. However, a co-tenant out of possession does have a right to share in rents and profits received from third parties. The best friend will be allowed to keep all of the money he earned from his fruit stand and need not share those profits gained by his own use of the property. However, the uncle wiLl be entitled to a share of the rents paid by the teacher. Choice (A) is incorrect. Even though the best friend was the co-tenant in possession, he is not entitled to keep all of the rent paid by the teacher. The best friend must share a portion of the rent with the uncle. Choice (C) is incorrect. In a majority of states, a tenant in possession has the right to retain profits gained by his use of the property. However, a co-tenant out of possession does have a right to share in rents and profits received from third parties. The best friend will be allowed to keep all of the money he earned from his fruit stand and need not share those profits gained by his own use of the property. However, the uncle will be entitled to a share of the rents paid by the teacher. Choice (D) is incorrect. Even though the best friend was the co-tenant in possession, he is not entitled to keep all of the rent paid by the teacher. The best friend must share a portion of the rent with the uncle.
How well did you know this?
1
Not at all
2
3
4
5
Perfectly
55
Q
  1. A man died in 1985. His will devised his estate in a Midwestern state to his son and daughter “to share and share alike as tenants in common.”At the time of their father’s death, the daughter lived in a different part of the Midwestern state (in which the estate was located) while the son resided in a state in the Southwest. After the funeral, the daughter returned to her own residence, but the son decided to occupy the estate. He put his name on the mailbox and has paid the taxes and maintenance expenses. The daughter has been generally aware of this, but because she cared little about the estate, the daughter has never pressed her brother about the property. Since 1985, the daughter has not paid any rent or other compensation to her brother, nor has he requested such payment.In January 2010, a series of disputes arose between the son and daughter for the first time concerning their respective rights to the estate. The state in which the property is located recognizes the usual common law types of cotenancies and there is no applicable legislation on the subject. Now, the son claims the entire fee simple absolute title to the estate and brings an action against his sister to quiet title in himself.If the Midwestern states in which the property is located has an ordinary 20-year adverse possession statute, judgment should be for whom?(A) The daughter, because one cotenant cannot acquire title by adverse possession against another.(B) The daughter, because there is no evidence that her brother has performed sufficient acts to constitute an ouster.(C) The son, because the acts of the parties indicate the sister’s intention to renounce her ownership rights to the estate.(D) The son, because during the past 25 years, the son has exercised the type of occupancy ordinarily considered sufficient to satisfy adverse possession requirements.
A
  1. (B) Under the unity of possession, each co-tenant is entitled to possess and enjoy the whole of the property subject to the equal right of his (or her) co-tenant. If one tenant wrongfully excludes another co-tenant from possession of the whole or any part of the whole of the property, such conduct amounts to an ouster. Here, choice (B) is correct because there is no evidence that the son was wrongfully excluding his sister from the estate. Choice (A) is incorrect. (A) is wrong because Burby points out that “possession may become adverse if the co-tenants are excluded from possession (i.e., ouster) or if the co-tenants are specifically notified of the adverse claim or can be charged with notice because of the acts of the possessor.” Real Property, pg. 232. Because one co-tenant can acquire title by adverse possession against another if there are acts sufficient to constitute ouster, this choice is incorrect. Choice (C) is incorrect. Ouster occurs when one co-tenant manages to wrongfully exclude his co-tenants from possession of the property. If there were a proper ouster, it is the actions of the son (as the co-tenant in possession) that would be relevant. The daughter’s actions and behavior alone cannot constitute an ouster because an ouster will be determined by whether or not the son acted to wrongfully exclude his sister. Choice (D) is incorrect. Even if this were true and the son behaved in such a way as to satisfy the requirements of adverse possession, there must still be acts sufficient to constitute an ouster of the daughter before the son can adversely possess the entire estate. This answer choice fails to take into account the need for ouster, thus making this choice incomplete.
How well did you know this?
1
Not at all
2
3
4
5
Perfectly
56
Q
  1. A rancher is currently the owner of record of a parcel of land designated as “A.” It is the higher of two neighboring properties on a mountain. A farmer is the owner of parcel “B,” which is situated lower than “A” on the mountainside.In 2000, the rancher, who owned both parcels of land, constructed a private drainage system. This system consisted of an underground pipeline that extended across both parcels of land.Sewage from parcel “A” drained through the system to a municipal sewer, which was located at the bottom of the mountain.In 2005, the rancher sold parcel “B” to the farmer. The deed to the farmer did not contain any mention of the rancher’s drainage system. Located on each of the parcels are large ranch-style homes occupied by the respective owners.Lately, the sewage drain has begun to emit an unpleasant odor across parcel “B.” As a result, the farmer sued the rancher in trespass praying for damages caused by the drainage system.Judgment should be for(A) the rancher, because the deed to the farmer did not contain any covenants restricting the rancher’s use of the land.(B) the rancher, because the farmer’s proper cause of action would be for nuisance, rather than trespass.(C) the farmer, because the drainage system was the proximate cause of the plaintiff’s damages as a matter of law.(D) the farmer, because the rancher did not have the right to improve his own land in a manner that caused harm to another.
A
  1. (B) The farmer’s proper cause of action should be based on nuisance, rather than trespass. The basic distinction that is now recognized is that trespass is an invasion of the plaintiff’s interest in the exclusive possession of his land, whereas nuisance is an interference with his use and enjoyment of it. In all likelihood, the rancher has an implied easement for his sewer pipe from parcel “A” across parcel “B.” It is generally agreed that in cases involving an implied reservation (or quasi-easement) in favor of the grantor, there must be reasonable necessity forthe existence of the implied easement. Choice (A) is incorrect. Just because the deed contained no covenants restricting the rancher’s use, that doesn’t allow the rancher to use the easement in such a way as to interfere with the use and enjoyment of the farmer’s property. The holder of an easement may make reasonable use of the servient tenement so long as her conduct does not unreasonably interfere with the use of the servient tenement owner. Because the type of harm that the farmer is suffering is more appropriately redressed under a nuisance action, Choice (B) is better. Because choice (A) does not address the fact that the farmer is bringing an improper cause of action, it is not as complete as choice (B). Choice (C) is incorrect. The farmer’s proper cause of action would be a suit in nuisance (due to the odors), not negligence. For this reason, proximate cause will not need to be part of the analysis. Choice (D) is incorrect. The farmer’s proper cause of action would be a suit in nuisance. Because this answer choice names the farmer as the prevailing party in a trespass suit against the rancher, it is an incorrect choice.
How well did you know this?
1
Not at all
2
3
4
5
Perfectly
57
Q
  1. A man is the owner of record of a parcel of land designated as “A.” It is the highest of three neighboring properties on a mountain. The man’s friend is the owner of parcel “B,” which is situated lower than “A” on the mountainside. The man’s cousin owns parcel “C,” which lies below parcels “A” and “B” on the mountain slope.In 2000, the man who originally owned all three parcels of land constructed a private drainage system. This system consisted of an underground pipeline that extended across all three parcels of land.Sewage from parcel “A” drained through the system to a municipal sewer, which was located at the bottom of the mountain.In 2005, the man sold parcel “B” to his friend. The following year, the cousin purchased parcel “C” from the man. The deeds to the friend and the cousin did not contain any mention of the man’s drainage system. Located on each of the parcels are large ranch-style homes occupied by the respective owners.From the standpoint of the friend and the cousin, thedrainage system would most likely be defined as a(an)(A) easement appurtenant.(B) easement by implication.(C) prescriptive easement.(D) express easement.
A
  1. (B) An easement may arise by implication if the existence of the easement is strictly (or reasonably) necessary forthe beneficial use of the land. The creation of such an easement is based upon the presumed intent of the grantor and grantee. If the claim is made that the easement arose out of a prior conveyance, there must be proof of the fact that at one time both the dominant and servient estates were under one ownership. Technically, choice (A) is correct, but choice (B) is preferred because the present example is an illustration of an implied easement appurtenant. So choice (A) is incorrect. The easement is appurtenant in that it is attached to a piece of land and benefits the owner of such land in his use and enjoyment thereof. However, this choice still does not explain how the easement was created. Because choice (B) better explains how the easement was created, it is a preferred choice. Choice (C) is incorrect. With respect to (C), an easement (or profit) may also be acquired by prescription in a manner similar to that by which ownership of apossessory estate may be acquired by adverse possession. However, because there are facts that support an easement created by implication (common ownership of both estates and reasonable necessity for creation of the easement), it is a stronger argument for the man to make than to argue that this is an easement created by prescription. Choice (D) is incorrect. This is not an express easement because the facts state that neither the friend’s nor the cousin’s deed made any reference to the man’s pre-existing drainage system.
How well did you know this?
1
Not at all
2
3
4
5
Perfectly
58
Q
  1. A woman is the owner of record of a parcel of land designated as “A.” It is the highest of three neighboring properties on a mountain. The woman’s sister owns parcel “C,” which lies the lowest on the mountain. Parcel “B” lies between Parcels “A” andIn 2000, the woman who originally owned all three parcels of land constructed a private drainage system. This system consisted of an underground pipeline that extended across all three parcels of land.Sewage from parcel “A” drained through the system to a municipal sewer, which was located at the bottom of the mountain.In 2005, the sister purchased parcel “C” from the woman. The deed to the sister did not contain any mention of the woman’s drainage system. Located on each of the parcels are large ranch-style homes occupied by the respective owners. A portion of the sister’s unimproved land began to subside over the drainage system. This was because of an especially pliable soil condition of which the woman was unaware prior to construction of the drainage system.In an action brought by the sister against the woman to recover property damage for the subsidence of her land, judgment should be for whom?(A) The sister, because she is entitled to lateral support for her land in its natural condition.(B) The sister, because she is entitled to subjacent support for her land in its natural condition.(C) The sister, because she is entitled to support for her land in its improved condition.(D) The woman, unless she sister proves that the woman was negligent in constructing the drainage system.
A
  1. (B) The right to subjacent support means support from underneath the surface of the land as distinguished from the sides. Unlike with lateral support, a landowner has an absolute right to subjacent support for land in its natural state and for any structures on the land. Choice (A) is incorrect. The sister would not be bringing an action for loss of lateral support in this instance. Because the sister suffered property damage for the subsidence of her land, she will be suing for a loss of subjacent support. Choice (C) is incorrect. The sister only suffered damage to an unimproved portion of her property when the subsidence began. Because there are no facts to suggest that any structure of the ranch-style home was damaged, this is not the best choice. Choice CD) is incorrect. The sister does not need to prove negligence on the part of the woman in order to recover for the property damage due to a loss of subjacent support.
How well did you know this?
1
Not at all
2
3
4
5
Perfectly
59
Q
  1. A developer built a mini-mall in the city. In order to finance the construction of the mall, the developer obtained a $500,000 commercial loan from a bank. The loan was secured by a mortgage that contained an after-acquired property clause that provided:“(The developer) hereby covenants that the terms of this mortgage shall apply to all after-acquired property which said mortgagor shall hereafter acquire; andMortgagee-bank’s security interest in such after- acquired property shall take priority over all other parties claiming an interest in such property.”The bank duly recorded this mortgage on August 15, 2007. On September 6, 2008, the developer purchased a four-story office building in a nearby city. The building was in need of much renovation. To finance the improvements, the developer borrowed $250,000 from his friend. As consideration for the loan, the developer executed a promissory note for $250,000, payable to the friend in 10 years and secured by a mortgage on the office building. This mortgage was dated September 9, 2008 and recorded the following day.During the early part of 2009, the developer experienced serious financial setbacks and failed to make the monthly mortgage payments to the bank and the friend. After the developer went into default on both loans, the bank instituted a foreclosure action on both the mini-mall and the office building. The friend intervened, claiming that his security interest in the office building took priority over the after-acquired rights claimed by the bank.This jurisdiction maintains a grantor—grantee index and has the following recording statute in effect:“An unrecorded mortgage, deed, or other instrument is invalid as against a subsequent mortgagee or bona fide purchaser for value without notice.”In determining which security interest in the office building should be given priority, the court should hold in favor of(A) the bank, because its mortgage was recorded prior in time.(B) the bank, because the after-acquired property clause in its mortgage is enforceable and takes priority over the friend’s subsequent security interest.(C) the friend, because the bank’s prior recorded mortgage is outside the friend’s chain of title.(D) the friend, because an after-acquired property clause is void against public policy and cannot encumber real estate subsequently acquired by the mortgagor.
A
  1. (B) According to Osborne, mortgages on after-acquired property are valid as between the parties, “but to be effectual against third persons, it must point out the subject matter so that such persons by inquiry notice may be able to identify the property intended to be covered.” In this regard, Osborne further notes that “a clause covering all after-acquired property is good” and is sufficient to adequately identify the property between the mortgagor-mortgagee and to put third parties on notice. Mortgages, p. 69. In this question, the facts state that the developer’s mortgage with the bank covered “all after-acquired property.” Therefore, the description is sufficiently definite to cover the office building subsequently acquired by the developer. Choice (A) is incorrect. Although this is true that the bank recorded its mortgage prior in time, this choice does not explain why the bank has an interest in property that was mortgaged to another party. Choice (B) gives a more specific reason why the bank prevails because (B) explains that it is comprehensive nature of the after-acqufred property clause that gives the bank a security interest in this second property. By recording this interest prior in time, this gave constructive notice to the friend that the bank would have an interest in all property acquired later in time by the developer. Because this choice only states a fact without explaining why that is legally significant, it is incomplete. Choice (C) is incorrect. The friend could look in the Grantor Index under the developer’s name to see what the developer did as a grantor. The friend would find that the developer had given a mortgage to the bank that was recorded in August 2007. In that mortgage was the after-acquired property clause. The mortgage would not be outside the friend’s chain of title. Choice (D) is incorrect. In this question, the facts state that the developer’s mortgage with the bank covered “all after-acquired property.” Therefore, the description is sufficiently definite to cover the office building subsequently acquired by the developer. Because of this, the clause is valid and will not be declared void as against public policy.
How well did you know this?
1
Not at all
2
3
4
5
Perfectly
60
Q
  1. A senator, had been a state senator for three years. During that period of time, it had been his regular practice to stay at a hotel in the state’s capital while the legislature was in session (namely from September through March). As a general rule, the senator stayed at the hotel each and every night during that seven-month period. When the senator checked into the hotel on September 1, 2007 for the 2007—2008 term, the hotel manager quoted the senator his usual senatorial discount rate of $50 per night. Although nothing was said regarding payment terms, the senator customarily paid the bill in weekly installments.During the months of September and October, he paid the hotel at the end of each week $350 plus tax and other accumulated expenses. On the last day of October, the senator made his usual weekly payment covering the rental period from October 25 to October 31. Then, on the morning of November 4 (before checkout time for that day), the senator decided to suddenly move out of the hotel and register at a new motel located two blocks away. He notified the manager at the hotel of his intent to check out and tendered a check for $150 plus tax and other incidental charges incurred from November 1 through November 3. When the manager noticed that the senator was paying only for three days, she remarked, “You still owe us $200 plus tax for the rest of the week and $350 plus taxes for an additional week because you didn’t give us a week’s notice that you planned to vacate.” The senator refused to pay the additional charges.If the hotel brings suit against the senator to recover for the unpaid balance, how many additional days lodging will he be required to pay for?(A) None.(B) Four.(C) Seven.(D) Eleven.
A
  1. (A) Every leasehold includes the following elements: (1) an estate in the tenant; (2) a reversion in the landlord; (3) exclusive possession and control of the land in the tenant; and (4) generally, a contract between the parties. The first three elements constitute privity of estate, and the fourth constitutes privity of contract. Whereas in a lease exclusive possession must reside in the tenant, in a license the possession remains in the licensor and the licensee has a mere privilege of being on the land without being treated as a trespasser. Smith and Boyer, Real Property, p. 48. Ordinarily, a lodging contract does not create a landlord—tenant relationship but only that of licensor and licensee, whereby the occupant contracts for the use of a room and facilities without a possessory interest in the land. Exclusive possession and control remains in the landowner. The senator, as a hotel guest, does not have exclusive possession and control of his hotel room. He is a licensee, rather than a tenant. Furthermore, the facts do not strongly support a periodic tenancy since his oral agreement with the hotel was not referred to as a “lease,” nor was the weekly payment referred to as “rent.” Therefore, no seven-day prior notice will be required. In the absence of any leasehold, the senator will owe no additional balance beyond the day he checks out. Choice (A) is correct. Choice (B) is incorrect. This is the balance of the week that the senator failed to complete. He was not required to lease in whole weeks since the facts do not strongly support a periodic tenancy because his oral agreement with the hotel was not referred to as a “lease,” nor was the weekly payment referred to as “rent.” Because no periodic tenancy had been created, there was no reason why the senator needed to pay for an entire week’s worth of days. He could leave the hotel room in the middle of a given week and only pay for those days that he had occupied the room. Choice (C) is incorrect. If a periodic tenancy had been created, then the senator would likely have to pay for eleven days—the four remaining on this week plus the next seven days because he would have to give one full period’s notice before moving out. Seven days would not make sense in this situation because the senator would not be required to pay only one more week from the date in question; he would have to pay for eleven days. Choice (D) is incorrect. The facts do not strongly support a periodic tenancy because his oral agreement with the hotel was not referred to as a “lease,” nor was the weekly payment referred to as “rent.” Because no periodic tenancy had been created, there was no reason why the senator needed to pay for an entire week’s worth of days. He could leave the hotel room in the middle of a given week and only pay for those days that he had occupied the room. Because of this, the senator was not required to give one full period’s notice before moving out and not required to pay for that extra time that he would be staying in the room.
How well did you know this?
1
Not at all
2
3
4
5
Perfectly
61
Q
  1. A woman is the fee simple owner of a 1,000-acre tract of land. Half of the tract is swampland totally unfit for cultivation, though it is heavily covered with valuable hardwood timber. The other half is also covered with valuable timber, but it would be land fit for the raising of crops if it were cleared of the timber. The latter section of land is more valuable for cultivation than it is for the growing of timber and has clearings where someone could build a home and live.The woman conveyed the tract to her brother for life. At the time of the conveyance, the swampland had never been used for the production of timber. The brother took possession and cleared 40 acres of the timber on the section that was suitable for cultivation. In addition, the brother cut 60 acres of timber in the swampland, thus becoming the first person to exploit this area. The brother then proceeded to clear some timber on the tract and used it in repairing fences, buildings, equipment, and the like on the property.In an action by the woman to permanently enjoin the brother from cutting any more timber on the swampland section of the tract and to account for profits received in the sale of the timber, the woman will most likely(A) succeed, because a life tenant must account for permissive waste to the reversioner or remainderman.(B) succeed, because a life tenant may not exploit natural resources where no such prior use had been made.(C) not succeed, because a life tenant is not liable for permissive waste.(D) not succeed, because a life tenant has a right to make reasonable use of the land.
A
  1. (B) The swampLand is not usable for the growing of crops, nor can it be made to be. It seems to be capable of growing trees only. This swampland, as far as the facts disclose, has never been used (and at the time of the taking effect of the brother’s life estate) was not being used for the production of lumber. The brother is the first to cut timber on the property. It is well settled that a tenant for life (or for years) has a right to continue the operation of old mines on the leased premises for the reason that such is the use to which the land has been put or is being put at the time of the beginning of the tenant’s estate. The lease implies that such use may be continued. On the other hand, such tenants have no right to open new mines and appropriate the minerals. Likewise, the timber should not be available for consumption and destruction by a tenant of a limited estate as against the reversioner or remainderman who owns the inheritance. Therefore, the brother should be permanently enjoined from cutting any more trees on the swampland, and should account for the proceeds from the sale of the timber that was cut on the swampland. Choice (A) is incorrect. Permissive waste consists of injury to the premises or land caused by the tenant’s failure to act when it is her/his duty to act (e.g., there is a hole in the roof of a leased dwelling house, which the tenant fails to repair, thus causing rain to leak and damaging the hardwood floors of the house). This is not the type of waste that the brother engaged in because he did not allow the premises to fall into a state of disrepair due to his neglect. If anything, he did some affirmative act to the land. Choice (C) is incorrect. A court can grant an injunction against the life tenant to enjoin any further waste on the property. Moreover, the court can also order an accounting for waste already committed by the life tenant. Because this answer choice falsely suggests that a Life tenant is not liabLe for permissive waste, it is incorrect. Choice (D) is incorrect. Certainly, a life tenant has the right to make reasonable use of the Land. What the life tenant does not have the right to do is to use (or neglect) the land in such a way as to impair the interests of the holder of any reversion or remainder interest in the land.
How well did you know this?
1
Not at all
2
3
4
5
Perfectly
62
Q
  1. A grantor is the fee simple owner of a 1,000-acre tract of land. Half of the tract is swampland totally unfit for cultivation, though it is heavily covered with valuable hardwood timber. The other half is also covered with valuable timber, but it is land that would be fit for the raising of crops if it were cleared of the timber. The latter section of land is more valuable for cultivation than it is for the growing of timber.The grantor conveyed his tract to his brother for life. At the time of the conveyance, the swampland had never been used for the production of timber. The brother took possession and cleared 40 acres of the timber on the section that was suitable for cultivation. In addition, the brother cut 60 acres of timber in the swampland, thus becoming the first person to exploit this area.The brother’s cutting of the timber on the section of tract suitable for cultivation would be an example of(A) permissive waste.(B) ameliorative waste.(C) equitable waste.(D) unreasonable exploitation.
A
  1. (B) Ameliorative waste is a change in the physical characteristics of the occupied premises by an unauthorized act of the tenant that increases the value of the property.The law takes account of this by awarding to the tenant the timber that is removed for such purpose. Thus, the tenant for life (or for years) has a right to use land as good husbandry dictates. As such, this usually incLudes the right to transform timberland into arable land for the plow, provided of course the soil can be made suitable for such purpose. Choice (A) is incorrect. Permissive waste consists of injury to the premises or land caused by the tenant’s failure to act when it is her/his duty to act (e.g., there is a hole in the roof of a leased dwelling house, which the tenant falls to repair, thus causing rain to Leak and damaging the hardwood floors of the house). This is not the type of waste that the brother engaged in because the brother did not allow the premises to fall into a state of disrepair due to his neglect. If anything, he did an affirmative act to the land. Choice (C) is incorrect. Equitable waste consists of injury to the reversionary interest in land, which is inconsistent with good husbandry. It is recognized only by the equity courts and does not constitute legal waste. It usually arises when the expression “without impeachment of waste” appears in the grant, which means that the life tenant or tenant for years may use the premises as a fee simple owner might use the Land. Choice (D) is incorrect. Although the brother’s behavior might be construed as an unreasonabLe exploitation of the timber, his conduct is more appropriately termed ameliorating waste. Because choice (B) is a more specific answer, choice (D) is incorrect.
How well did you know this?
1
Not at all
2
3
4
5
Perfectly
63
Q
  1. A woman and her husband purchased a home in1982 and recorded the deed as tenants in common. After 10 years of marriage, the couple began experiencing marital difficulties. In 1992, they separated and the husband moved to another state where his family lived. The woman continued to reside in the dwelling.In 1994, the husband quitclaimed his interest in the home to his son, who promptly recorded his deed. In 1995, the couple divorced. In 1996, the woman changed the locks to the home, preventing the son from entering the property, and the woman refused the son’s repeated demands to allow him access to the home and property. From 1995 to 2005, the woman paid all the taxes and insurance on the property. She also was responsible for making whatever repairs were necessary. The period of adverse possession in this jurisdiction is 10 years.In 2006, the woman filed suit to quiet title to the home claiming absolute fee simple ownership. The son has countersued asserting that he owns an undivided 50 percent interest in the property. If the court rules in favor of the woman, it will be because(A) the son will be unjustly enriched, inasmuch as the woman has paid all the taxes, insurance, and made necessary repairs.(B) the son acquired his interest in the property by quitclaim deed.(C) the deed from the husband to the son was void for lack of consideration.(D) the woman obtained the son’s interest by adverse possession.
A
  1. (D) Choice (D) is correct because possession may become adverse if there is an ouster, in which case, one co-tenant can acquire title against the other. If one tenant wrongfully excludes another co-tenant from possession of the whole or any part of the whole of the property, such conduct amounts to an ouster. By refusing the son’s request to live in the house and by changing the locks, the woman acted in derogation of one of the son’s rights of possession. Choice (A) is incorrect. This choice suggests that the woman would retain an entire fee simple interest in the property for the reason that it would be inequitable to allow the son to have an ownership interest because she paid all the taxes and insurance and made repairs. This would mean that those actions alone would be enough for the woman to retain an entire fee simple interest in the property. Without a proper ouster, the woman cannot own the entire estate just by those actions alone. The woman owns the property because she ousted the son for the entire statutory period. That is why she will prevaiL and not because of her payment of expenses and making repairs. Choice (B) is incorrect. This choice suggests that the woman would prevail simply because the son took via a quitclaim deed. A quitclaim deed will only be reLevant if the son is suing the husband for violation of one of the six deed covenants. However, in the woman’s contest with the son for title to the property, how the son tooktitle is irrelevant to the woman’s success in her quiet title action. Choice (C) is incorrect. Even if the transaction between the husband and the son were somehow invaLid, the fact remains that the husband would then own the other undivided one-half interest in the property. This answer choice still does not explain how the woman acquired title to all of the property.
How well did you know this?
1
Not at all
2
3
4
5
Perfectly
64
Q
  1. An investor and a developer own adjoining lots in the central portion of a city. Each of their lots had an office building on it. The developer decided to raze the existing building on her lot and to erect a building of greater height. The developer has received all governmental approvals required to pursue her project.There is no applicable statute or ordinance (other than those dealing with various approvals for zoning, building, etc.).After the developer had torn down the existing building, she proceeded to excavate deeper. The developer used shoring that met all local, state, and federal safety regulations, and the shoring was placed in accordance with those standards.The investor notified the developer that cracks were developing in the building situated on the investor’s lot. The developer took the view that any subsidence suffered by the investor was due to the weight of the investor’s building and correctly asserted that none would have occurred had the investor’s land been in its natural state. The developer continued to excavate. The building on the investor’s lot did suffer extensive damage, requiring the expenditure of $750,000 to remedy the defects.Which of the following is the best comment concerning the investor’s action to recover damages from the developer?(A) The developer is liable, because she removed necessary support for the investor’s lot.(B) The developer cannot be held liable simply upon proof that support was removed, but may be held liable if negligence is proved.(C) Once land is improved with a building, the owner cannot invoke the right of lateral support.(D) The developer’s only obligation was to satisf’ all local, state, and federal safety regulations.
A
  1. (B) The right of a landowner to have her land supported lateraLLy by the neighboring land is an absolute right inherent in the land itself. The right of lateral support means the land in its natural condition. The right of lateral support does not include the right to have the additional weight of artificial structures supported. If there is damage to artificial structures, the majority view is recovery is limited to damage to the land in its natural condition and may not include any damage to the artificial structures on the land. However, if there is negligence on the part of the wrongdoer who removes lateral support, then the defendant is liable for all damages that proximately result, including damages to both the land and artificial structures. Choice (A) is incorrect. If there is negligence on the developer’s part during her removal of lateral support, then she would be liable for the damage done to the investor’s building. However, there is never strict liability for damage done to artificial structures. This choice falsely suggests that the developer would be liable just because she removed the necessary support, thus creating strict liability on the developer’s part. Because this is not the test, this choice is incorrect. Choice (C) is incorrect. If there is negligence on the developer’s part during her removal of Lateral support, then she would be liable for the damage done to the investor’s building. Because this choice suggests that the investor’s erecting a building on her property has somehow foreclosed any right she has to lateral support, it is incorrect. Choice (D) is incorrect. Even if the developer were in compliance with all local, state, and federal safety regulations, she still owed a duty to the investor to maintain the investor’s right to lateral support. If there were negligence on the developer’s part, she would be Liable to the investor and it would be no defense that she was in compliance with these regulations.
How well did you know this?
1
Not at all
2
3
4
5
Perfectly
65
Q
  1. A landowner and a developer own adjoining lots in the central portion of the city. Each of their lots had an office building built on it. The developer decided to raze the existing building on her lot and to erect a building of greater height. The developer has received all governmental approvals required to pursue her project.When the building was completed, the homeowner discovered that the shadow created by the new higher building placed her building in such deep shade that her ability to lease space was diminished and that the rent she could charge and the occupancy rate were substantially lower. Assume that these facts are proved in an appropriate action that the landowner instituted against the developer for all and any relief available.Which of the following is the most appropriate comment concerning this lawsuit?(A) The landowner is entitled to a mandatory injunction requiring the developer to restore conditions to those existing with the prior building insofar as the shadow is concerned.(B) The court should award permanent damages, in lieu of an injunction, equal to the present value of all rents lost and loss on rents for the reasonable life of the building.(C) The court should award damages for losses suffered to the date of trial and leave open recovery of future damages.(D) Judgment should be for the developer, because the investor has no cause of action.
A
  1. (D) In most jurisdictions, negative easements for light, air, and view have been repudiated, absent an express grant between the parties, because such restrictions would seriously hamper land development. Burby, Real Property, pg. 75. In this question, the developer will prevail because she is not liable for the shadow created by her building. Choice (A) is incorrect. In most jurisdictions, negative easements for light, air, and view have been repudiated. Therefore, the landowner will not be entitled to an injunction against the developer’s construction of her building. Choice (B) is incorrect. In most jurisdictions, negative easements for Light, air, and view have been repudiated. Therefore, the landowner will not be entitled to permanent damages for the developer’s construction of her building. Choice (C) is incorrect. In most jurisdictions, negative easements for light, air, and view have been repudiated. Therefore, the landowner will not be entitled to damages for losses suffered from the developer’s construction of her building.
How well did you know this?
1
Not at all
2
3
4
5
Perfectly
66
Q
  1. In 2001, the owner of a mansion and its surrounding property died. In his will, he devised the property to “my son and his heirs, but if my son dies without issue, to my daughter and her heirs.”Assume that when the owner died, the daughter was still alive. The daughter did not have any children at the time that the owner died. However, 10 years after her father’s death, the daughter had a child.After the owner’s death the daughter’s interest in the property may best be described as a(A) contingent remainder.(B) springing executory interest.(C) shifting executory interest.(D) vested remainder, subject to complete divestiture.
A
  1. (C) After the owner’s death, the daughter’s interest in the property could best be described as a shifting executory interest (or executory devise because it was created by will), which divests the preceding estate upon the happening (or non-happening) of a stated event. Executory devises are interests that are identical with springing and shifting uses, except that they are created by will, instead of by deed. Thus, the owner devised a fee simple to his son and his heirs, which was subject to the daughter’s executory interest. If the event happens (the son dies without issue), then the daughter will have a possessory estate in fee. Choice (A) is incorrect. A remainder is a future interest created in a third person that is intended to take after the natural termination of a preceding estate. A remainder is contingent if it is created in favor of an ascertained person, but (1) is subject to a condition precedent, or (2) is created in favor of an unborn person, or (3) is created in favor of an existing but unascertained person. The daughter’s interest is not a remainder because she does not take after the natural termination of the preceding estate; rather, she takes only upon the happening (or non-happening) of a specified event. This is why her interest is best categorized as an executory interest. Choice (B) is incorrect. A springing executory interest is an estate created to begin in futuro and “cuts short” or terminates a reversion held by the grantor. A springing executory interest is one that divests a grantor of the fee interest. Here, the interest was held by the son and not by the grantor. Thus, the executory interest is of the shifting type, not springing. Choice (D) is incorrect. A remainder is vested subject to complete divestment when the remainderman is in existence and ascertained and her interest is not subject to a condition precedent, but her right to possession or enjoyment on the expiration of the prior interests is subject to termination by reason of(1) an executory interest, or (2) a power of appointment, or (3) a right of entry. However, the daughter does not have a remainder because she does not take after the natural termination of the preceding estate; rather, she takes only upon the happening (or non-happening) of a specified event. This is why her interest is best categorized as an executory interest.
How well did you know this?
1
Not at all
2
3
4
5
Perfectly
67
Q
  1. In 1963, Smith, the record owner of Winterthur died and in his will he devised Winterthur to “my son, Samuel, and his heirs, but if Samuel dies without issue to my daughter, Dixie, and her heirs.” Then Samuel deeded to “Bailey and his heirs,” which was executed on July 1, 1973 and recorded on the same date.In 1974, Bailey died and in his will he devised Winterthur to “my brother Corley for life, then to Corley’s heirs.” In 1983, Corley executed a general warranty deed in which he conveyed “all my right, title and interest in Winterthur to my children, Jesse and James.” James died intestate in 1988, leaving his 14-year-old son, Butch, as his only heir. Corley died in 1993, leaving Butch and Jesse as heirs.In 1998, Butch and Jesee conveyed Winterthur by a general warranty deed to “Barnes and his heirs.” Although both Butch and Jesse were married, their respective wives did not join in the deed. Barnes has been in continuous possession of the home since 1998. Samuel died on January 1, 2009 without issue, and Dixie is still alive.In March 2009, Dixie, asserting that her title was held free of any claim by Barnes, instituted suit against Barnes to quiet title to Winterthur. Judgment should be for(A) Barnes, because his prior recorded deed would be deemed to be outside Dixie’s chain of title.(B) Barnes, because the devise to Dixie would be violative of the Rule against Perpetuities.(C) Dixie, because she acquired fee simple interest at the time of Samuel’s death in January 2009.(D) Dixie, because she acquired an indefeasible vested remainder under her father’s will in1963.
A
  1. (C) In a suit to quiet title to the home, Dixie would prevail. At the time of Samuel’s death on January 1, 2009 (without issue), Dixie’s executory interest was executed into a legal estate in fee. In fact, Dixie’s interest vested automatically upon the Samuel’s death without issue, without any affirmative act on her part. Choice (A) is incorrect. Dixie is the holder of a shifting executory interest. When Samuel died without issue, title vested in Dixie. The issue in this question is whether or not Dixie takes as a future interest holder. Chain of title issues will arise when competing grantees are trying to acquire, title to the same present possessory estate. Dixie, as a future interest holder, has no one to compete with because as soon as Samuel died without issue, title was vested in her. Because there is no competition for which grantee holds title, chain of title is not in issue here. Choice (B) is incorrect. The Rule against Perpetuities would apply to Dixie’s shifting executory interest. Furthermore, Samuel will be a valid measuring life in this situation. Upon Samuel’s death, it will be known for certain whether the condition was met or not. Because Samuel died without issue, Dixie’s executory interest will vest immediately and will not violate the Rule against Perpetuities. Choice (D) is incorrect. A remainder is a future interest created in a third person which is intended to take afterthe naturaltermination of a preceding estate. However, Dixie does not have a remainder because she does not take after the natural termination of the preceding estate; rather, she takes only upon the happening (or non-happening) of a specified event. This is why her interest is best categorized as an executory interest.
How well did you know this?
1
Not at all
2
3
4
5
Perfectly
68
Q
  1. On January 1, a seller entered into an agreement of sale to convey his farm to a buyer. According to the contract, settlement (or closing) was to take place on March 1 of the same year.On January 20, a barn on the property was destroyed by fire. When the buyer was notified of the loss the following day, he contacted the seller and told him that the deal was off because of the fire. As a result, the buyer filed suit against the seller on January 25 for a refund of his down payment of $5,000.In his suit, the buyer will be unsuccessful and will be obliged to complete the contract if the court determines that on January 20(A) the buyer held the legal title to the farm.(B) the seller held the legal title to the farm.(C) the buyer held the equitable title to the farm.(D) the seller held the equitable title to the farm.
A
  1. (C) Equitable conversion applies when there is an enforceable obligation to sell land. The doctrine treats interests in land as if the land had already been converted into personal property. The doctrine states that when the sales contract is made, equity then considers the vendee as the owner of the land and the vendor as the owner of the purchase money. Applying the facts of the case to this doctrine, the buyer would be construed as the equitable owner of the land on January 20 when the barn was destroyed by fire. As a result, the buyer (as equitable owner) would become subject to all losses not occasioned by the fault of the seller. Therefore, if the court determined that the buyer held equitable title to the farm on January 20, he would be unsuccessful in his Lawsuit against the seller. Choice (A) is incorrect. Under equitable conversion the purchaser is regarded as the “equitable” owner of the land, although legal title remains with the seller. Under this doctrine, the seller would be the party to hold legal title, not the buyer. Because this choice misstates the type of title held by the buyer, it is incorrect. Choice (B) is incorrect. Under the doctrine of equitable conversion, the seller would be the party to hold legal title, not the buyer. However, this does not fully explain why the buyer would be unsuccessful in his suit and be required to complete the contract of sale. The most accurate reason why the buyer would be required to go through with the deal is that he held equitable title to the farm, making (C) the best choice. Choice (D) is incorrect. Under equitable conversion, the purchaser is regarded as the “equitable” owner of the land, although legal title remains with the seller. Under this doctrine, the seller would be the party to hold Legal title, not equitable title.
How well did you know this?
1
Not at all
2
3
4
5
Perfectly
69
Q
  1. An architect is the owner of a large two-story office building. The architect designed and supervised the building’s construction. He and his associate occupy the first floor for their architectural business. In June 2005, the architect entered into a valid and binding written lease with a tenant to rent the second floor office space for four years at a monthly rental of $5,000. The lease contained a provision wherein the architect was required to repaint the second floor premises after two years of occupancy.On June 1, 2007, the tenant sent the architect a written letter requesting the repainting of the second-floor office space. The architect made no reply and failed to perform the repainting as the lease required. The cost of repainting the tenant’s office was estimated at $1,750. On July 1, 2008, the architect had still not repainted the premises. The following day, the tenant moved out, mailed the keys to the architect, and refused to pay any more rent. The lease was silent as to the rights and remedies of the tenant due to the architect’s failure to perform the repainting. There is no applicable statute in the jurisdiction.In an appropriate action by the architect against the tenant for the rent due, the architect will(A) win, because there was no constructive eviction.(B) n, because the tenant had the remedy of self-help.(C) lose, because he cannot maintain an action for rent while in breach of an express covenant.(D) lose, because the obligation to pay rent is dependent on the architect’s performance of his express covenant.
A
  1. (A) At common law, because a tenant was deemed to receive an estate in land, his rights and duties were treated as independent of the Landlord’s rights and duties. Thus, if the landlord promised to keep the property in repair, a breach of this promise did not relieve the tenant from the duty of paying rent. According to the common law rationale, the rent was owed as payment for the estate, and the promise to do repairs was merely a colLateraL promise, which could be enforced only by a separate contractual suit brought by the tenant. This doctrine is generally referred to as the independence of covenants. Choice (B) is incorrect. (B) is wrong; the reason the Landlord will prevail is because the premises were not so uninhabitabLe (the basis for constructive eviction) as to force the tenant out and relieve the tenant of the duty to pay rent. This choice falsely suggests that the tenant had the remedy of self- help and failed to avail himself of that remedy by failing to perform the re-painting himself. In other words, self-help is not an appropriate remedy inasmuch as the tenant had no duty to make such repairs. Choice (C) is incorrect. Because the duty to pay rent was independent of the landlord’s rights and duties, a breach of the promise to keep the premises in repair still did not relieve the tenant from the duty of paying rent. This choice states that the landlord will lose in his action to collect rent because the architect cannot collect it while in breach of an express covenant. However, because the duty to pay rent is independent of the covenant to paint, the architect can sue for the rent and treat that as separate from his own failure to paint the premises. Because this choice states that a landlord cannot do both at the same time, it is incorrect. Choice (D) is incorrect. Because the duty to pay rent was independent of the landlord’s rights and duties, a breach of the promise to keep the premises in repair still did not relieve the tenant from the duty of paying rent. Because this choice states that the obligation to pay rent is dependent on the covenant to repair, it is incorrect.
How well did you know this?
1
Not at all
2
3
4
5
Perfectly
70
Q
  1. In a jurisdiction using the grantor—grantee indices, the following events have occurred in the order listed below:(1) In 1962, a grantor conveyed to a man a tract of land by a deed that was immediately recorded.(2) In 1982, the man died a widower and devised the tract of land to his assistant by a will that was duly admitted to probate.(3) In 1993, the assistant mortgaged the tract of land to a mortgage company to secure a note for $25,000, due on December 31, 1993, with 9½ percent interest per annum.(4) In 1996, the mortgage company recorded the mortgage.(5) In 2003, in a signed writing, which the assistant delivered to his daughter, the assistant promised to convey the tract of land to the daughter by a quitclaim deed and pay the mortgage debt when it came due.If the assistant refused to convey the tract of land to the daughter and the daughter brings suit against the assistant to compel a conveyance of the property, the daughter will most likely(A) prevail, because there was an effective gift.(B) prevail, because there was an effective assignment.(C) not prevail, because there was no consideration to support the assistant’s promise to convey the property.(D) not prevail, because specific performance will not be granted where there is an adequate remedy at law.
A
  1. (C) The statute of frauds requires a writing identifying the parties, describing the subject matter, stating the purchase price, and signed by the party to be charged. The writing the assistant deLivered to the daughter did not state any consideration —it was intended as a gift. Furthermore, it was not an effective gift because it was only a promise to convey. An effective gift requires a present transfer of an interest. If the transfer is to take effect in futuro, it is a mere promise to make a gift and unenforceable for lack of consideration. Smith and Boyer, Survey of the Law of Property, p. 469. Choice (A) is incorrect. There was no effective gift because this was a mere promise to make a gift. Moreover, this is unenforceable for lack of consideration. Because this choice states that the daughter would prevaiL, it is incorrect. Choice (B) is incorrect . This choice is incorrect because the assistant’s writing did not create an assignment. Choice (D) is incorrect. The daughter does not have an adequate remedy at law because there is a lack of consideration. Specific performance will not be granted due to a Lack of mutuality.
How well did you know this?
1
Not at all
2
3
4
5
Perfectly
71
Q
  1. In a jurisdiction using the grantor—grantee indices, the following events have occurred in the order listed below:(1) In 1993, a woman mortgaged a tract of land to a mortgage company to secure a note for $25,000, due on December 31, 1993, with 9’/2 percent interest per annum.(2) In 1996, the mortgage company recorded the mortgage on the tract of land.(3) In 2007, in a signed writing, which the woman delivered to her daughter, the woman promised to convey the tract of land to her daughter by quitclaim deed and pay the mortgage debt when it came due.(4) In 2008, the woman delivered to the daughter a quitclaim deed that made no reference to the mortgage.If the woman thereafter defaulted on the mortgage debt, which of the following is probably correct?(A) Only the daughter has a cause of action against the woman.(B) Only the mortgage company has a cause of action against the woman.(C) Both the daughter and the mortgage company have causes of action against the woman.(D) Neither the daughter nor the mortgage company has a cause of action against the woman.
A
  1. (B) A mortgagee has both an in personam claim against the mortgagor on the debt or obligation and an in rem action against the security. As a result, choice (B) is correct because the mortgagee (the mortgage company) has an in personam action against the mortgagor (the woman). Choice (A) is incorrect. A quitclaim deed usually contains no assurances; it simply operates as a transfer to the grantee of whatever interest, if any, the grantor happens to own. Because no deed covenants were made, the woman made no promises to the daughter that the mortgage debt would be paid. So, if the daughter tries to pursue a claim against the woman for failure to pay the mortgage, she will not prevail. Choice (C) is incorrect. A quitclaim deed usually contains no assurances; it simply operates as a transfer to the grantee of whatever interest, if any, the grantor happens to own. Because no deed covenants were made, the woman made no promises to the daughter that the mortgage debt would be paid. So, if the daughter tries to pursue a claim against the woman for failure to pay the mortgage, she will not prevail. Choice (D) is incorrect. A mortgagee has both an in personam claim against the mortgagor on the debt or obligation and an in rem action against the security. Because the mortgage company does have an in personam action against the woman, this choice is incorrect.
72
Q
  1. In a jurisdiction using the grantor—grantee indices, the following events have occurred in the order listed below:(1) In 1993, a borrower mortgaged a ranch to a mortgage company to secure a note for $25,000, due on December 31, 1993, with 9V2 percent interest per annum.(2) In 1996, the mortgage company recorded the mortgage on the ranch.(3) In 2007, in a signed writing, which the borrower delivered to his daughter, the borrower promised to convey the ranch to the daughter by a quitclaim deed and pay the mortgage debt when it came due.(4) In 2008, the borrower delivered to the daughter a quitclaim deed that made no reference to the mortgage.If the borrower thereafter defaulted on the mortgage and the mortgage company brought an in personam action against the daughter to recover the amount due on the mortgage debt, the mortgagee will probably(A) succeed, because the mortgage company is a third-party beneficiary of the agreement between the borrower and the daughter.(B) succeed, because there was an implied delegation of duties to the daughter.(C) not succeed, because the daughter did not promise to pay the mortgage debt.(D) not succeed, because the law does not permit the mortgagor to delegate duties under this type of a property transfer.
A
  1. (C) In this case, the mortgage company, the mortgagee, is bringing a claim against the daughter, the grantee. Because the facts state that the deed “made no reference to the mortgage,” it is presumed that the daughter took “subject to” the borrower’s mortgage. Therefore, the daughter will not be personally liable, because she did not promise to pay the mortgage debt. Choice (C) is correct. Note: Only the borrower as mortgagor would be liable to the mortgage company under these facts. Choice (A) is incorrect. Because the deed “made no reference to the mortgage,” the daughter took “subject to” and will not be personally liable. So, there was never an agreement between the borrower and the daughter. Because there was no agreement between the borrower and the daughter, there was no contract to which the mortgage company could be a third-party beneficiary. Choice (B) is incorrect. Because the deed “made no reference to the mortgage,” the daughter took “subject to” and will not be personally liable. She is not liable and no duty was delegated to her, whether expressly or impliedly. Choice (D) is incorrect. Because the deed “made no reference to the mortgage,” the daughter took “subject to” and will not be personally liable. She is not liable and no duty was delegated to her.
73
Q
  1. A widower was the owner in fee of a 10-acre quarry. On August 1, the widower handed his friend a deed conveying the quarry to his niece and said to the friend, “Give this to my niece.” Unknown to the widower, the niece was in Africa.The next week, the friend properly recorded the deed with the Recorder’s Office. One month later, the widower died, and in his will he devised the quarry to his daughter. Thereafter, the niece returned from Africa, and a dispute arose between the niece and the daughter regarding ownership of the quarry.In a suit to quiet title to the quarry, the court ruled in favor of the niece. The ruling was most likely based upon which of the following?(A) There was an effective “causa mortis” gift.(B) The delivery was effective when the widower handed the deed to the friend.(C) The delivery was effective when the friend recorded the deed.(D) The deed was recorded before the devise was effectuated.
A
  1. (B) Deliveryof a deed means an intention on the part of the grantor that it shall operate or take effect as a conveyance. In this area, you will frequently encounter situations where the grantor gives a deed to a third party. There are three ways third-party delivery may be tested on the MBE. First, where A (the grantor) gives C a deed naming B as grantee and instructs C to give the deed to B. Here, there are no conditions and the delivery is valid. Second, A gives C a deed naming B as grantee and tells C to transfer the deed to B when B pays A $50,000 on December 1. A vaLid conditional delivery has occurred and title will automatically transfer upon the fulfillment of the condition. Third, A gives C a deed naming B as grantee and instructs C to give it to B “when he graduates from law school.” In this type of donative transaction, the delivery is valid. The key issue in donative transactions is determining whether the grantor has divested himself and relinquished control of the deed. Depending upon the amount of control relinquished by the grantor, the delivery may be valid or invalid. Choice (B) is correct because this is an example of a delivery to a third party with no conditions. Choice (A) is incorrect. Choice (A) is wrong because a “causa mortis”giftis made at the time of the testator’s death (or a “deathbed” gift). Choice (C) is incorrect. When the friend was instructed to give the deed to the niece, that instruction alone was enough to effectuate the delivery. The delivery of the deed was not dependent upon the friend’s recordation of it. Because the delivery took place earlier than this answer choice suggests, it is incorrect. Choice (D) is incorrect. This choice falsely suggests that the recording of the deed is the reason why there was a valid delivery. The niece prevails here because the deed was effectively delivered when the widower handed it to the friend with no conditions attached.
74
Q
  1. A landlord owned an apartment which he leased to a college senior for a three-year term beginning December 1, 2007, for a monthly rental of $1,000 to be payable on the first of each month. After the college senior resided in the apartment for six months, he assigned his interest in the premises to a college junior on June 1,2008. The college junior paid the rental on the apartment for two months and then assigned his interest to a college sophomore. The sophomore took possession on August 1, 2008 and paid the rental for August but did not pay the rental for the next three months of occupancy. Then on December 1, 2008 the college sophomore assigned the premises to a college freshman who took possession but didn’t pay any rental to the landlord during the period of his occupancy from December 1,2008 to April 1,2009.On April 5, 2009, the landlord brings an action against the college freshman to recover the outstanding rent that is due on the premises. The freshman is most likely liable for(A) $8,000.(B) $7,000.(C) $5,000.(D) $4,000.
A
  1. (C) An assignee of a leasehold is in privity of estate with the landlord. Liability for rent and for observing the other covenants that run with the leasehold can be based on privity of estate as well as privity of contract. Smith and Boyer, Real Property, p. 149. Therefore, a landlord can recover from the assignee rent obligations occurring while privity of estate continues or until the assignee in turn assigns the leasehold to another party. The college freshman, who has occupied the premises for five months, will be liable for that rent accruing while he was the sophomore’s assignee—December, January, February, March, and April—but not for any back rent during the sophomore’s tenancy. Choice (A) is incorrect. The freshman, the subsequent assignee, is not liable for the predecessor’s (the sophomore’s) unpaid rent. For this reason, the freshman is liable only for the unpaid rent during his own tenancy. Because this dollar amount exceeds the freshman’s liability, it is incorrect. Choice (B) is incorrect. The freshman, the subsequent assignee, is not liable for the predecessor’s (the sophomore’s) unpaid rent. For this reason, the freshman is liable only for the unpaid rent during her own tenancy. Because this dollar amount exceeds the freshman’s liability, it is incorrect. Choice (0) is incorrect. The facts state that rent was payable in advance on the first of each month. As a result, because the freshman was in possession on April 1, he will also be held liable for April’s rental. Therefore, the freshman owes $5,000, an.d not $4,000.
75
Q
  1. A developer was the owner of a 300-acre tract of land that was located in a rural county. Over the course of time, she developed the property into a residential subdivision. A four-lane public highway ran along the northern boundary of the residential subdivision. When the residential subdivision was first plotted, the developer constructed a private road along (and including) the western boundary of the subdivision. This private road was used only for the benefit of the owners who purchased lots in the subdivision.A homeowner owned a 20-acre tract immediately outside of the residential subdivision, which was situated just below the public highway and immediately adjacent to the west side of the private road. The homeowner’s property was divided into two 10-acre parcels: Parcel 1 was the northern half and bordered along the public highway; Parcel 2 was the southern sector and abutted a river. In 1997, the homeowner conveyed Parcel 1 to a man by warranty deed. No encumbrances were mentioned in the deed. Six months later, the homeowner conveyed Parcel 2 by warranty deed to a woman. Both the man and the woman promptly recorded their deeds with the County Recorder’s Office.Assume that no part of Parcel 2 adjoins a public road. Consequently, the woman constructed an access road connecting her property to the private road. After the woman used the private road for approximately two months, a teacher who was an owner within the residential subdivision, ordered the woman to discontinue using the private road. In an appropriate action by the teacher against the woman to enjoin the use of the private road, the woman will(A) win, because there is an easement by estoppel over the private road.(B) win, because there is an implied easement appurtenant over the private road.(C) lose, unless the private road is the only access road by which woman can gain ingress and egress to her property.(D) lose, because she has no legal right to travel over the private road.
A
  1. (D) In the teacher’s action to enjoin the woman’s use of the private road, the woman will lose because she has no legal right to drive over the private road. The developer constructed the road as a private road for use only by owners of the lots in the residential subdivision. The road was not intended to benefit outside landowners, such as the woman who did not reside in the subdivision. Choice (A) is incorrect because an easement by estoppel requires proof of an act or representation by the owner of the burdened estate in respect to the easement, justifiable reliance on that act or representation by the owner of the benefited estate, and damages suffered by the owner of the benefited estate if the easement is not recognized. These factors are not evidenced by the facts. Choice (B) is incorrect because an implied easement is based on the inferred intention of the parties, namely the grantor and the grantee. Use of the private road arises between the developer, the grantor, and hergrantees, such as the teacher. However, the woman, on the other hand, was the homeowner’s grantee and constructed the access road to the private road after she purchased Parcel 2 from the homeowner. Choice (C) is incorrect. This choice falsely suggests that if the private road were the only access road by which the woman could gain ingress and egress that fact would somehow be enough to allow the woman to use the road. Even if that were the only method, the woman still has no right to use the private road because the developer constructed that road as a private road for use only by owners of the Lots in the residential subdivision. The road was not intended to benefit outside Landowners, such as the woman, who chose to connect up to the private road to use it for their own benefit.
76
Q
  1. A farmer was the owner of a 20-acre tract that was situated just south of a major boulevard, a street that ran east and west. The farmer’s tract was located next to an adjacent 300-acre ranch to the east. Immediately to the west of the farmer’s parcel was a 100-acre tract. The farmer subdivided his property into two 10-acre parcels:Parcel 1 was the northern half and bordered along the major boulevard; Parcel 2 was the southern sector and abutted a river to the south. In March 1997, the farmer conveyed Parcel ito a man by warranty deed. No encumbrances were mentioned in the deed. Then in September 1997, the farmer conveyed Parcel 2 by warranty deed to a woman. Both the man and the woman promptly recorded their deeds with the County Recorder’s Office.In 1965, the farmer constructed a driveway from what is now Parcel 2, extending over Parcel 1, where it connected with the major boulevard. This driveway lies across what is now Parcel 1 and was used continuously by the farmer until 1997 when he conveyed his property in parcels to the man and the woman. When the woman took possession of Parcel 2, she immediately began to use the drive way across Parcel 1. Thereafter, the man requested that the woman cease to use the driveway over Parcel 1.After the woman refused, the man brought an appropriate action to enjoin the woman from using the driveway. In this action, the man will(A) win, because his deed antedated the woman’s deed.(B) win, because no encumbrances were mentioned in his deed.(C) lose, because the woman has an easement by necessity.(D) lose, because the woman has an express easement.
A
  1. (C) The man wilL not be able to enjoin the woman’s use of the driveway across Parcel1 because the woman has an easement by necessity. Choice (C) is correct. An easement by necessity requires: (1) severance of title to land held in common ownership; and (2) strict necessity for the easement at the time of severance. At the time of the farmer’s conveyance of Parcel 1 to the man, Parcel 2 had no other access to a public road, thereby allowing the farmer an easement by strict necessity over the man’s land. This easement passed to the woman upon the farmer’s conveyance to her of ParceL 2. Choice (A) is incorrect. The man will not prevaiL because of the existence of an easement by necessity across Parcel 1. Because this choice falsely suggests that a prior deed forecLoses the dominant tenant from using the easement by necessity, it is incorrect. Choice (B) is incorrect. Even though no encumbrances were mentioned in the deed, an easement by necessity was created. Easements do not need to be expressly mentioned in a deed or expressly granted. Where, as here, the easement arose by necessity, the servient tenant will have to abide by the easement even though it was not mentioned in his deed. Choice (D) is incorrect because the woman was never expressly granted an easement across what is now the man’s parcel.
77
Q
  1. A rancher is the owner of a ranch that is situated upon the top of a mountain. Located below the ranch is an estate that is owned by a millionaire. A stream is a non-navigable watercourse that originates at the top of the mountain and runs all the way down into a valley. Both the ranch and the estate are within the watershed of the stream.When the millionaire purchased the estate in 1956, he started taking water from the stream and used it to irrigate the southern half of his property, which he has used as a farm. Prior to 1956, the southern half of the estate had been cleared and placed in cultivation, while the northern half remained wooded and virtually unused. The millionaire continued this established pattern of use and has never stopped using the water in this way.In 1986, the rancher built a home on the ranch and started talcing water from the stream for domestic purposes. During that year there was heavy rainfall, and this caused the stream to run down the mountain at a high water level. However, in 1987, a drought caused the stream to flow at a very low level. Consequently, there was only enough water to irrigate the millionaire’s farmland or, in the alternative, to supply all of the rancher’s domestic water needs and one-quarter of the millionaire’s irrigation requirements. The mountain is located in a jurisdiction where the period of prescription is 15 years.The rancher is continuing to take water for his personal needs and there is insufficient water to irrigate the estate. The millionaire then brings an appropriate action in 1996 to declare that his water rights to the stream are superior to those of the rancher. In addition, the millionaire moves to have the full flow of the stream passed to him, notwithstanding the effect it might have on the rancher.If this state follows the common law of riparian rights, but does not follow the doctrine of prior appropriation, judgment should be for whom?(A) The rancher, because as an upstream landowner, he would have superior rights to the water than a downstream owner.(B) The rancher, because domestic use is superior to and is protected against an agricultural use.(C) The millionaire, because he obtained an easement by prescription to remove as much water as he may need.(D) The millionaire, because he put the water to a beneficial use prior to the rancher’s use and has continuously used the water.
A
  1. (B) Under the doctrine of riparian rights there are two theories: the “natu ral flow” theory and the “reasonable use” theory. The former stresses the right of each owner to have the natural state of the stream or lake undiminished in both quality and quantity. The latter stresses maximum use by each owner provided it does not interfere with like use by other owners. Also, under the riparian doctrine, use of water for natural purposes—domestic use, watering of stock—is superior to use of water for artificial purposes—irrigation, mining, industry. Smith and Boyer, Survey of the Law of Property, pp. 186—189. Choice (A) is incorrect because the theory of riparian rights allows a lower riparian (the millionaire) to sue an upper riparian (the rancher) where the upper’s use materially affects the quantity or quality of water (the natural flow) or unreasonably causes damages (reasonable use). This answer choice has reversed the parties and is incorrect. Choice (C) is incorrect because this falsely suggests that one can obtain water rights by a prescriptive easement. Prescriptive easements are more appropriate for those situations when one person is trying to claim an easement in the property of another by showing that s/he meets elements similar to those of adverse possession. When dealing with water rights, it is more correct to use either the riparian or prior appropriations doctrine to argue that someone has obtained paramount rights in a stream or lake that is used by many. Choice (D) is incorrect because it states the rule underthe prior appropriation theory.
78
Q
  1. In 1981, a devoted conservationist, was the owner of a 100-acre tract of undeveloped land. In that year, the conservationist conveyed the tract “to my nephew in fee simple, provided, however, that the grantee agrees that neither he nor his heirs or assigns shall ever use the property for any commercial purpose. If any portion of said tract is used for other than residential purposes, then the grantor or his successors in interest may re-enter as of the grantor’s former estate.” This deed was properly recorded. The nephew died intestate in 1999, survived by his wife. The conservationist died in 2002, survived by his two daughters, his only heirs.During the period between 1981 and 2007, the spreading development from a nearby city began to engulf the tract. Though still undeveloped, the tract became surrounded by office buildings, shopping malls, and other commercial edifices. In 2009, the wife executed and delivered to a developer a fee simple conveyance of the tract, which the developer immediately recorded. The deed did not contain any reference to the restriction noted above. After the developer acquired title to the tract, he commenced construction of a hotel complex on a portion of the tract that bordered an apartment building. The applicable recording statute in effect in this jurisdiction provides, in part, “No deed or other instrument in writing, not recorded in accordance with this statute, shall affect the title or rights to, in any real estate, or any devisee or purchaser in good faith, without knowledge of the existence of such unrecorded instruments.”If one of the daughters brings suit to enjoin the developer from constructing the hotel, the plaintiff will most likely(A) win, because either daughter’s right to the tract vested immediately upon the developer’s construction of the hotel complex.(B) win, because either daughter has the right of re-entry for condition broken.(C) win, because the daughters, as the conservationist’s only heirs, received a valid possibility of reverter from their father.(D) lose, because a common development scheme had been established for the entire tract.
A
  1. (B) In this example, the nephew has a fee simple subject to a condition subsequent, which is an estate in fee simple that may be terminated by the conveyor or those claiming under him upon the happening of a named event. The important characteristic that distinguishes this type of estate from a fee simple determinable is that the estate will continue in the grantee, or his successors, unless and until the power of termination is exercised. The basic difference, therefore, is that the fee simple determinable automatically expires by force of the special limitation contained in the instrument creating the estate, when the stated contingency occurs, whereas the fee simple on condition subsequent continues despite the breach of the specified condition until it is divested or cut short by the exercise by the grantor of his power to terminate. Although no particular words are essential to create an estate on condition subsequent the use in the conveyance of the traditional words of condition, “upon condition that,”“provided that,” and “but if,” coupled with a provision for re-entry bythe grantor will be construed to create an estate on condition subsequent. Choice (A) is incorrect. The fee simple subject to condition subsequent that the developer owns will continue on until it is divested by the daughter’s exercise of the right of re-entry. Only with a possibility of reverter does the initial fee simple terminate automatically. Because the developer’s fee interest in the tract will not terminate immediately upon his breaking of the condition, this choice is incorrect. Choice (C) is incorrect. The daughters, as the conservationist’s only heirs, received a right of re-entry in the tract, not a possibility of reverter. Choice (D) is incorrect. Certainly, the nearby city was changing, but nowhere does it say in the facts that there was ever a common development scheme for the tract itself. Because this choice is wrong on the facts presented, it is incorrect.
79
Q
  1. In 1954, a devoted conservationist, was the owner of a tract of undeveloped land. In that year, the conservationist conveyed the tract “to my aunt in fee simple, provided, however, that the grantee agrees that neither she nor her heirs or assigns shall ever use the property for any commercial purpose. If any portion of said tract is used for other than residential purposes, then the grantor or his successors in interest may re-enter as of the grantor’s former estate.” This deed was properly recorded.The applicable recording statute in effect in this jurisdiction provides, in part, “No deed or other instrument in writing, not recorded in accordance with this statute, shall affect the title or rights to, in any real estate, or any devisee or purchaser in good faith, without knowledge of the existence of such unrecorded instruments.”The aunt developed the tract into a residential subdivision between 1960 and 1969. This 100-acre tract was divided into single acre lots, all of which were sold during the same decade. Single-family residential dwellings were constructed on all of the lots except four, which were sold to an investor in 1968. The deed to the four lots contained the same restriction as stipulated in the deed between the conservationist and the aunt. In 1980, a bank acquired the investor’s four lots in a foreclosure proceeding, and in the same year sold the property to a woman. The deed to the woman made no mention of the restriction.If the woman seeks to construct a service station on her parcel of land, which of the following arguments, if true, would offer her the best chance of implementation?(A) The restriction against residential use is void as violative of the Rule against Perpetuities.(B) The commercial development surrounding the property has made the deed restriction unenforceable.(C) When the woman purchased the property from the bank, the deed did not contain any mention of the restriction.(D) The jurisdiction in which the tract is located approved a zoning ordinance permitting the commercial development of the area surrounding the woman’s property.
A
  1. (B) An equitable servitude may be extinguished by the existence of conditions that make the purpose and object of the servitude impossible of achievement, such as change in the character of the neighborhood from a residential to a business area. Though it is not certain that a court would be convinced by this argument, it is the strongest argument of four to explain why the woman should be excused from complying with the restriction. Choice (A) is incorrect. The Rule against Perpetuities applies to (1) contingent remainders, (2) vested remainder subject to open, (3) executory interests, (4) powers of appointment, (5) rights of first refusal, and (6) options to purchase land not incident to a lease. The Rule does not apply to this deed restriction. Choice (C) is incorrect. This is a weak argument because even though the deeds made no mention of the restriction, a court will still imply a restrictive covenant. There was a common scheme in that 96 of the 100 lots all contained single-family residences, and the uniform characteristic of the neighborhood would likely have put the woman on inquiry notice of the restrictive covenant. Choice (D) is incorrect because a zoning ordinance will not generally invalidate an equitable servitude.
80
Q
  1. A womam owned a farm. She granted to a neighbor, the owner of an adjacent tract, an easement of way. The woman then went to Japan to live. The neighbor then moved into possession of the farm and used it openly and exclusively, paying the taxes for 23 years. He did not use his easement during that period. The woman returned and tried to evict the neighbor from the farm. The court held that the neighbor had acquired the farm by adverse possession. The neighbor then sold the farm back to the woman, who then put a chain across the easement. The neighbor has now brought an action to remove the chain.In most jurisdictions, the neighbor will(A) lose, because he abandoned his easement.(B) lose, because he did not use his easement for the statutory period of 20 years.(C) lose, because, rather than using his easement, he used all of the farm.(D) win, because mere non-use of an easement does not extinguish it.
A
  1. (C) This is an example of the extinguishment of an easement by merger. An easement appurtenant is terminated by merger if the dominant and servient tenements come into common ownership. Easements (or profits) may be terminated in the following ways: (1) by merger, (2) by written release, (3) by non-use plus other acts indicating an intent to abandon, (4) by prescription, (5) by destruction of servient tenement, (6) by tax sale of servient tenement, or (7) by estoppel. When the court held that the neighbor had acquired the farm by adverse possession, this merger of the estates extinguished the easement. If the neighbor had simply restricted his use to the pathway he had been granted across the farm, there wou’d have never been a merger of the two estates and the easement would have stayed exactly as it was. However, because he chose to adversely possess the entire the farm and received a judicial decree giving him title to all of the farm, there is no more easement across the farm. Because the easement was not granted again, the neighbor can no longer cross the farm. Choice (A) is wrong because failure to use an easement or profit will not result in its extinguishment. Note, however, that non-use coupled with physical acts (clearly indicating the user’s intent to abandon) may be sufficient to effectuate an abandonment. Choice (B) is incorrect. Failure to make use of an easement forthe statutory period does not automatically mean that the easement holder would lose their right to use that land. If there had been non-use coupled with physical acts (clearly indicating the user’s intent to abandon), that may have been sufficient to effectuate an abandonment. However, time alone will not effectuate an abandonment even if that duration is for the statutory period. Choice (0) is incorrect. While this statement is true in that mere non-use of an easement does not extinguish it, the fact remains that the neighbor’s acquisition of the farm did result in a merger of the servient and dominant tenements, therefore resulting in an extinguishment by another means. Because this choice states that the neighbor will prevail and because this choice fails to take into account the merging of the two estates, it is incorrect.
81
Q
  1. A cattle rancher owned substantial oil and mineral interests in the southwest. In July 1990, the rancher acquired title to a 200-acre tract in a rural area.In 1997, the rancher conveyed the 200-acre tract for a consideration of one dollar, receipt of which was acknowledged, “to my uncle, his heirs, and assigns, but if my uncle’s wife dies without issue, to my personal assistant and her heirs and assigns.”After taking possession of the 200-acre tract, the uncle discovered considerable oil reserves on the property. He then began oil drilling operations and removed large quantities of oil, which he sold. At no time did he give notice to the personal assistant of his oil-drilling activities. Months passed before the personal assistant learned that the uncle was selling oil from the drilling operations.The personal assistant has now brought an action in equity for an accounting of the value of the oil removed and for an injunction against any further oil removal. If the decision is in favor of the uncle, it will be based on the fact that(A) the right to remove oil is an incident of a defeasible fee simple.(B) the personal assistant has no interest in the 200-acre tract.(C) there was no showing that the uncle acted in bad faith.(D) the right to remove oil is an incident of the right to possession.
A
  1. (A) or (D) Choice (A) is correct because the right to remove oil is an incident of possession of any fee simple interest, whether a fee simple absolute or a fee simple defeasible. Removal of oil turns on the uncle’s right of exclusive possession. Furthermore, the personal assistant, as the holder of the shifting executory interest, has no vested interest in the 200-acre tract but rather a mere expectancy, until such time that the wife might die without children. Until then, the uncle can remove oil and his doing so as a fee simple owner does not constitute waste. Choice (B) is incorrect. It is incorrect to say that the personal assistant has no interest in the 200- acre tract. Her future interest is a shifting executory interest in the 200-acre tract. Choice (C) is incorrect. While it may be true that there was no showing that the uncle ever acted in bad faith, there is a more concrete reason why the uncle would prevail. The uncle prevails because the right to remove this oil is one of his rights as the owner of a fee interest in land. Because the personal assistant has, at most, a mere expectancy interest in the property, she will not prevail in her action against the uncle. Choice (D) is also correct. The owner of a possessory estate in land is under a duty to use his possessory rights in a reasonable manner. Physical damage to the land is waste if it causes a substantial diminution in the value of estates owned by others in the same land. Burby, Real Property, p. 33. However, the owner of an estate in fee simple has almost complete freedom of action with respect to the use and enjoyment of the land. The uncle is a fee simple owner and as an incident of possession has “the right to the exclusive possession of the surface of the ground, the airspace above and the soil underneath.” Smith and Boyer, Survey of the Law of Property, p. 71.
82
Q
  1. An investor owned substantial oil and mineral interests in the Rocky Mountains. In July 1990, the investor acquired title to a 100-acre tract located just outside the city limits in a rural area.In 1997, the investor decided to sell his vast ranching and oil interests in the Rocky Mountains. In July, the investor conveyed the tract for a consideration of one dollar, receipt of which was acknowledged, “to my nephew, his heirs, and assigns, but if my nephew’s wife dies without issue, to my chef and her heirs and assigns.”After taking possession of the 100-acre tract, the nephew discovered considerable oil reserves on the property. He then began oil drilling operations and removed large quantities of oil, which he sold. At no time did he give notice to the chef of his oil-drilling activities. Months passed before the chef learned that the nephew was selling oil from the drilling operations.The chef has now brought an action in equity for an accounting of the value of the oil removed and for an injunction against any further oil removal.If the nephew’s wife died without issue before the beginning of the nephew’s oil drilling operations, would this improve the likelihood of the chef’s success than if the wife were still alive?(A) No, because the right to remove oil is an incident of the right to possession.(B) No, because the nature of the nephew’s estate would not be altered by the wife’s death.(C) Yes, because the wife’s death without issue would convert the nephew’s fee into a reversionary interest.(D) Yes, because although the nephew still has a fee, it would now be certain to terminate.
A
  1. (D) In this Multistate example, all four answer choices are technically incorrect. However, by process of elimination, (D) is the “best of the worst.” Thus, choice (D) is the best answer, even though the nephew’s fee terminated at his wife’s death. Choice (A) is incorrect because once the wife died without issue, the chefs shifting executory interest “vested” and the nephew’s fee was cut off, thus altering the nature of the nephew’s estate and reducing his rights to drill for oil. Because the wife’s death does improve the chefs chances, this choice is incorrect. Choice (B) is incorrect because once the wife died without issue, the chefs shifting executory interest “vested” and the nephew’s fee was cut off, thus altering the nature of the nephew’s estate and reducing his rights to drill for oil. Because the wife’s death does improve the chef’s chances, this choice is incorrect. Choice (C) is wrong because the nephew’s fee does not “revert” back to the grantor, the investor, but shifts to the benefit of the grantee, the chef.
83
Q
  1. A landowner owned a 10-acre tract of land, in fee simple. The tract was located in a small town. In 1974, the landowner conveyed the tract to the town development association “on condition that a ski lodge and resort area be built thereon and the land be used solely for skiing purposes and, in the event that said property is not used as a ski resort, the property shall revert to myself, the grantor, my heirs or assigns.” Subsequently, a ski lodge was built and the land was continuously used as a ski resort for more than 20 years.In 1979, a neighbor, owner of a farm abutting the northeast corner of the 10-acre tract, began to use a portion of the 10-acre tract for ingress and egress to his farm. He used this access road openly, visibly, and notoriously until 2000. The period of prescription in this particular jurisdiction was 20 years. In March 2000, the neighbor decided to sell the farm to a skier. Thereupon, the neighbor and the skier entered into a written real estate sales contract, with closing set for May 1, 2000.On April 1, 2000, the president of the town development association was approached by the skier with the request that a written easement be given over the used area and that an appropriate instrument be delivered to the skier concurrently with the skier’s closing with the neighbor. Following the president’s meeting with the skier, the town development association held a duly announced meeting and voted unanimously to authorize its trustees to grant such an easement.On April 15, 2000, the landowner died. The landowner’s sole descendant and successor in interest was his wife. Upon her husband’s death, the wife immediately notified the town development association that if it granted the easement to the skier, the 10-acre tract would revert to her. The skier then brought an appropriate action, joining the neighbor, the wife, and the town development association, seeking a declaratory judgment that the neighbor has a perpetual easement appurtenant to the 10-acre tract.In this action, judgment should be in favor of which of the following parties?(A) The wife, because the skier could not obtain rights against the town development association by prescription.(B) The wife, because the easement would violate the negative restriction that the landowner originally sought to impose.(C) The skier, because the neighbor had already obtained an easement by prescription.(D) The skier, because the language of the conveyance to the town development association was too vague to support any claim by the wife.
A
  1. (C) An easement by prescription arises by adverse use of the servient tenement by the dominant tenant for the period of the statute of limitations. To mature such an easement against a landowner, the use must be (1) adverse as distinct from permissive, in derogation of right rather than in subordination to the rights of the landowner;(2) open and notorious; (3) continuous and without interruption; and (4) for the period of prescription. See Smith and Boyer, Law of Property, p. 387. Because the neighbor openly and notoriously used the northeast portion of the 10-acre tract to gain access to the farm for the 20-year period as required by statute, he obtained an easement by prescription. This easement passed on to the skier, who was in privity with the neighbor by means of the 1980 sales contract. Therefore, the skier’s rights are enforceable against the whole world and he will prevail against the wife. Choice (C) is the correct answer. Choice (A) is incorrect. The skier could not obtain rights against the town development association by prescription because he had just purchased the property from the neighbor. However, because the neighbor had already acquired the easement by prescription, he was able to pass it to the skier. So, the skier can use the easement and will prevail against the wife even though the skier himself was not able to satisfy all the elements of an easement by prescription. Choice (B) is incorrect. The neighbor satisfied all of the elements required to obtain an easement by prescription. Whether or not this violates the negative restriction that the Landowner initially put into the words of grant, the fact remains that this easement now exists and must be given to the neighbor and now to the skier as the successor. Thus, the wife’s argument in choice (B) would be of no help to her, especially when the skier will prevail because the neighbor met all of the required elements. Choice (0) is incorrect. The skier will prevail because the neighbor had acquired the easement by prescription before the neighbor passed it on to the skier. That is all that the skier needs to prove in order to prevail against the wife. Interpreting the language of the conveyance is not necessary and not relevant because the skier can simply rely on the neighbor’s obtaining a prescriptive easement.
84
Q
  1. A vintner owned a vineyard in fee simple absolute. The deed by which the vineyard was conveyed to her was properly recorded. The vintner sold the vineyard to a husband and wife as tenants in common, who did not record the warranty deed by which the vintner conveyed the vineyard to them. The husband subsequently sold and conveyed his undivided one-half interest in the vineyard to a friend. The friend promptly recorded his deed. Some time later, the vintner purported to convey the vineyard to a farmer, for valuable consideration. The farmer, who had no actual knowledge of any previous conveyances of the vineyard, recorded her deed. The jurisdiction maintains alphabetical grantor and grantee indices. The applicable recording statute provides, “Every conveyance of real property is void as against any subsequent purchaser of the same property (or any part thereof) in good faith and for a valuable consideration, whose conveyance is first duly recorded.” None of the various grantees of the vineyard has entered into possession in a manner that would put anyone on inquiry notice as to their interest therein.Who owns the vineyard?(A) The friend, because he recorded his deed before the farmer recorded.(B) The friend and the wife, because the friend’s deed would put the farmer on inquiry notice as to the wife’s interest in the vineyard.(C) The farmer, because the husband—friend deed is outside the farmer’s chain of title.(D) The friend and the farmer, as tenants in common.
A
  1. (C) In a jurisdiction that maintains only grantor and grantee indices, a deed such as the one the husband used to convey to the friend is outside the chain of title to a property such as the vineyard. The normal title search involves an examination of the grantee index from the present backwards until the seller of the property is found. Then the Grantor Index is examined from that date forward, to insure that the seller has not previously conveyed the property. Here, no deed of record would appear showing that the vintner had ever conveyed the vineyard, because the vintner—husband/wife deed was never recorded. The farmer would have no reason or information to search the Grantor Index for the husband’s name or the grantee index for the friend’s name, because the farmer would never have heard of either of them. Thus, the husband—friend deed, although recorded first, is outside of the farmer’s chain of title and is treated as though never recorded as to her. Because she is a subsequent purchaser for value who has no actual, inquiry, or constructive notice, and recorded first (because the husband—friend deed is ignored), the farmer’s interest voids both wife’s and the friend’s interest in the vineyard. Choice (A) is incorrect. Because the friend’s deed is outside the chain of title to the vineyard, the friend gains nothing by recording his deed first. He is treated as though he had never recorded and thus the farmer, who purchased subsequently for value and duly recorded, takes a superior interest in the vineyard. The friend should have done a title search and learned of the non-recordation of the vintner—husband deed, and cause that deed to be recorded, so that anyone subsequently purporting to purchase the vineyard from the vintner would find the vintner in the Grantor Index. Choice (B) is incorrect. The friend’s deed imparts no notice, constructive or otherwise, to the farmer because it is outside the farmer’s chain of title; she could not reasonably be expected to search for grantors and grantees of whom she has no knowledge. The farmer’s proper title search would reveal that the vintner had not conveyed the vineyard to anyone because she (the vintner) had acquired that property, and thus the farmer would proceed with the purchase, in good faith. Of the three (the husband, the friend, and the farmer), only the farmer is unable to correct the record. Thus, she is entitled to the protection of the recording act. Choice (D) is misleading because it suggests that the friend’s recordation protects him, while the wife, whose interest was never recorded, loses out to the farmer under the recording act. The friend is no more protected by his recordation than is the wife, who never recorded.
85
Q
  1. In 1940, a state conveyed to a man a tract of land by deed that was immediately recorded. The man died a widower in 1990, devising all of his real property to his son. The will was duly probated shortly thereafter.In 2005, the son conveyed the tract of land to a friend. This deed was not recorded until after the son’s death in 2009. The son’s will, duly probated following his death, named the friend as sole legatee and devisee of the son’s entire estate.In 2008, the son mortgaged the tract of land to the bank. The mortgage instrument, which was recorded in 2008, recited that it was subordinate to a mortgage on the same land given by the son to a rancher in 1988 and recorded in 1988. In that instrument, the son purported to grant the rancher a mortgage on the tract of land.All of the deeds mentioned in the aforementioned transactions are general warranty deeds. In addition, the State of Baden has a notice-type recording statute and follows a title theory for mortgages.What is the probable legal effect of the son’s mortgage agreement in 2008 with the bank?(A) The mortgage would be invalid, because the son conveyed his interest in the property to the friend in 2005.(B) The mortgage would be invalid, because the first mortgage in 1988 would take precedence over the second mortgage.(C) The mortgage would be invalid, unless the mortgagee bank had knowledge of the prior conveyance to the friend.(D) The subsequent mortgagee, the bank, would prevail as against the prior conveyee (the friend), who failed to record before the mortgage was effectuated.
A
  1. (D) Under notice-type recording statutes, an unrecorded conveyance or other instrumentis invalid as against a subsequent bona fide purchaser (creditor or mortgagee) for value and without notice. Under this type of statute, the subsequent bona fide purchaser prevails over the prior interest whether the subsequent purchaser records or not. Insofar as the subsequent purchaser is concerned, there is no premium on his race to the recorder’s office; his priority is determined upon his status at the time he acquires his deed or mortgage. Choice (A) is incorrect. Even though the son conveyed his interest to the friend in 2005, that deed was not recorded until 2009, which was after the bank recorded its mortgage. At the time that the son gave a mortgage to the bank, there was no constructive notice of the friend’s interest in the property. Because this is a notice jurisdiction, the bankwould prevail as against the friend who failed to record before the mortgage to the bank was effectuated. Thus, without a timely recordation, it is irrelevant that the son conveyed to the friend in 2005. Choice (B) is incorrect. The existence of a first mortgage would never render a second mortgage invalid; the second mortgage would simply be subordinated to the first. Because this choice falsely suggests that a second mortgage is invalid, it is incorrect. Choice (C) is incorrect. This choice states that the only way that the mortgage would be valid is if the bank had knowledge of the prior conveyance to the friend. This would actually be a reason why the bank’s mortgage would be invalid. If the bank were to have known that the property had already been conveyed to the friend, the bank would be put on notice that there was no property left to place a mortgage on. Because this choice improperly states that knowledge would allow the mortgage to be valid, it is incorrect.
86
Q
  1. A farmer owned a tract of land in a state that has a notice recording statute and follows the title theory of mortgages. There were two farms situated on the property, one farm on the eastern half and one farm on the western half of the property. The farmer recorded a general warranty deed conveying the property to his son.Two years later, the son obtained a mortgage, secured by the farm property, in favor of his friend. The friend promptly recorded the mortgage. Twenty years later, the son sold the property to an investor by a general warranty deed, with the investor assuming the friend’s mortgage. The following year, the investor subdivided the property and sold the western farm to a developer, while retaining the eastern farm. There was no mention of the mortgage in the general warranty deed between the investor and developer.The developer was unable to rezone the property to build houses as he had planned, so the developer sold the western farm to a florist. The conveyance to the florist made no mention of the friend’s mortgage. Three years later, the florist entered into a contract with a rancher, pursuant to which the rancher would purchase the western farm for $75,000. The closing date for the deal was set for January 15.Assuming that on January 10, the rancher conducted a title search that revealed the friend’s mortgage on the property, which of the following is correct?(A) The rancher would be entitled to rescind the real estate contract with the florist immediately.(B) The encumbrance renders title to the western farm unmarketable.(C) The mortgage would only encumber the eastern farm, not the western farm, because the investor’s partition of the property created a joint tenancy.(D) The encumbrance would not entitle the rancher to rescind the real estate contract until closing on January 15.
A
  1. (D) There can be no rescission of an executory contract for the sale of land merely because of lack of marketable title (e.g., existence of an encumbrance) in the seller prior to the date when performance is due (on the “closing” date). Neither can a buyer place the seller in default by tendering payment and demanding a deed in advance of the time and under circumstances not contemplated by the contract. Because the seller has until the closing date to remedy the defect in title (i.e., the mortgage), there is no right to rescind until that date. Choice (A) is incorrect. The seller is only obligated to tender a “good and marketable” title on the date when the conveyance is to be executed (i.e., “closing” date), and a purchaser may not rescind a land sale contract before the time for performance. For this reason, the buyer may not rescind immediately and must wait until the closing date to see if the defect had been remedied. Choice (B) is incorrect. This is a true statement standing alone; a mortgage on the property does render the title to the western farm unmarketable. However, the seller has until the closing date to remedy this defect. Usually, the proceeds from the sale will be sufficient to satisfy a prior mortgage and the seller has until the closing date to make the title marketable. Because this answer choice fails to take into consideration the fact that the seller has time to remedy this defect (and thus make title marketable) it is incomplete. Choice (C) is incorrect. No joint tenancy was created here because the investor sold one entire portion of the property to the developer and kept the remaining portion for himself. Because there was never any intent to create any type of concurrent ownership in the same piece of property, this statement is incorrect on the facts.
87
Q
  1. In 1888, a landowner owned a dairy farm. The landowner conveyed this real property to his son in1938.In 1953, the son conveyed the dairy farm to his friend. This deed was not recorded until after the son’s death in 1957.In 1956, the son mortgaged the dairy farm to the bank. The mortgage instrument, which was recorded in 1956, recited that it was subordinate to a mortgage on the same land given by the son to an investor in 1936 and recorded in 1936. In that instrument the son purported to grant the investor a mortgage on the dairy farm.In 1979, the friend conveyed the dairy farm to a farmer. This deed was duly recorded, but did not mention any mortgage.In 2008, a buyer entered into an agreement with the farmer, whereby the farmer would convey the dairy farm in fee simple to the buyer for the sum of $75,000. The closing date was set for January 15, 2009.All of the deeds mentioned in the aforementioned transactions are general warranty deeds. In addition, this jurisdiction has a notice-type recording statute and follows a title theory for mortgages.On January 15, 2009, the sale of the dairy farm is finalized and the buyer paid the farmer $75,000. The fanner executed a general warranty deed. The deed contains the following covenants of title:(1) Covenant for seisin.(2) Covenant of the right to convey.(3) Covenant against encumbrances.After the buyer takes possession of the dairy farm, he learns of the son—investor 1936 mortgage, which was not satisfied, and seeks monetary damages for breach of the covenant against encumbrances. Judgment should be for(A) the buyer, because the covenant against encumbrances is a guarantee to the grantee that the property is not subject to outstanding rights or interests.(B) the buyer, because the covenant against encumbrances would be breached at the time the deed was delivered, thereby entitling the covenantee to recover damages.(C) the farmer, because the covenant against encumbrances may only be breached, if at all, at the time of conveyance.(D) the farmer, unless the covenantee is disturbed in his actual enjoyment of the land thereby conveyed.
A
  1. (B) Here, it is clear that the farmer’s covenant against encumbrances was breached the very instant he conveyed the dairy farm to the buyer because the encumbrance of the 1936 mortgage burdened the dairy farm at that time. Even if the mortgagee never forecloses or threatens to foreclose and the subsequent purchaser is never called upon to pay off the encumbrance, there is, nevertheless, a breach of covenant, and the covenantee may recover damages. Choice (A) is incorrect. The covenant against encumbrances is a covenant by the grantor that the estate he will convey is free of encumbrances. All this choice does is define what the covenant is without explaining why the buyer is entitled to damages because this choice does not explain that the covenant was breached nor does this choice explain when it was breached. Because choice (B) states that the covenant was breached at the time the deed was delivered, (B) does a better job of explaining that the buyer is entitled to damages because the deed was already delivered. Because choice (A) fails to explain why the buyer is now entitled to damages, it is not as complete as (B). Choice (C) is incorrect. This is a true statement; the covenant against encumbrances is breached at the time of conveyance and it was, in fact, breached in this situation. However, because this choice states that the farmer (the seller) would prevail, it is incorrect. Choice (D) is incorrect. This choice suggests that the only way that the buyer would prevail is if he were disturbed in his actual enjoyment of the land just to recover for a breach of this covenant. Because the covenant is breached at the time of the conveyance, it is not required that the covenantee be disturbed in his actual enjoyment.
88
Q
  1. A teacher is the owner in fee simple of a seven-acre quarry, on which he maintains a dwelling house for himself and his family. Adjoining the quarry to the west is a 10-acre ranch, owned by a librarian. In order to gain access to the highway, the teacher has been granted an easement to cross over the ranch using the road along the ranch’s southernmost boundary.The teacher has recently purchased a 12-acre farm, which abuts the ranch, but is not appurtenant to the quarry. Directly north of the farm is the ranch and directly west is the highway. The teacher has begun constructing a farmhouse on the farm and is using the existing easement (across the ranch) to gain access to the 1 2acre farm. The teacher has never received permission from the librarian to use the road across the ranch to gain access to the farm.In an appropriate action by the librarian to enjoin the teacher from using the existing easement to gain access to the farm, the plaintiff will most likely(A) succeed, because the teacher is making use of the servient tenement beyond the scope and extent of the easement as it was originally created.(B) succeed, because the teacher has no right to use the servient tenement in connection with a tract of land that is not part of the dominant tenement.(C) not succeed, because the teacher has an easement by necessity.(D) not succeed, because the teacher has a right to use the easement in a manner not inconsistent with the rights of the owner of the servient tenement.
A
  1. (B) According to Smith and Boyer, every easement appurtenant requires two pieces of land that are owned by two different persons. The two pieces of Land involved are (1) the dominanttenement, which is the land whose owner is benefited by the easement; and (2) the servient tenement, which is the Land whose owner is burdened by the easement. When an easement appurtenant is created (either by conveyance or prescription) over a servient tenement, the boundaries and extent of such easement become fixed and are binding on both the servient and the dominant tenants. Neither tenant has the right to change the location of such easement or change the boundaries of the servient or dominant tenement(s). In this example, choice (B) is best because the teacher has an easement appurtenant between the quarry (the dominant tenement) and the ranch (the servient tenement). He is attempting to use the ranch to go towards the south to access the farm, another tract of land that is not part of the dominant tenement. Choice (A) is incorrect. If the teacher attempted to make excessive use of the ranch in relationship to the quarry, then choice (A) would be correct. For example, if the teacher had a right-of-way for crossing the ranch (from the quarry) on foot or by bicycle, then it would be beyond the scope of the easement for him to cross the ranch (from the quarry) by automobile. This, however, is not the situation. Therefore, choice (B) is the better answer here. Choice (C) is incorrect. The facts state that the easement was given by a grant. The teacher does not have an easement by necessity. Choice (D) is incorrect. While it may be true that the teacher has the right to use the easement to get to the highway in a manner that the librarian could do also, that does not include the right to use the easement in connection with a tract of land not part of the original dominant tenement. So, while this statement is true in the sense that both the servient and dominant tenant can use the easement, it disregards the fact that the teacher began to use the easement to access a new parcel of land not originally envisioned by the parties.
89
Q
  1. A professor, being fee simple owner of an apartment building, made a conveyance thereof in these words, “I hereby convey my apartment building to my son and daughter as tenants in common.” In the deed instrument, which was properly executed, the professor reserved a life estate to himself. The professor then told his children of his conveyance and put it in the family vault in the library for safekeeping. Thereafter, the son married a doctor. The professor, who disliked the doctor, then executed another deed that he labeled “a correction deed.” In the “correction deed,” the professor conveyed the apartment building “to my son and daughter as joint tenants with right of survivorship.” According to the new deed, the professor again reserved a life estate to himself. Both children accepted delivery of the “correction deed.” Six months later, the son died intestate, leaving the doctor as his sole heir.Title to the apartment building is in(A) the daughter and the doctor as tenants in common.(B) the daughter subject to the professor’s life estate.(C) the daughter and the doctor as tenants in common, subject to the professor’s life estate.(D) the daughter and the doctor as joint tenants with survivorship rights, subject to the professor’s life estate.
A
  1. (C) An owner of real property can convey title thereto by deed by doing two things: (1) making a deed, and (2) making delivery thereof to the intended grantee(s). In this case, the professor did make out the deed. In other words, delivery requires that the grantor relinquish all control over the instrument as an effective transfer of title. In their handbook on Real Property, Smith and Boyer point out that delivery “does not mean giving up control over the mere physical piece of paper on which the writing or printing appears.” On the other hand, such intention is in the mind of the grantor and is usually a question of fact. In the present example, the professor appears to have intended that the original deed operate or take effect as a conveyance. Once a deed has been delivered, a cancellation of that instrument will not revest ownership. The professor could not unilaterally re-deed the property to his children once the first deed had been delivered because the professor had nothing left to convey. The most that the professor could have conveyed was the life estate that he had retained for himself. Therefore, the “correction deed” has no legal effect; thus, the daughter and the doctor take title to the apartment building as tenants in common (following the son’s death) subject to the professor’s life estate. Choice (A) is incorrect. The daughter and the doctor will, in fact, take as tenants in common, but they must wait until the termination of the professor’s life estate. Because choice (C) takes the life estate into account, it is more complete than (A). Choice (B) is incorrect. This would be true if the daughter and the son did, in fact, have a joint tenancy with right of survivorship. However, because the “correction deed” is void, the daughter and the son continued to have a tenancy in common. So, when the son died, there was no automatic vesting in the daughter under a right of survivorship theory. Therefore, the daughter and the doctor will take title as tenants in common subject to the professor’s life estate. Choice (D) is incorrect. Because the “correction deed” is void, the original deed will be operative. Because that original deed stated that there would be a tenancy in common, that is the form of concurrent ownership that will exist after the professor’s life estate.
90
Q
  1. An investor was the record owner in fee simple absolute of a 125-acre parcel of land located in the Southwest. The investor conveyed the property “to my assistant for life with remainder to my son.” At the time of the conveyance, the son was in medical school outside the United States. The assistant immediately recorded the deed and took possession.Shortly thereafter, the assistant discovered that the property contained large coal deposits, which she mined and began to sell. Three years after the original conveyance, the son graduated from medical school and returned to the Southwest. He then learned of the conveyance and also ascertained that the assistant had not paid taxes on the parcel of land for the last three years. After discovering that the property was subject to a pending tax foreclosure, the son demanded that the assistant pay the delinquent taxes. Even though the profits from the coal sales were quite substantial, the assistant refused to pay the outstanding taxes. The son thus paid the taxes himself.If the son sues the assistant to recover the taxes and for an accounting of the proceeds received from the coal sales, judgment should be(A) in favor of the son for the taxes, but not for the coal.(B) in favor of the son for the coal, but not for the taxes.(C) in favor of the son for both the taxes and the coal.(D) against the son for both the taxes and the coal.
A
  1. (C) Before attempting to answer this question, students must first determine what interest, if any, the son has in the parcel of land. As we know, the investor (the grantor) conveyed the property “to the assistant for life with remainder to my son.” Does the son take a vested or contingent remainder under this conveyance? The answer is that the son takes a vested remainder. A vested remainder is one that is limited to an ascertained person who has the right to immediate possession if and when the prior estate is terminated. It may be subject to no other condition, in which case it is said to be indefeasibly or absolutely vested. In that case, it is certain to become an estate in possession. In our case, the son has a vested remainder. Note that a tenant for life or for years is liable to the (vested) remainderman or reversioner for voluntary waste; therefore, the assistant is liable to the son for an accounting of the proceeds from the coal sales. In addition, a vested remainderman has a right to compel the prior estate owner to pay taxes and interest on encumbrances to the extent of the value of rents and profits. Consequently, choice (C) is correct. Choice(A) is incorrect. A tenant for life is liable to the (vested) remainderman or reversioner for voluntary waste. Therefore, the assistant is liable to the son for an accounting of the proceeds from the coal sales. In addition, a vested remainderman has a right to compel the prior estate owner to pay taxes and interest on encumbrances to the extent of the value of rents and profits. Because this choice only allows the son to recover taxes, it is incorrect. Choice (B) is incorrect. A tenant for life is liable to the (vested) remainderman or reversioner for voluntary waste. Therefore, the assistant is liable to the son for an accounting of the proceeds from the coal sales. In addition, a vested remainderman has a right to compel the prior estate owner to pay taxes and interest on encumbrances to the extent of the value of rents and profits. Because this choice only allows the son to recover for the coal mining, it is incorrect. Choice (D) is incorrect. A tenant for life is liable to the (vested) remainderman or reversioner for voluntary waste. Therefore, the assistant is liable to the son for an accounting of the proceeds from the coal sales. In addition, a vested remainderman has a right to compel the prior estate owner to pay taxes and interest on encumbrances to the extent of the value of rents and profits. Because this choice does not allow the son to recover anything, it is incorrect.
91
Q
  1. A developer is the owner of a parcel of land in fee simple absolute (the record title is also in the developer). The parcel of land is and has been unpossessed land. Assume that each person listed below as the grantee in a deed paid value and took without actual notice of any facts that would defeat her title and without knowledge of any facts that would put her on inquiry, both at the time she took her deed and at the time she recorded it.The following conveyances, each by a general warranty deed, take place in the order listed: An artist conveys to a bartender; the developer conveys to the artist; the artist conveys to a counselor; the bartender conveys to a dentist; the counselor conveys to an engineer.No deed was recorded until after the artist-to- counselor deed was executed and then before the execution of any other deed, the order of recording was as follows: developer to artist; artist to bartender; artist to counselor.The bartender-to-dentist deed and counselor-to- engineer deed were each recorded immediately after their respective executions.The owner of the parcel of land in a notice jurisdiction is(A) the bartender.(B) the counselor.(C) the dentist.(D) the engineer.
A
  1. (D) A notice-recording act protects a subsequent bona fide purchaser (BFP), whether or not she was the first to record. In other words, a subsequent BFP is protected if the grantee of an earlier deed has not recorded at the time the subsequent BFP takes title, regardless of who records first. Thus, the last BFP without notice will prevail under the notice system. In this question, the crucial determination is not the order of recording, but the fact that the counselor took title from the artist without notice of the artist-bartender deed (which was still unrecorded at the time that the counselor took). As a BFP, the counselor will prevail over the bartender, so the counselor’s grantee (the engineer) will prevail over the bartender’s grantee (the dentist). Choice (A) is incorrect. The bartender will not prevail for two reasons: first, the bartender failed to record, and that failure meant that the counselor would take without notice of the artist’s prior conveyance to the bartender. So, at the very (east, the bartender would lose as against the counselor. Second, the bartender has already sold the property for value to the dentist, so the bartender is not the owner. Choice (B) is incorrect because the counselor already sold the property for value to the engineer, so she would not be able to claim ownership in the parcel of land. Choice (C) is incorrect. Because the counselor prevails over the bartender, the counselor’s grantee (the engineer) would then prevail over the bartender’s grantee (the dentist). In this contest between the dentist and engineer, the engineer will prevail.
92
Q
  1. A veterinarian was the owner of 1,500 acres of undeveloped timberland.The land in question had never been occupied, fenced, or cleared except that for 24 years, a mining company, one mile from the property, regularly drove trucks over a cleared path pursuant to an agreement with the veterinarian. The agreement, which was duly recorded, provided that “the parties expressly agree and the veterinarian promises that the veterinarian and his successors shall refrain from obstructing the said described pathway across the veterinarian’s land, which the mining company and its successors may perpetually use as a road, in consideration of which the mining company and its successors will pay the sum of $700 per annum.”The provision in the agreement between the veterinarian and the mining company granting “the use of the pathway” to the mining company may best be described as a (an)(A) license.(B) easement appurtenant.(C) easement in gross.(D) prescriptive easement.
A
  1. (C) An easement is deemed “in gross” when the right of special use benefits the owner or possessor personally rather than in connection with any land the holder owns. In other words, every easement in gross requires only one piece of land (i.e., the servient tenement) that is owned by a person other than the owner of the easement in gross. There is no dominant tenement. The servient tenement is the land subject to and burdened by the easement. The veterinarian granted the mining company an express easement to use the roadway to reach other parts of the timberland, thus giving the mining company an easement in gross. Choice (A) is incorrect because a license is a revocable privilege to enter upon the lands of the licensor. Choice (B) is wrong because an easement appurtenant benefits the holder of the easement in his/her physical use or enjoyment of another tract of land. For an easement appurtenant to exist, there must be two tracts of land: the dominant tenement and the servient tenement. There is no dominant tenement here that is enhanced by the granting of the easement. Because there is only a servient tenement, there is no easement appurtenant. Choice (D) is erroneous because a prescriptive easement is not in writing, but arises through the long continued use of the property (as analogous to adverse possession). Because there is an express writing that granted the easement, it is an express easement, not a prescriptive easement.
93
Q
  1. A doctor was the owner of 1,500 acres of undeveloped timberland. In September 1989, the doctor executed a warranty deed conveying the timberland property to a dentist in fee simple. The dentist recorded immediately in the Grantor—Grantee Index. Then in April 1990, the dentist conveyed the same tract to a buyer in fee simple by warranty deed. The buyer paid full market value and recorded the deed at once in the Grantor—Grantee Index.The land in question had never been occupied, fenced, or cleared except that between the years 1986—2010, a mining company, one mile from the property, regularly drove trucks over a cleared path pursuant to a 1986 agreement with the doctor. The agreement, which was duly recorded, provided that “the parties expressly agree and the doctor promises that the doctor and his successors shall refrain from obstructing the said described pathway across the doctor’s land, which the mining company and its successors may perpetually use as a road, in consideration of which the mining company and its successors will pay the sum of $700 per annum.”In 1990, after the conveyance from the dentist, the buyer informed the mining company that he would no longer honor the 1986 agreement permitting the mining company to use the pathway.The mining company brought an action for specific performance. Judgment should be for(A) the mining company, because their property interest would “run with the land.”(B) the mining company, because the possessor of a servient interest would prevail against subsequent owners.(C) the buyer, because the mining company’s interest was extinguished by the subsequent conveyance.(D) the buyer, because there was no privity of estate between the buyer and the mining company.
A
  1. (A) There was an intent in the 1986 agreement that the easement in gross granted to the mining company would “run with the land” and thus be enforceable against successors of the original parties. As a consequence, the fact that the mining company has continued to use the easement is sufficient to establish the right to use the servient tenement in manner consistent with the easement right. Choice (B) is incorrect on the facts. In this easement in gross situation, the buyer is the owner of the servient interest. Choice (C) is incorrect. There was an intent in the 1986 agreement that the easement in gross granted to the mining company would “run with the land” and thus be enforceable against successors of the original parties. Because of this intent, the mining company’s interest was not extinguished simply because the dentist conveyed the servient tenement to the buyer. Choice (D) is incorrect. Certainly there is no privity of estate between the buyer and the mining company because neither party conveyed any interest in property to the other. However, that does not mean that the easement in gross does not still exist. Because the easement was intended to “run with the land” it is irrelevant that there is no privity of estate between these two parties. It is enough that the buyer is a successor to the dentist, who was one of the original covenanting parties.
94
Q
  1. A farmer conveyed 100 acres of his farm to a landscaper. The deed contained the following covenants: (1) seisin, (2) right to convey, and (3) against encumbrances. Subsequently, the landscaper conveyed the property to a buyer by warranty deed. However, the buyer is later evicted by a woman because of paramount title.The buyer now brings suit against the farmer for breach of covenants in the deed. Judgment should be for(A) the buyer, because the covenants contained inthe deed run with the land.(B) the farmer, because no privity of estate existsbetween the buyer and the farmer.(C) the buyer, but only for the covenants of seisinand right to convey.(D) the farmer, because the covenants are personalin nature and do not run with the land.
A
  1. (D) The covenants of seisin, right to convey, and against encumbrances are personal covenants which, from their nature, must be broken instantaneously on the delivery of the deed or they are never broken. It follows that such personal covenants never run with the land and, therefore, the subsequent grantee may not maintain a cause of action against the original grantor. Choice (A) is incorrect. The covenant of seisin, the covenant of the right to convey, and the covenant against encumbrances are not continuous and do not run with the land. These three present covenants are breached at the time of conveyance if they are breached at all. The covenant of warranty, the covenant of quiet enjoyment, and the covenant of further assurances are continuous in nature and may be enforced by remote parties on the basis of privity of estate. Choice (B) is incorrect. The covenant of warranty, the covenant of quiet enjoyment, and the covenant of further assurances are continuous in nature and may be enforced by remote parties on the basis of privity of estate. In other words, privity of estate does exist between remote parties. Because this choice falsely states that there is no privity between the farmer and the buyer, it is incorrect. Choice (C) is incorrect. The covenants of seisin, right to convey and against encumbrances are personal covenants which, from their nature, must be broken instantaneously on the delivery of the deed or they are never broken. It follows that such personal covenants never run with the land and, therefore, the subsequent grantee may not maintain a cause of action against the original grantor. The buyer may not enforce any of those three covenants against the farmer.
95
Q
  1. A landowner conveyed 50 acres of his orchard to a fanner. The deed contained the following covenants:(1) seisin, (2) right to convey, and (3) against encumbrances.When the landowner conveyed the property to the farmer, there was an outstanding mortgage. The farmer took possession, and shortly thereafter, threatened with foreclosure, he paid off the mortgage with interest.The farmer now sues for breach of covenant against encumbrances. The court will most likely allow(A) recovery for the amount in principal and ininterest thereon from the time of the mortgagepayment.(B) recovery for only the principal that the farmerpaid on the mortgage.(C) recovery for the measure of value betweenthe value of the land with and without such anencumbrance.(D) no recovery.
A
  1. (A) Because the covenant against encumbrances (i.e., existence of mortgage) was breached at the instant of the delivery of the deed, the farmer will succeed in his action because the encumbrance of the mortgage burdened the property at the time of conveyance, entitling him to recover the amount he was compelled to pay in principal and in interest with interest accruing from time of payment. Choice (B) is incorrect. Generally, the covenanting party (in this case, the landowner) is liable for all amounts paid by the covenantee (the farmer) so long as the covenantor is not being made to pay more than the price paid by the covenantee. There is no reason why the farmer would be restricted to recovering the principal only. The farmer will be allowed to collect interest as well for being made to pay this mortgage that became an encumbrance on title. Choice (C) is incorrect. This type of encumbrance has a specific dollar value that can be calculated with certainty. Diminution in the value of the land is more appropriate for encumbrances that reduce the value of the land such as easements and other restrictive covenants whose dollar value is harder to measure and can really only be quantified by how much they devalue the property. A mortgage is not really of the same type of encumbrance as an easement because it has dollar limits. In fact, we know exactly how much the farmer paid because the facts tell us he paid off the mortgage. Those payments that the farmer made will be recoverable, and the farmer does not need to rely on diminution in value as his basis of recovery. Choice (D) is incorrect. Because this covenant was breached, damages will be allowed as stated above.
96
Q
  1. In 1993, a rancher had good record title to a 20-acre orange grove in fee simple absolute. In 1994, the rancher delivered to his son, for a sum of $1,000, a deed signed by the rancher, naming the son and his heirs as grantee, and appearing valid on its face. The son neglected to record the deed.In 1998, a farmer, aware of the existence of the rancher-to-son deed, sought out the rancher and asked to buy for $10,000 a deed to the orange grove from the rancher to the fanner and his heirs. The rancher executed such a deed, and the fanner promptly recorded it. The farmer’s intent was to acquire color of title and obtain ownership of the orange grove by adverse possession. In 1998, the farmer constructed a fence around the orange grove.In 1999, the son presented his deed of the orange grove to a retiree, and for $15,000, paid by the retiree, signed and delivered a deed of the orange grove in favor of the retiree and his heirs. After receiving the deed, the retiree made no effort to search the title, to examine the property, or to record the deed.In 2003, a buyer paid the fanner $20,000, and the farmer delivered to the buyer a deed of the orange grove in favor of the buyer and his heirs. The buyer had examined the property, had searched the title, and had no knowledge of the farmer’s awareness of the prior rancher-to-son instrument. Although the buyer did not reside on the property, he regularly visited the orange grove twice a week. The buyer recorded his deed.In 2007 for $25,000 paid by an orange grower, the retiree signed and delivered adeed of the orange grove naming the orange grower and his heirs as grantees.Before the grower had paid the retiree and taken his deed, the grower visited the orange grove and observed the fence. However, the buyer was not present when the grower visited the property and nothing suggested who—if anyone—was using it. In any case, the grower did not attempt to search the title before making his purchase. This jurisdiction uses Grantor—Grantee Indices and has no Tract Index.In 2008, what is the present state of title to the orange grove if the jurisdiction’s recording act provides: “Every conveyance of real property shall be invalid as against any person, except the grantor, his heirs, and assigns, having actual notice thereof, unless it is recorded as provided by statute”?(A) In a notice jurisdiction, the grower, as a subsequent bonafide purchaser, is only chargeable with notice of what appears in his chain of title and, therefore, would acquire record title to the orange grove.(B) In a race—notice jurisdiction, the farmer would acquire equitable title to the orange grove, because he erected the fence and failed to inform the buyer of the prior rancher-to-son instrument.(C) In a race—notice jurisdiction, the buyer, as subsequent bonafide purchaser without notice of the prior rancher-to-son instrument, would acquire record title to the orange grove.(D) In a notice jurisdiction, the retiree would acquire record title to the orange grove even though he failed to record his deed.
A
  1. (A) The aforementioned recording act is a pure notice type statute. Under this type of statute, the subsequent bona fide purchaser prevails over any prior interests— whether the subsequent purchaser records or not. The subsequent purchaser’s priority is determined by his status at the time he acquires his deed; there is no premium on his race to the recorder’s office. The orange grower has a superior right in the property because he took with no notice of any prior conveyances of the same property. Even though the farmer—buyer deed had been recorded first, the grower would not have been able to see that this is part of his chain of title even if the grower saw that deed in the index. This is because the grower would not be able to tell that this deed had any connection to his own chain of title. Also, even though the rancher—farmer deed had been recorded, the grower would not be able to tell that this deed had any relationship to his own chain of title. The grower bought from the retiree, but because the retiree never recorded the son—retiree deed, there was no record of the retiree. Thus, there was virtually no chain of title for the grower to search. Because of this, the grower could believe in good faith that this property had not been granted to others. This makes the grower a subsequent bona fide purchaser forvalue who established his priority as soon as he purchased the property. Choice (B) is incorrect. Based on the statute, this is not a race-notice jurisdiction. Moreover, the farmer could not prevail in any jurisdiction with a notice component to the statute because he will not qualify for bona fide purchaser (BFP) status because he knew about the prior sale of the orange grove to the son. Choice (C) is incorrect. Based on the statute, this is not a race-notice jurisdiction. Even though the buyer had no knowledge of the prior grant from the rancher to the son, the buyer still loses to the grower who was a subsequent purchaser who took without notice. Because the farmer—buyer deed was not in the grower’s chain of title, the grower can still take in good faith because he had no notice of prior grants of the same property. Choice CD) is incorrect. The retiree sold the property to the grower, so the retiree is not in any contest for ownership of the orange grove.
97
Q
  1. In 1993, a landowner had good record title toa vineyard in fee simple absolute. In 1994, thelandowner delivered to his son, for a sum of $1,000,a deed signed by the landowner, naming the son andhis heirs as grantee, and appearing valid on its face.The son neglected to record the deed.In 1998, a farmer, aware of the existence of the landowner-to-son deed, sought out the landowner and asked to buy for $10,000 a deed of the vineyard from the landowner to the farmer and his heirs. The landowner executed such a deed, and the farmer promptly recorded it. The farmer’s intent was to acquire color of title and obtain ownership of the vineyard by adverse possession. In 1998, the farmer constructed a fence around the vineyard.In 1999, the son presented his deed of the vineyard to an investor and, for $15,000, paid by the investor, signed and delivered a deed of the vineyard in favor of the investor and his heirs. After receiving the deed, the investor made no effort to search the title, to examine the property, or to record the deed.In 2003, a vintner paid the farmer $20,000, and the farmer delivered to the vintner a deed of the vineyard in favor of the vintner and his heirs. The vintner had examined the property, had searched the title, and had no knowledge of the farmer’s awareness of the prior landowner-to-son instrument. Although the vintner did not reside on the property, he regularly visited the vineyard twice a week. The vintner recorded his deed.In 2007, for $25,000, paid by a farmer, the investor signed and delivered a deed of the vineyard naming the farmer and his heirs as grantees.Before the farmer had paid the investor and taken his deed, the farmer visited the vineyard and observed the fence. However, the vintner was not present when the farmer visited the property, and nothing suggested who—if anyone—was using it. In any case, the farmer did not attempt to search the title before making his purchase. The farmer then recorded the deed. This jurisdiction uses Grantor— Grantee Indices and has no Tract IndexIn 2008, what is the present state of title to the vineyard if the jurisdiction’s recording act provides:“Every conveyance of real estate which is not recorded is void against a subsequent purchaser in good faith for valuable consideration, whose conveyance shall be first duly recorded”?(A) In a race—notice jurisdiction, the farmer, as a subsequent bonafide purchaser, would acquire record title to the vineyard, because he was the last in time to record.(B) In a pure race jurisdiction, the farmer would acquire record title to the vineyard, because he purchased the property for value and recorded first in time.(C) In a race—notice jurisdiction, the vintner, as a subsequent bonafide purchaser without notice of the prior landowner-to-son instrument, would acquire record title to the vineyard, because he recorded his deed first in time.(D) In a pure race jurisdiction, the vintner, as a subsequent bonafide purchaser without notice of the prior landowner-to-son instrument, would acquire record title to the vineyard, because he recorded his deed first in time.
A
  1. (C) The recording act referred to in this question is a race-notice type statute. Under such a statute, an unrecorded conveyance or other instrument is invalid as against a subsequent bona fide purchaser for value without notice, who records “first.” In order for a subsequent party to prevail in a race-notice jurisdiction, he must be both a bona fide purchaser for value without notice of the prior interest and record first. The vintner was a subsequent BFP for value and without notice of the prior landowner-to-son deed. Moreover, the vintner purchased the vineyard in 2003 and immediately recorded the deed. The vintner recorded his deed first in time. (Note:the farmer did not purchase the property until 2007). Thus, the vintner is the record titleholder of the vineyard in 2008 under this race-notice recording statute. Choice (A) is incorrect. The last in time to record is never rewarded in a race-notice jurisdiction. If anyone, it is the first to record that will take the property. Choice (B) is incorrect. This is not a pure race jurisdiction. Furthermore, in this race-notice jurisdiction, the farmer would not be able to acquire title because the farmer knew about the prior grant from the landowner to the son. Therefore, he could never take without notice. Choice (D) is incorrect. This is worded the same as choice (C), except this choice presumes that this is a pure race jurisdiction. Because this is a race-notice jurisdiction, this choice is incorrect.
98
Q
  1. In 1993, a farmer had good record title to a farm in fee simple absolute. In 1994, the farmer delivered to his son, for a sum of $1,000, a deed signed by the farmer, naming the son and his heirs as grantee, and appearing valid on its face. The son neglected to record the deed.In 1998, an investor, aware of the existence of the farmer-to-son deed, sought out the farmer and asked to buy for $10,000 a deed of the farm from the farmer to the investor and his heirs. The farmer executed such a deed, and the investor promptly recorded it. The investor’s intent was to acquire color of title and obtain ownership of the farm by adverse possession. In 1998, the investor constructed a fence around the farm.In 1999, the son presented his deed of the farm to a rancher and, for $15,000, paid by the rancher, signed and delivered a deed of the farm in favor of the rancher and his heirs. After receiving the deed, the rancher made no effort to search the title, to examine the property, or to record the deed.In 2003, a teacher paid the investor $20,000, and the investor delivered to the teacher a deed of the farm in favor of the teacher and his heirs. The teacher had examined the property, had searched the title, and was aware of the prior farmer-to-son instrument. Although the teacher did not reside on the property,he regularly visited the farm twice a week. The teacher recorded his deed.In 2007, for $25,000, paid by a professor, the rancher signed and delivered a deed of the farm naming the professor and his heirs as grantees.Before the professor had paid the rancher and taken his deed, the professor visited the farm and observed the fence. However, the teacher was not present when the professor visited the property, and nothing suggested who—f anyone—was using it. In any case, the professor did not attempt to search the title before making his purchase. The professor then recorded the deed. Note: This jurisdiction uses Grantor—Grantee Indices and has no Tract IndexThe jurisdiction’s recording act provides: “Every conveyance of real estate which is not recorded is void against a subsequent purchaser in good faith for valuable consideration, whose conveyance shall be first duly recorded.”In 2008, in an action to quiet title to the farm, which of the following parties would have priority of title?(A) The professor.(B) The investor.(C) The teacher.(D) The rancher.
A
  1. (A) In 2008, the professor would succeed in an action to quiet title to the farm under a race-notice type recording statute. In order to gain priority of title in a race-notice jurisdiction, a subsequent party must be (1) a BFP for value without notice of any prior interests, and (2) record first. Because the investor and the teacher were both aware of the existence of the priorfarmer-to-son deed, the notice requirement could not be met as to them. Furthermore, the rancher, who was unaware of any prior interests, failed to record first in time. Therefore, the professor would prevail because he was the only party to satisfy both requirements; he was a subsequent BFP for value without notice (paying $25,000 for the property) and was the first to record (between the rancher and himselO. Choice (B) is incorrect. Because the investor was aware of the prior farmer-to-son deed, the investor could not fulfill the (lack oO notice requirement and would not have priority of title under a race-notice recording statute. Choice (C) is incorrect. Because the teacher was also aware of the prior farmer-to-son deed, the teacher could not fulfill the (lack o1 notice requirement and would not have priority of title under a race-notice recording statute. Choice (D) is incorrect. Even though the rancher was unaware of any prior conveyances, he did fail to record, so he cannot meet the requirement of being the first to record. Therefore, the rancher cannot prevail here.
99
Q
  1. On February 1, a retiree conveys his farm to an artist, and the artist duly records the conveyance. The following day, the artist conveys the property to a bartender; she does not record her deed. Then on February 4, the artist executes an identical conveyance of the farm to a caterer. The caterer gives the artist a check for $100,000 for the property and records the conveyance, even though he has actual knowledge of prior conveyance to the bartender. The bartender, however, records her deed on February 6. The caterer then conveys his interest in the farm to a dancer, who gives a purchase price of $115,000 to the caterer. On February 5, the dancer purchases the farm without notice of the conveyance to the bartender and duly records the deed.In conducting a title search, the dancer should pursue his investigation by looking in the(A) Grantor Index under the caterer’s name to ascertain if the caterer acquired title.(B) Grantee Index under the caterer’s name only.(C) Grantee Index under the caterer’s name, then the Grantor Index under the caterer’s name, and then in the Grantee Index again, this time under the artist’s name to discover if he acquired title.(D) Grantee Index under the bartender’s name, then to the Grantor Index, also under the bartender’s name to find out if she made any prior conveyances.
A
  1. (C) Because the dancer takes from the caterer, the dancer will want to learn all about the caterer. The dancer will look under the caterer’s name in the Grantee Index to find that the caterer’s grantor was the artist. Then the dancer will want to look in the Grantor Index under the caterer’s name to also see if the caterer granted to someone else before granting to the dancer. Then the dancer would want to see what happened when the artist was a grantee and see when the artist acquired title and would find the retiree as grantor. It is not clear from the facts if the retiree is the original owner, but at some point in this search, the dancer will go back far enough and find the origin and will have created his chain of title. Though this answer choice does not go all the way back in time, it does explain the method that the dancer must utilize to go back in time, so that he can conduct a title search properly. Because the method outlined is correct as stated, this is the correct choice. Choice (A) is incorrect. It does not make sense to only look under the caterer’s name in the Grantor Index because that will only tell the dancer if the caterer had previously granted the property to someone else in the past. The dancer also wants to know what happened when the caterer himself was a grantee because that will help the dancer go back in time and start building the chain of title. Because the process laid out in this choice does not show how the dancer would go back in time (to build a proper chain of title) it is incomplete and incorrect. Choice (B) is incorrect. This procedure would only tell the dancer who granted the property to the caterer. By using the word “only,” this choice states that all that would be necessary to conduct a proper title search is to find out who granted to the caterer. However, that singular act alone will not help the dancer search the entire title. The dancer will certainly want to see if the caterer granted to another person before granting to the dancer. That alone would require the dancer to also search the Grantor Index to see any other transactions where the caterer was a grantor. Because the process laid out in this choice does not show how the dancer would go back in time (to build a proper chain of title) it is incomplete and incorrect. Choice (D) is incorrect. This choice focuses the search on the wrong party. The dancer would want to search under his own grantor’s name, not under the name of another grantee such as the bartender.
100
Q
  1. On February 1, a woman conveys her farm to a man, and the man duly records the conveyance. The following day, the man conveys the property to his sister; she does not record her deed. Then on February 4, the man executes an identical conveyance of the farm to a friend. The friend gives the man a check for $100,000 for the property and records the conveyance, even though he has actual knowledge of the prior conveyance to the sister. The sister, however, records her deed on February 6. The friend then conveys his interest in the farm to a farmer, who gives a purchase price of $115,000 to the friend. On February 5, the farmer purchases the farm without notice of the conveyance to the sister and duly records the deed.If the property in question was located in a state having a notice-type statute, which of the following parties would ultimately prevail?(A) The farmer.(B) The sister.(C) The friend.(D) The man.
A
  1. (A) Under a notice-type statute, the subsequent bona fide purchaser without notice prevails over the prior interest, whether the subsequent purchaser records or not. The farmer is the last person to take without notice. The farmer had no constructive notice that the man had already conveyed to the sister because the sister did not record until after the friend conveyed to the farmer. The farmer also had no actual notice of the sister because the facts state that the farmer had no notice of the prior conveyance to the sister. Choice (B) is incorrect. The sister will not prevail because there is another subsequent bona fide purchaser who came after her who paid consideration and tookwithout notice. If the sister had wanted to forecLose the possibility of a later conveyee prevailing over her, she could have simply recorded immediately. This recordation would have put the farmer on notice that the man had already conveyed to the sister (on February 2) and thus had nothing to convey to the friend (on February 4). Choice (C) is incorrect. The friend actually knew that the man had already conveyed to the sister so he could not, in good faith, believe that the man had any estate left to grant to the friend. The friend cannot be protected by a notice statute and will not prevail. Choice (D) is incorrect. The farmer would prevail under a notice statute, not the man.
101
Q
  1. On February 1, a man conveys his orchard to a gardener, and the gardener duly records the conveyance. The following day, the gardener conveys the property to a landscaper; she does not record her deed. Then on February 4, the gardener executes an identical conveyance of the orchard to a friend. The friend gives the gardener a check for $100,000 for the property and records the conveyance, even though he has actual knowledge of the prior conveyance to the landscaper. The landscaper, however, records her deed on February 6. The friend then conveys his interest in the farm to an investor, who gives a purchase price of $115,000 to the friend. On February 5, the investor purchases the farm without notice of the conveyance to the landscaper and duly records the deed.Suppose that the jurisdiction in which the property is located has a pure race statute as their deed recordation law. Under these circumstances, which of the aforementioned parties would ultimately prevail?(A) The investor.(B) The landscaper.(C) The friend.(D) The gardener.
A
  1. (A) In a pure race jurisdiction, the first to record wins; a subsequent purchaser need not be bona fide and without notice because he will prevail if he records first. Because the investor recorded before the landscaper, he will prevail under a race statute over the landscaper. Note that the friend will not be part of the competition for title because he has already conveyed the property to the investor. At most, it will be a competition between the investor and the landscaper. Choice (B) is incorrect. The landscaper will not prevail because as between her and the investor, she was last to record. Because of this, she will not be the one to ultimately prevail. Choice (C) is incorrect. The friend wiLL not be part of the competition for title because he has aLready conveyed the property to the investor. Thus, it will be a competition between the investor and the landscaper. Choice (D) is incorrect. Because the investor will prevail in a race jurisdiction, the gardener will not.
102
Q
  1. A deed executed by a woman in 2001 conveyed a tract of land for a consideration of one dollar, receipt of which was acknowledged, “to my friend for life, but if liquor is ever sold on the tract of land, then to my cousin and his heirs, and if for any reason the interest hereby conveyed to my cousin is not valid, then I reserve the right to re-enter the tract of land and take back my property.” In 2004, the friend died intestate before the wheat he had planted could be harvested.Who is entitled to the proceeds of the crop?(A) The friend’s heirs.(B) The cousin.(C) The woman.(D) Divided equally between the friend’s heirs and the woman.
A
  1. (A) For Multistate purposes, students are required to know that there are two types of crops: (1) fructus naturales, those that grow naturaLly without the aid of man, such as trees, bushes, grasses and the fruits of these, and (2) fructus industriales, those that come primariLy from annual planting, cultivating, and fertiLizing, such as wheat, beans, corn, and citrus fruits in orchards. According to the prevailing view, fructus naturales are and remain real property for all purposes until they are actually severed from the land. On the other hand, fructus industriales (also referred to as emblements) are usually annual crops that are, for most purposes, personal property. See Smith and Boyer, Law of Property, pp. 244—245. Because wheat is a fructus industriales crop (or an emblement), it is viewed as being personatty, and so the proceeds from its sale pass to the life tenant’s heirs. Consequently, choice (A) is correct. Choice (B) is incorrect. Because the wheat will be considered an emblement and will pass as the personal property of the deceased life tenant, the friend’s heirs will be entitled to take the proceeds of the crop. Therefore, the cousin will not be entitled to this personal property. Choice (C) is incorrect. Because the wheat will be considered an emblement and will pass as the personal property of the deceased Life tenant, the friend’s heirs wiLL be entitled to take the proceeds of the crop. Therefore, the woman will not be entitled to this personal property. Choice (D) is incorrect. Because the wheat will pass as the personal property of the deceased life tenant, the friend’s heirs will be entitled to the proceeds of the crop and the heirs do not need to share the proceeds with the woman, the original grantor.
103
Q
  1. A widower was the record owner of a lemon grove, a 30-acre parcel of land in a suburb. The widower lived in a farmhouse on the lemon grove and used the property to raise produce. Adjoining the lemon grove directly to the west was a 10-acre farm that was owned in fee simple by a farmer. There was a four-lane highway that adjoined the lemon grove directly to the east.The widower, by way of gift, executed a deed naming his daughter as grantee. The deed granted to the daughter and her heirs an antebellum mansion located near the southern edge of the lemon grove. The antebellum mansion was accessible by a little- used road that ran west to east from the farmlgrove border to the four-lane highway along the southern boundary of the grove. The daughter recorded her deed and took immediate possession of the property. A short while later, the daughter and the farmer fell in love and began seeing each other quite frequently. In order for the farmer to reach the daughter’s house, it was necessary for him to travel over the little-used road across the lemon grove.Many years later, the farmer, who was still having an affair with the daughter, met her father at a Rotary Club meeting. They struck up a conversation, and the widower asked the farmer, “Have you been driving your pickup along that little-used back road on my property?” The farmer, who was afraid to tell the widower about his love affair with the daughter, responded with a half-truth, “Yes, I’ve been using it as a shortcut to the four-lane highway.” Unaware that the farmer was also using the path to get to the antebellum mansion to see the daughter, the widower said, “No problem, I just wanted to be sure that it was you who was using the road.”Thereafter, the widower found out about his daughter’s relationship with the farmer. Infuriated, the widower confronted the farmer and told him, “Listen, you lying sneak, if I catch you on my property again, I’m going to have you arrested for trespass.” The farmer replied, “Sorry, Pops, but I’ve acquired an easement over that roadway, and I’ll continue to use it anytime I want.”Then, the widower institutes an appropriate action to enjoin the farmer from using the roadway across the lemon grove. If the widower prevails, it will be because the farmer’s use was(A) fraudulent.(B) permissive.(C) not continuous.(D) not open and notorious.
A
  1. (B) An easement by prescription arises by adverse use of the servient tenement by the dominant tenant for the period of the statute of limitations. To mature, such an easement against a landowner the user must be (1) adverse as distinct from permissive, (2) open and notorious, (3) continuous and without interruption, and (4) for the period of prescription. In order to acquire title to an easement by adverse user (as well as to gain title to land by adverse possession) it must be gained without the consent of the landowner. Remember that no one who makes use of the servient tenement With the consent or permission of the owner can claim title to a prescriptive easement. Thus, if the farmer had the widower’s permission to use the roadway, then he was given a license. A license simply permits one person to come onto land in the possession of another without being a trespasser. As such, a license arises frOm consent given by the one in possession of land. Consent being given, no prescriptive right can arise through a license. Choice (A) is incorrect. In order to gain an easement by prescription, the use must be hostile and adverse, meaning not permissive. There is no special requirement that the person using the land intended to do so in such a way as to perpetrate a fraud against the true record owner. While it would be helpful to the widower to argue that his statement to the farmer constituted permission, it would be irrelevant for the widower to argue that the farmer’s use was fraudulent. In other words, the farmer’s desire to commit a fraud would be irrelevant if the use were truly adverse. Choice (C) is incorrect. There are not enough facts here to know for certain whether or not the farmer used the roadway for the prescriptive period. However, the facts do state for certain that the widower did not mind the farmer’s use of the road. It was only after the widower found out about the affair that he considered it trespassing. For this reason, the widower’s strongest argument here would be that the use was permissive instead of non-continuous. Choice (D) is incorrect. This element requires that the use of the road cannot be secret or clandestine. The use must be done in a way that a true owner would use the roadway. There are not enough facts here to know for certain whether or not the farmer used the roadway in such a way that the whole world could see. However, the facts do state for certain that the widower did not mind the farmer’s use of the road. It was only after the widower found out about the affair that he considered it trespassing. For this reason, the widower’s strongest argument here would be that the use was permissive instead of not open and notorious.
104
Q
  1. A landowner was the record owner of a 30-acre orchard outside the city. The landowner lived in a farmhouse on the orchard and used the property to raise produce. Adjoining the orchard directly to the west was a 10-acre vineyard that was owned in fee simple by a farmer. A four-lane highway adjoined the orchard directly to the east.The farmer discovered that the southern portion of the orchard was rarely used by the landowner for any of the landowner’s farming activities and found a convenient gravel road leading from the vineyard across the orchard all the way to the four- lane highway. The farmer used this road adversely and openly for the entire 20-year prescriptive period, and in doing so, the farmer has acquired a prescriptive easement over the roadway across the orchard. Thereafter, the farmer conveys the vineyard to a buyer in fee simple. The deed recited that “the grantor hereby conveys the vineyard, together with an easement for the right of way across the orcharX, to the grantee.”After the buyer took possession of the vineyard, the landowner brought an appropriate action to prevent him from using the roadway across the orchard. The issue that will determine the outcome of this suit is whether(A) the description in the farmer’s deed to the buyer was adequate to identify the portion of the orchard that the farmer used as a roadway.(B) the buyer will make excessive use of the roadway.(C) easements can be reserved to third parties. (D) the easement was appurtenant.
A
  1. (D) According to the general rule, when an easement appurtenant is created either by prescription or by conveyance over a servient tenement, the boundaries and extent of such easement become fixed and are binding on both the servient and dominant tenants. Neither such tenant has any more right to change the location of such easement than he has to change the boundaries of the physical servient or dominant tenements. The easement is an incorporeal right, which is real property. So, if this roadway is found to be an easement appurtenant that benefits the dominant tenement, it will be valid and can be conveyed because it would be a real-property interest. Choice (A) is incorrect because the description in the deed instrument is sufficient to identify the land conveyed. Choice (B) is wrong because mere excessive use of an easement (or profit) does not forfeit or extinguish the easement. In the case of excessive use, the owner of the easement is simply making use of the servient tenement beyond the scope or extent of the use permitted by the easement. So, even if this issue were decided, at most it would tell the parties how much the easement can be used. Because the landowner is trying to stop the buyer completely, the resolution of this issue will still not be helpful. Choice (C) is not correct because according to the language of the deed, the farmer (the grantor) is not reserving an easement to the buyer as a third party. The farmer is simply conveying the vineyard (together with the easement for the right of way across the orchard) to the buyer as the grantee (or second party) to the real estate transaction. It obviously wasn’t the farmer’s intent to reserve an easement to the buyer as a third party. Moreover, most cases hold that a reservation can be made for no one other than the grantor.
105
Q
  1. A 73-year-old widower owned a 40-acre farm. The widower had two children, a son and a daughter. After the daughter married, she and her husband lived on the farm in a small cottage. From 1985 to 1989, the daughter and the husband helped the widower farm and maintain the property. The widower, whose health was deteriorating, needed the services of the daughter and her husband in order to continue to live on the farm. In December 1989, the daughter told the widower that she and her husband were planning to move out of state. Worried that he could not survive without their help, the widower said to the daughter, “Please don’t ever leav.e. I’m totally dependent on you and your husband. If you stay and continue to care for me and help with the farming, the farm will be yours when I die.” The daughter turned down a job offer in a neighboring state and decided to do as the widower requested. For nine years, the daughter cared for her father while her husband handled most of the farming operations. In 1998, the widower died intestate with the daughter and the son as his only surviving heirs. The period required to acquire title by adverse possession in the jurisdiction is seven years.In an appropriate action to determine the legal and equitable rights of the daughter and the son, respectively, in the farm, the result will depend upon the application of the principles of and exceptions to the(A) statute of frauds.(B) parol evidence rule.(C) law for adverse possession.(D) doctrine of resulting trusts.
A
  1. (A) The statute of frauds requires a writing for enforceability of any contract to create or transfer any interest in land, which includes not only regal estates but also equitable interests and liens, rents, expectant interests and estates, leases, easements, and restrictions upon the use of land. Equity, however, will decree specific performance of an oral land contract, despite the statute of frauds, where there has been substantial reliance on the oral promises. Because this question concerns itself with the daughter and her husband’s reliance on the widower’s oral promise to convey the farm, the statute of frauds is the applicable legal principle. Choice (B) is wrong because parol evidence requires a complete and integrated writing. Here, no writing was entered into between the parties. Choice (C) is incorrect. Adverse possession, as denoted in choice (C), is inapplicable because the daughter and her husband’s possession of the farm was not hostile and adverse. Choice CD) is incorrect. Resulting trusts arise when an express trust fails or when an existing trust fails to exhaust all of the assets of the trust. At this point the court will step in and create a resulting trust based on the settlor’s intent. Choice (D) is incorrect because the facts do not indicate the widower intended to establish a trust.
106
Q
  1. A land development company was the owner of a400-acre tract of land in the Great Lakes region. Over the course of time, the land development company developed two residential subdivisions of the land, an eastern development and a western development, each of which contained 150 acres. These subdivisions were created by separate plats that made no reference to each other. The restrictions in the plats were, however, substantially identical. The plats and each deed provided that “the use of the land herein conveyed is restricted to single- family dwellings only, and this covenant is intended to apply to each and every lot in the subdivision and runs with the land, binding every lot owner, his heirs, and assigns.”After all but four lots in each subdivision had been sold by the land development company, it sold 50 acres of the remaining 100 acres of land to a country club by a deed containing the following provisions:“This deed is executed and accepted with the understanding that the property above described is hereby restricted so that(1) said property may be used as a country club, with a golf course, pool, tennis courts, club house, eating facilities, and other improvements appropriate to a country club.(2) said property may also be subdivided and platted as a residential subdivision similar to the eastern development and the property shall thereafter be used in accordance with and conveyed subject to residential restrictions that shall conform with those restrictions in force against the eastern development.(3) the restrictions herein contained shall be deemed covenants running with the land, and for breach of any covenant herein, grantor— land development company, its successors and assigns may, at its option, re-enter and terminate the estate conveyed hereby.”At the time of this conveyance, the land development company retained title to the remaining 50 acres in the original 400-acre tract.Which of the following would best describe the country club’s interest in the SO-acre tract that it purchased from the land development company?(A) Fee simple determinable.(B) Fee simple subject to condition subsequent.(C) Determinable fee subject to an executory interest.(D) Easement appurtenant.
A
  1. (B) A fee simple subject to condition subsequent means a fee simple subject to being terminated by exercise of the power of termination or right of re-entry for condition broken. What distinguishes this type of estate from a fee simple determinable is that the estate will continue in the grantee, or his successors, unless and until the power of termination is exercised. The estate in fee simple subject to a condition subsequent does not end automatically upon the happening of the named event. The basic difference, therefore, is that the fee simple determinable automatically expires by force of the special limitation contained in the instrument creating the estate when the stated contingency occurs, whereas the fee simple on condition subsequent continues despite the breach of the specified condition until it is divested or cut short by the exercise by the grantor of his power to terminate. In clause (c), the land development company and its successors had the right to reenter and terminate the estate and this granted them a right of re-entry. Choice (A) is incorrect. A fee simple determinable is a fee simple estate created to continue until some specified event occurs. The estate terminates automatically. The principal difference between the two is as follows: In the determinable fee, the estate automatically comes to an end when the stated event happens, whereas in the fee simple subject to condition subsequent, the termination of the estate is not automatic, but must be terminated by the entry or exercise of the reserved power by the grantor. Typical words that will create a fee simple determinable include “so long as” and “during.” What the country club has here is best described as a fee simple subject to condition subsequent because there were no words of duration used in the grant and because the land development company specifically reserved a right of re-entry. Choice (C) is incorrect. A determinable fee subject to an executory interest is a fee simple estate, whereupon the happening of a named event, ownership is to pass from the grantee to one other than the grantor. If anything, clause (c) reserved a right for the grantor or successors/assigns to re-enter and terminate the present estate. Because the ownership would pass to grantor, this is not a determinable fee subject to an executory interest. Choice (D) is incorrect. An easement is the right of one person to go onto the land in possession of another and make a limited use thereof. An easement is appurtenant when it is attached to a piece of land and benefits the owner of such land in his use and enjoyment thereof. Every easement appurtenant requires two pieces of land that are owned by two different persons. The country club did not receive a right to use the land of another; rather, the country club received a defeasible fee interest in the property.
107
Q
  1. A land-development company was the owner of a400-acre tract of land in the Southwest. Over the course of time, the land-development company developed two residential subdivisions of the land, an eastern tract and a western tract, each of which contained 150 acres. These subdivisions were created by separate plats that made no reference to each other. The restrictions in the plats were, however, substantially identical. The plats and each deed provided that “the use of the land herein conveyed is restricted to single-family dwellings only, and this covenant is intended to apply to each and every lot in the subdivision and runs with the land, binding every lot owner, his heirs, and assigns.”After all but four lots in each subdivision had been sold by the land-development company, it sold 50 acres of the remaining 100 acres of land to a country club by a deed containing the following provisions:“This deed is executed and accepted with the understanding that the property above described is hereby restricted so that(1) said property may be used as a country club, with a golf course, pool, tennis courts, club house, eating facilities, and other improvements appropriate to a country club.(2) said property may also be subdivided and platted as a residential subdivision similar to the eastern tract and the property shall thereafter be used in accordance with and conveyed subject to residential restrictions that shall conform with those restrictions in force against the eastern tract.(3) the restrictions herein contained shall be deemed covenants running with the land, and for breach of any covenant herein, the land-development company, its successors and assigns may, at its option, re-enter and terminate the estate conveyed hereby.”At the time of this conveyance, the land- development company retained title to the remaining 50 acres in the original 400-acre tract. Thereafter, the land-development company developed an exclusive shopping center on 25 acres of the retained land. In February 2001, the land-development company sold the remaining eight residential lots in the eastern tract and the western tract. The next month, the land-development company executed the following instrument to the country club:“The land-development company, for itself, its successors, and assigns, does hereby release,surrender and quitclaim all rights, title, or other property interest in that certain acres owned by the country club.”At the time this instrument was executed, the country club had built a club house, golf course, and tennis courts on a portion of its land, and it had 25 acres of vacant land remaining. The country club wishes to commence construction of a new high- rise complex (containing a hotel, shopping mall, apartments, etc.) on the 25 acres of vacant land it possesses.In an action by one of the homeowners in the western tract to enjoin construction of the complex, plaintiff will most likely(A) succeed, because the homeowner (or any other landowner in the eastern tract or the western tract) as assignee of the land development company could re-enter the land upon breach of condition subsequent.(B) succeed, because a common development scheme had been established for the entire 400-acre tract, and the country club’s proposed complex would constitute a non-conforming use.(C) not succeed, because the land development company’s instrument of March 2001 effectuated an abrogation of the deed restrictions on the country club’s use of its property.(D) not succeed, because in accordance with deed restriction (b), only a homeowner in the eastern tract would have standing to challenge the country club’s proposed construction plan.
A
  1. (C) Choice (C) is the best alternative because the land-development company’s March,2001 instrument effectively abrogated the deed restrictions on the country club’s use of its property. By abrogating the restrictions, the land development company was essentially giving up their future interest in the property because the land- development company could no longer use the violation of the restriction as a means to re-enter and take the property. Clearly, a common developmental scheme was not established for the 400-acre tract. Because the land-development company developed a shopping center on 25 acres of its retained land and also authorized the country club to construct a country club and golf course on its 50 acres, the land-development company intended that the residential developmental scheme should extend only to the 300 acres within the eastern tract and western tract subdivisions. Choice (A) is incorrect. First, the land-development company could no longer re-enter the land upon breach of condition subsequent because the condition had been waived by the land-development company, the grantor. Second, the homeowner is, at most, a grantee of the land-development company and is not a successor or assign of the land-development company. Third, the homeowner is not asking to re-enter the premises and is not asking for an ownership interest in it; rather, the homeowner was seeking to enjoin the construction of the complex. Choice (B) is incorrect. A common developmental scheme was not established for the 400-acre tract. Because the land-development company developed a shopping center on 25 acres of its retained land and also authorized the country club to construct a country club and golf course on its 50 acres, the land-development company intended that the residential developmental scheme should extend only to the 300 acres within the eastern tract and the western tract subdivisions. Choice (D) is incorrect. The deed restriction is there to illustrate the type of restriction that the country club-would have been required to abide by and follow. This is not to say that only the eastern tract residents would have had standing to challenge the construction plan. If there were a common development scheme in place for the entire 400-acre tract, then the homeowner would have had standing to challenge the construction as a resident of the project despite deed restriction (b). The better reason why the homeowner will not succeed is that there are no more restrictions on the country club, and, thus, no residential owner can enjoin the project.
108
Q
  1. A land-development company was the owner of a 400-acre tract of land. Over the course of time, the land-development company developed two residential subdivisions of the land, an eastern subdivision and a western subdivision, each of which contained 150 acres. These subdivisions were created by separate plats that made no reference to each other. The restrictions in the plats were, however, substantially identical. The plats and each deed provided that “the use of the land herein conveyed is restricted to single-family dwellings only, and this covenant is intended to apply to each and every lot in the subdivision and runs with the land, binding every lot owner, his heirs, and assigns.”After all but four lots in each subdivision had been sold by the land-development company, it sold 50 acres of the remaining 100 acres of land to a country club by a deed containing the following provisions:“This deed is executed and accepted with the understanding that the property above described is hereby restricted so that(1) said property may be used as a country club, with a golf course, pool, tennis courts, club house, eating facilities, and other improvements appropriate to a country club.(2) said property may also be subdivided and platted as a residential subdivision similar to the eastern subdivision and the property shall thereafter be used in accordance with and conveyed subject to residential restrictions that shall conform with those restrictions in force against the eastern subdivision.(3) the restrictions herein contained shall be deemed covenants running with the land, and for breach of any covenant herein, the land development company, its successors, and assigns may, at its option, re-enter and terminate the estate conveyed hereby.”At the time of this conveyance, the land- development company retained title to the remaining 50 acres in the original 400-acre tract. Within a few months of the execution of this deed, the country club had built a club house, golf course, and tennis courts on a portion of its land, and it had 25 acres of vacant land upon which it wished to build a complex containing a hotel and shopping mall surrounded by high-rise buildings and luxury apartments.With respect to the 50-acre tract that the country club purchased from the land-development company, which of the following most accurately describes the deed restrictions (1) and (2)?(A) Affirmative covenant(s).(B) Equitable easement(s).(C) Express easement(s) appurtenant.(D) Equitable servitude(s).
A
  1. (D) An equitable servitude is a restriction on the use of land enforceable in a court of equity. An equitable servitude is more than “a covenant running with the land in equity” because itis an interest in land. The term “equitable servitude” is broader than “equitable easement” because it applies not only to land, but also to chattel property such as a business. Choice (A) is incorrect because the deed restrictions were negative—or restrictive—covenants rather than affirmative covenants. Choice (B) is incorrect. An equitable easement is an implied easement created by the courts to do equity when one party has been using the land of another and the party being burdened sues to compel removal of the use or encroachment. There is no need to resort to an implied easement because the equitable servitude was created by express language. Choice (C) is incorrect. An easement is the right of one person to go onto the land in possession of another and make a limited use thereof. An easement is appurtenant when it is attached to a piece of land and benefits the owner of such land in his use and enjoyment thereof. Every easement appurtenant requires two pieces of land that are owned by two different persons. The country club did not receive a right use the land of another in these deed restrictions; rather, the country club was being restricted in their use of the property.
109
Q
  1. In 1995, an investor purchased a 100-acre tract located in a rural county. Shortly thereafter, the investor prepared a subdivision plan, which created 90 one-acre residential building lots on this tract with the remaining 10-acre lot proposed for a public school building. In the investor’s sales brochure promoting the subdivision, he stated that “in addition to the close proximity of the proposed school for subdivision residents, the county school district would not need to expend tax money to acquire this property for school construction.” In 1996, the subdivision plan was recorded with the county recorder’s office.During the next few years, the investor sold 50 residential lots to individual purchasers.In 2002, the investor conveyed the remaining 40 lots and the 10-acre tract to a builder by deed that included language identical to that contained in the first 50 deeds. By 2007, the builder had sold all of the 40 lots. Each of these deeds identified each lot as being a part of the subdivision. On January 9, 2008, the builder sold the 10-acre tract to a buyer. This deed made no mention of the subdivision.On January 15, 2008, the county school board voted to build a new school on the 10-acre tract. Two weeks later, the buyer began construction of a pizzeria on the 10-acre tract.In an action by the school board against the buyer to enjoin construction of the pizzeria on the 10-acre tract, the court would grant judgment for(A) the buyer, because his own deed made no mention of the subdivision.(B) the buyer, because the dedication was not made to the public in general.(C) the school district, because the 10-acre tract was designated for public use.(D) the school district, because the 10-acre tract constituted an equitable servitude.
A
  1. (C) The proposal in the subdivision plan and also in the development brochure to set aside a 10-acre tract for the location of a school building would constitute a public dedication offering. Dedication at common law required no particular form and could be made by any method by which the dedicator expressed his intention: by words, conduct, or writing. When acceptance by public adoption is manifested, there arises a conveyance of an easement for such public use, with the fee remaining in the grantor. In the present case, the school district would be entitled to erect a school on the tract, thereby enjoining the buyer’s pizzeria. Choice (A) is incorrect. Even though the buyer’s deed made no mention of the subdivision, he will nonetheless be charged with constructive notice of the subdivision and the dedication because the deed that the builder received from the investor mentioned the subdivision and the dedication. Because those restrictions were in the buyer’s chain of title, he will be put on constructive notice of them. Choice (B) is incorrect. A dedication can be made from a private land holder to the public generally or to a public body. Here, the school board would be a valid public body. Choice (D) is incorrect. The school district first would need to argue that the land was dedicated to them because this would explain why they can enjoin the construction of the pizzeria. To argue that there is an equitable servitude only explains there is a restriction on the land and still does not explain why the school board has an enforceable interest in the land. Because choice (C) explains how the school acquired its interest (through a dedication), it is the strongest choice.
110
Q
  1. In 2006, an investor purchased a 100-acre tract located in a rural county. Shortly thereafter, the investor prepared a subdivision plan that created 90 one-acre residential building lots on this tract with the remaining 10-acre lot proposed for a public school building. In the investor’s sales brochure promoting the subdivision, he stated that “in addition to the close proximity of the proposed school for subdivision residents, the county school district would not need to expend tax money to acquire this property for school construction.” In 2007, the subdivision plan was recorded with the county recorder’s office.On January 15, 2009, the county school board voted to build a new school on the 10-acre tract.The investor’s proposed designation of the 1 0-acre tract for construction of a school building would best be described as a (an)(A) equitable servitude.(B) restrictive covenant.(C) unenforceable restriction.(D) easement for public use.
A
  1. (D) The public dedication of the 10-acre tract for the erection of a school would be construed as an easement for public use. Although ordinarily, dedication arises out of the creation of a public use in private land, a municipality or other subdivision may likewise acquire title and then create a public use by dedication. Choice (A) is incorrect. The investor transferred an interest in land from his own ownership to a public body, the school board. He did this through a dedication. An equitable servitude is not a transfer of an interest in land. Therefore, this is an incorrect description of the investor’s proposed designation of the tract. Choice (B) is incorrect. Although the investor’s dedication does contain a restrictive covenant, this is not the proper description for what is really a transfer of an interest in land. A restrictive covenant is not an interest in land and therefore an incorrect description of the investor’s proposed designation of the tract. Choice (C) is incorrect. It is incorrect to call the investor’s dedication a restriction because the investor transferred an interest in land to the school board. A restriction is not an interest in land and, therefore, an incorrect description of the investor’s proposed designation of the tract.
111
Q
  1. In 1995, a developer purchased a 100-acre tract located in a northern county of a state. Shortly thereafter, the developer prepared a subdivision plan that created 100 one-acre residential building lots on this tract. In 1996, the subdivision plan was recorded with the county recorder’s office.During the next few years, the developer sold 60 residential lots to individual purchasers. Each deed specified that every lot designated on the subdivision plan was to be recorded in the county recorder’s office. Each deed also provided the following:“No house trailer or mobile home shall be built or maintained on any lot within the subdivision.”In 2002, the developer conveyed the remaining 40 lots to a builder by deed that included language identical to that contained in the first 60 deeds. By 2007, the builder had sold all of the 40 lots. Each of these deeds identified each lot as being a part of the subdivision, but did not include the clause relating to mobile homes.On January 30, 2008, a man who had purchased one of the residential lots from the builder placed a mobile home on his property. A woman who owns a lot in the subdivision initiates suit against the man to force him to remove the mobile home.Which of the following would be the most accurate statement of law?(A) There is no enforceable restriction because the mobile-home provision did not “run with the land.”(B) There is no enforceable restriction because the man’s deed did not include the mobile-home provision.(C) The mobile-home restriction would be enforceable because a common development scheme had been established for the entire subdivision.(D) The outcome turns on whether a common development scheme had been organized for the entire subdivision.
A
  1. (C) When a developer subdivides her land into many parcels and some of the deeds contain a certain restriction and others do not, those restrictions may still be binding upon other purchasers, even though their deeds never contained the restriction. The following two requirements, however, must be met: (1) a common development scheme, and (2) notice. In order for there to be a common scheme, there must be a showing that when the project began, there was a common plan in place that all parcels would adhere to the same restrictive covenant. Here, there would be such a showing because the developer restricted the first 60 parcels and then sold the remaining 40 parcels to the builder by a singular deedthat included the same mobile-home restriction. Notice may be met by either actual notice, constructive notice, or inquiry notice, whereby the appearance of the properties would put a prospective buyer (such as the man) on notice that the neighborhood was planned to have certain characteristics. Because there was a common development plan in place forthe entire subdivision, the woman will prevail even though the man’s deed made no mention of the mobile-home restriction. Choice (A) is incorrect. The original deed provision between the developer and the builder prohibiting “house trailers and mobile homes to be built or maintained on any lot within the subdivision” is an example of a restrictive or negative covenant “running with the land.” Therefore, the restriction would be enforceable against the man, even though the prohibition was not contained in his deed. In order for a covenant to “run with the land,” the following four requirements must be met: (1) the covenant must be in writing; (2) the covenant must “touch and concern” the land; (3) privity of estate must exist between covenantor and covenantee; and (4) there must be intent that the covenant “run with the land.” Because this covenant did run with the land, this choice is incorrect. Choice (B) is incorrect. When a developer subdivides her land into many parcels and some of the deeds contain a certain restriction and others do not, those restrictions may still be binding upon other purchasers, even though their deeds never contained the restriction. The following two requirements, however, must be met: (1) a common development scheme, and (2) notice. In order for there to be a common scheme, there must be a showing that when the project began, there was a common plan in place that all parcels would adhere to the same restrictive covenant. Here, there would be such a showing because the developer restricted the first 60 parcels and then sold the remaining 40 parcels to the builder by a singular deed that included the same mobile-home restriction. Notice may be met by either actual notice, constructive notice, or inquiry notice, whereby the appearance of the properties would put a prospective buyer (such as the man) on notice that the neighborhood was planned to have certain characteristics. Because there was a common deveLopment plan in place for the entire subdivision, the woman will prevail even though the man’s deed made no mention of the mobile-home restriction. Because this answer choice states that the woman cannot enforce any restriction because the man’s deed was silent, it is incorrect. Choice (D) is incorrect. A common development scheme wILL be found to exist in the subdivision. In order for there to be a common scheme, there must be a showing that when the project began, there was a common plan in place that all parcels would adhere to the same restrictive covenant. Here, there would be such a showing because the developer restricted the first 60 parcels and then sold the remaining 40 parceLs to the builder by a singuLar deed that incLuded the same mobile-home restriction. Choice (C) is a more complete choice because in presuming that there is a common development scheme, choice (C) states that the restriction would be binding and that the woman would thus prevail.
112
Q
  1. In 1996, a developer purchased a 100-acre tract located in a northern county in a state. Shortly thereafter, the developer prepared a subdivision plan that created 100 one-acre residential building lots on this tract. In 1997, the subdivision plan was recorded with the county recorder’s office.During the next few years, the developer sold 60 residential lots to individual purchasers. Each deed specified that every lot designated on the subdivision plan was to be recorded in the county recorder’s office. Each deed also provided the following:“No house trailer or mobile home shall be built or maintained on any lot within the subdivision.”In 2003, the developer conveyed the remaining 40 lots to a builder by deed that included language identical to that contained in the first 60 deeds. This deed from the developer to the builder was recorded. By 2008, the builder had sold all of the 40 lots. Each of these deeds identified each lot as being a part of the subdivision, but did not include the clause relating to mobile homes.On January 30, 2009, a buyer, who had purchased one of the residential lots from the builder, placed a mobile home on his property.Which of the following statements is LEAST accurate with respect to the buyer’s deed?(A) The covenant prohibiting mobile homes ran with the land as far as the builder, but not as far as the buyer.(B) The covenant prohibiting mobile homes could be enforced by any subdivision lot owner.(C) The buyer should have had constructive notice of the restriction against mobile homes.(D) All subsequent grantees of the builder would be in privity of estate.
A
  1. (A) Choice (A) is the LEAST accurate because a covenant “running with the Land” will be enforceable against successors of the original parties where there is (1) privity of estate; (2) the covenant “touches and concerns” the land; (3) intent that the covenant “run with the Land”; and (4) the covenant be in writing. This statement is the least accurate because if the covenant runs with the land, then it can bind grantees beyond the builder. Choice (B) is incorrect. When a developer subdivides her land into many parcels and some of the deeds contain a certain restriction and others do not, those restrictions may still be binding upon other purchasers, even though their deeds never contained the restriction. The following two requirements, however, must be met: (1) a common development scheme, and (2) notice. In order for there to be a common scheme, there must be a showing that when the project began, there was a common plan in place that all parcels wouLd adhere to the same restrictive covenant. Here, there would be such a showing because the developer restricted the first 60 parcels and then sold the remaining 40 parcels to the builder by a singuLar deed that included the same mobile-home restriction. Notice may be met by either actual notice, constructive notice, or inquiry notice, whereby the appearance of the properties would put a prospective buyer (such as the buyer in this question) on notice that the neighborhood was planned to have certain characteristics. Because there was a common development pLan in place for the entire subdivision, any lot owner (in the subdivision) can enforce the covenant against the buyer, even though the buyer’s deed made no mention of the mobile-home restriction. Because this answer choice is an accurate statement, it is incorrect. Choice (C) is incorrect. Though the restrictive covenant was not written into the buyer’s deed, he would, nevertheless, be held to have constructive knowledge of the prohibition against mobile homes and house trailers. Because the deed from the developer to the builder was in the buyer’s chain of titLe, it will put the buyer on constructive notice of those restrictions because the covenants appeared in the deed. Because the buyer will be held to have constructive notice of these covenants, this answer choice is an accurate statement and thus incorrect. Choice (D) is incorrect. This is an accurate statement because all subsequent grantees of the builder would be in privity of estate and thus would be bound by the covenant. Because the builder is one of the original covenanting parties (the developer being the other), any grantee of the builder’s would be in privity of estate and thus be bound.
113
Q
  1. In 1996, an investor purchased a 100-acre tract located in a northern county of a state. Shortly thereafter, the investor prepared a subdivision plan that created 100 one-acre residential building lots on this tract. In 1997, the subdivision plan was recorded with the county recorder’s office.During the next few years, the investor sold all 100 residential lots to individual purchasers. Each deed specified that every lot designated on the subdivision plan was to be recorded in the county recorder’s office. Each deed also provided the following: “No lot shall be used except for residential purposes.” By 2009, the area surrounding the subdivision was rezoned for commercial and business uses. One of the lot owners now decides to operate a small beauty parlor in the basement of her home.In an action by the homeowners in the subdivision to prevent such commercial use, the court will most likely hold that(A) the residency restriction is no longer enforceable because of the change in the character of the neighborhood surrounding the development.(B) the residency restriction is no longer enforceable because the area surrounding the development was rezoned for commercial use.(C) the residency restriction is enforceable, thus preventing any commercial use.(D) the operation of a beauty parlor would not constitute a violation of the residency restriction.
A
  1. (C) The restriction “except for residential purposes” should be interpreted to prohibit the operation of a beauty parLor in defendant’s home. Such commercial activity clearly violates the plan and obvious purpose of the covenant and is prohibited by its terms. Choice (A) is incorrect because the change in character of the surrounding neighborhood would not justify failure to enforce the restriction where the original purpose of the restriction could still be realized. Choice (B) is incorrect because the rezoning to commercial use (in the surrounding neighborhood) would not justify failure to enforce the restriction where the original purpose of the restriction could stiLl be realized. Choice (D) is wrong because the operation of a beauty parlor in defendant’s home would constitute a violation of the restrictive covenant so as to justify injunctive relief.
114
Q
  1. A lawyer owned a 70-acre tract of land. In 1989, the lawyer sold 15 acres of the tract to a friend. The deed of conveyance contained the following clause:“The parties hereby covenant that if the grantor (the lawyer) proposes to sell any or all of the remaining 55 acres of the tract during (the friend’s) lifetime, then the grantee shall have the right of first refusal to purchase said parcel on the same terms and conditions as proposed; and, in the alternative, if grantee (the friend) proposes to sell any or all of the 15 acres of his parcel during (the lawyer’s) lifetime, then (the lawyer) shall have the reciprocal right of first refusal.”The friend was approached by a co-worker who offered to purchase his 15-acre parcel for $100,000. The friend did not afford the lawyer an opportunity to exercise his right of first refusal, and he went ahead and sold the property to the co-worker. After the co-worker took possession, the lawyer then learned about the sale. The lawyer immediately brought suit against the friend and the co-worker to enforce the right of first refusal in the deed.Based on the facts as presented, the lawyer will(A) win, because the friend has a reciprocal right of first refusal.(B) win, because the lawyer’s right of first refusal does not violate the Rule against Perpetuities because it is limited to his lifetime.(C) lose, because the rights of first refusal are unreasonable restraints on alienation.(D) lose, because the rights of first refusal relate only to land that is not conveyed by deed.
A
  1. (B) The reciprocal rights of first refusal must “vest,” if at all, during the respective lifetimes of the lawyer and the friend. Because the Lawyer is a measuring life, we will know within the Lawyer’s lifetime whether he wilL exercise his option to purchase the friend’s Land. ConsequentLy, there is no violation of the rule. Choice (A) is incorrect. The lawyer does not win because the friend also had his own right of first refusal. Even if there were no reciprocal right of first refusal, the Lawyer would still prevail because his right of first refusal, standing alone, does not vioLate the RuLe against Perpetuities. Because this answer choice suggests that in order to have a valid right of first refusal, someone else must be given the same courtesy in return, it is incorrect. Choice (C) is incorrect. The inclusion of a right of first refusal in a deed instrument constitutes a partial restraint on alienation. Such restraints are usually valid, provided they do not violate the Rule against Perpetuities. Choice (D) is incorrect. A deed conveying land can include aright of first refusal. Such a right that is included in a deed can be enforced if necessary. Because rights of first refusaL can relate to land that is conveyed by deed, this answer choice is incorrect.
115
Q
  1. A retiree was the record title owner in fee simple absolute of a 100-acre farm. In 1998, the retiree devised the property to his daughter and his friend as joint tenants with right of survivorship. The next year, the daughter executed a deed to a co-worker as follows:“I hereby convey all of my right, title, and interest in the northeast quarter of the farm to the co-worker and his heirs.”Thereafter, the daughter borrowed $100,000 from a lender, and a promissory note was executed as evidence of the debt. In 2008, the daughter defaulted on the loan, and the lender, as judgment creditor, levied upon and sold to his nephew on execution sale all of “(the daughter’s) right, title and interest in the south half of the farm.” In December 2009, the daughter died intestate, leaving her husband as her sole surviving heir.Who owns the farm?(A) The friend and the co-worker are tenants in common of the northeast quarter of the farm; the friend and the husband are tenants in common of the northwest quarter of the farm; and the friend and the nephew are tenants in common of the south half of the farm.(B) The friend and the co-worker are tenants in common of the northeast quarter of the farm; the friend is the owner in fee of the northwest quarter of the farm; and the friend and the nephew are tenants in common of the south half of the farm.(C) The friend and the co-worker are tenants in common of the northeast quarter of the farm; the friend and the husband are tenants in common of the northwest quarter of the farm; and the nephew is the owner in fee of the south half of the farm.(D) The friend and the co-worker are tenants in common of the northeast quarter of the farm; and the friend is the owner in fee of the remaining three-quarters of the farm.
A
  1. (B) In a joint tenancy, each joint tenant owns the whole of the property, nota share or fractional part thereof in this regard, a joint tenant may convey away his entire interest in jointly owned property or dispose of a fractional part thereof. The daughter’s deed to the co-worker carved out and vested in the co-worker an undivided one-half interest in the northeast quarter of the farm, which the co-worker owned as a tenant in common with the friend. Next, because a joint tenant has the right and powervoluntarily to dispose of his interest in jointly owned property, his creditors have the right and power to take that interest involuntarily. Therefore, the daughter’s judgment creditor, the lender, had the right to levy upon and sell the daughter’s interest in the south half of the farm to the nephew. As such, the friend and the nephew are tenants in common of the south half of the farm, each owning an undivided one-half interest therein. FinaLly, with respect to the northwest quarter of the farm, the daughter and the friend remained joint tenants of that quarter until the daughter’s death. The friend’s right of survivorship defeats the right of the daughter’s surviving heir, the husband. ConsequentLy, the husband can claim no interest in the northwest quarter of the farm. Choice (A) is incorrect. Regarding the northwest quarter of the farm, the daughter and the friend remained joint tenants of that quarter until the daughter’s death. The friend’s right of survivorship defeats the right of the daughter’s surviving heir, the husband. Consequently, the husband can cLaim no interest in the northwest quarter of the farm. Choice (C) is incorrect. Regarding the northwest quarter of the farm, the daughter and the friend remained joint tenants of that quarter until the daughter’s death. The friend’s right of survivor- ship defeats the right of the daughter’s surviving heir, the husband. Consequently, the husband can claim no interest in the northwest quarter of the farm. Moreover, the friend and the nephew are tenants in common of the south half of the farm. Choice (D) is incorrect. Because the daughter executed a deed to the co-worker and borrowed from the lender, it is not possible for the friend to be the fee owner outright of the remaining three-quarters under a right of survivorship theory.
116
Q
  1. A student acquired a 357-acre cattle ranch by inheritance from her father. In 2001, the student conveyed 57 acres of the cattle ranch tract to a buyer. As indicated in the diagram below, the 57-acre tract was not contiguous to any highway or public road. The deed from the student to the buyer conveyed the 57 acres and stated that the parcel would be used “for the purpose of building a home” and also stated that the buyer would receive “access to the land herein conveyed over the main trail across the cattle ranch.” The main trail is owned and maintained by the student.After the buyer took possession of his parcel, he built a home on the premises. Because no electrical power extended to his property, the buyer installed underground electrical wires to obtain electricity. The electrical wires extended beneath the main trail for a distance of approximately three miles.After the underground lines were installed, the student brought an appropriate action against the buyer demanding payment for the right to make affirmative use of the land beneath the main trail.If the court rules against the student and holds that the buyer has the right to maintain such electrical wires without any payment to the student, it will most likely be because(A) use of the main trail for installation of the electrical wires is within the scope of the easement contained in the deed instrument.(B) the buyer has an easement by implication because the main trail was in existence at the time of the conveyance to the buyer.(C) an easement in gross is established by necessity for the benefit of the quasi-dominant tenement owner.(D) there was an apparent and continuous prior use of the main trail by the student before the severance of ownership.
A
  1. (A) Generally, an easement carries with it certain other rights that allow the easement holder to make those improvements and maintenance requirements that are reasonable in order to allow that easement to carry out its intended purpose. One of the intended purposes was to build a home, and that is what the buyer did. A court will likely allow the buyer the further right to lay wires under the property to connect with an electrical source in order to effectuate the purposes of the easement and to fully enjoy the easement granted to him. Choice (B) is wrong because at the time of the conveyance an express easement was created in the deed instrument to permit the buyer to have access across the main trail. In order to have an easement by implication, two basic requirements must be met: (1) the benefited parcel and the burdened parcel must at one time have had a common owner, and (2) the circumstances surrounding the severance of the two parcels must indicate an intentto create such an easement. However, it is not necessary to create an easement in this way because an express easement was granted to the buyer. Choice (C) is wrong because an easement by necessity also arises by implication. However, it is not necessary to discuss the necessity of the easement because there was an express easement granted to the buyer. Choice (D) is incorrect because it simply states two requirements for the creation of an easement by implication. However, this choice fails to explain why the buyer has the right to go beyond the words of the express grant and lay wires under the main trail. This choice is incomplete and incorrect.
117
Q
  1. A widow was the owner in fee simple of an apartment building. The widow drafted a will by which she devised the apartment building to her granddaughter with residue to her friend and her assistant. One year later, the granddaughter died intestate and was survived by her son who was her sole heir at law.This jurisdiction has the following statute in effect:“If a devisee of a grandparent or lineal descendant of a grandparent dies at the time of execution of the will or fails to survive the testator, the issue of the deceased person shall take the decedent’s share under the will.”The widow then died and her will was admitted to probate. A dispute arose among the friend, the assistant, and the son regarding ownership of the apartment building.Is the son entitled to any share or interest in the apartment building?(A) Yes, because under the anti-lapse statute, the granddaughter’s interest passes to her son.(B) Yes, because rules relating to lapse of class gifts do not apply to specific devises.(C) No, because intestate succession is inapplicable to devolution of title to specific devisees.(D) No, because the granddaughter predeceased the widow, title to the apartment building passes to the friend and assistant under the residuary clause of the will.
A
  1. (A) On the MBE, when a statute is given in the facts, the words of that statute become very important to answering the question. In the absence of statute, the exercise of a testamentary power fails if the appointee predeceases the donor. A similar rule applies with respect to testamentary gifts. However, a majority of states have enacted anti-lapse statutes that pro vide if a devisee or legatee predeceases the testator, there is not necessarily a lapsed legacy. Designated relatives of the devisee or legatee, such as children, are entitled to take in his stead. Because this jurisdiction has a relevant anti-lapse statute in effect, the granddaughter’s interest passes to her son. Choice (B) is incorrect. This choice discusses an issue that is not relevant to these facts. There was no risk of a class gift ever lapsing here because the widow devised the apartment building to the granddaughter and the granddaughter alone. Because the anti-lapse statute will allow the estate to pass to the son (a singular person), there is no issue of a class gift. Choice (C) is incorrect because it falsely suggests that the son cannot take because the granddaughter died intestate and that the statute cannot protect a specific devisee, such as the son. However, the anti-lapse statute does not make such a distinction, and the fact that the granddaughter died intestate will not harm the son’s ability to take. Choice (D) is incorrect. Because there is an anti-lapse statute in effect, the apartment building will not pass to the friend and the assistant under the residuary clause of the will. Rather, itwill pass to the son as the granddaughter’s heir.
118
Q
  1. A farmer was the record title owner of a 30-acre tract of farmland. The farmer lived out of state and rarely visited the farmland, which remained unoccupied. Adjoining the farmland was a 50-acre ranch, which was owned by a rancher. In 2004, the rancher forged the farmer’s signature on a deed and purported to convey the farmland to a grocer. The grocer paid the rancher $100,000 as the purchase price for the farmland.The following year, the grocer sold the property to a buyer for $125,000. The deed from the grocer to the buyer was properly executed and recorded by the buyer.This jurisdiction has the following “pure race” recording statute in effect:“No conveyance or other instrument is valid as against purchasers for a valuable consideration who record first.”After the buyer took possession of the farmland, he renovated the farmhouse that was located on the property and made improvements valued at $50,000. In 2006, the farmer returned to visit his sister. During his visit, the farmer went to the farmland to inspect his farmland. While doing so, he encountered the buyer, who informed the farmer that he was the new owner of the property.The farmer thereupon instituted suit to quiet title to the farmland. After judgment was rendered in the farmer’s favor, the buyer was ejected from the property. The buyer has now filed an appropriate action against the grocer seeking restitution for the loss he incurred with respect to the purchase and improvements made on the farmland. The buyer will likely recover(A) nothing, because the grocer was also a fraud victim and suffered a substantial financial loss.(B) $25,000.(C) $50,000.(D) $125,000.
A
  1. (D) In this question, the buyer is seeking restitution. Although this question appears to involve misrepresentation or deceit, in reality it deals with mutual mistake. Because the grocer and the buyer were mutually mistaken as to validity of their land sale transaction, the conveyance may be rescinded and the buyer is entitled to recover the purchase price. When parties enter into an agreement under a misunderstanding (or under mistaken assumptions of fact), courts may rescind the contract and order restitution. In such cases, the mistake must be a serious one or go to the “essence” of the contract. Exam Tip: Here, the court is ordering rescission of the land-sale contract due to mutual mistake and simultaneously granting restitution to restore the parties to the position they were in before the agreement was entered into. Choice (A) is incorrect. Although this question appears to involve misrepresentation, fraud, or deceit, in reality it deals with mutual mistake. Because the grocer and the buyer were iiwtually mistaken as to validity of their land-sale transaction, the conveyance may be rescinded and the buyer is entitled to recover the purchase price. So, even though the grocer was also a fraud victim, his proper recourse would be against the rancher. As far as the buyer—grocer contract is concerned, mutual mistake will be allowed as a means of allowing rescission and restitution. Choice (B) is incorrect. This amount would be the amount of profit that the grocer gained in selling the property to the buyer. However, to award the buyer only $25,000 would not place him in the position he occupied before the agreement with the grocer was entered into. To allow a true rescission of the deal and give full restitution, the buyer will be awarded the entire $125,000 purchase price. Choice (C) is incorrect. This amount has no significance aside from the fact that it appears to be double the profit that the grocer gained in selling the property to the buyer. However, to award the buyer only this $50,000 would not place him in the position he occupied before the agreement with the grocer was entered into. To allow full restitution, the buyer will be awarded the entire $125,000 purchase price.
119
Q
  1. A veterinarian was the owner in fee of a 50-acre tract of farmland. The veterinarian contracted to sell her property to an investor for $300,000. The parties signed a written land-sale agreement that provided for 30 monthly installment payments of $10,000 each. According to the terms of the sale agreement, the veterinarian would deliver a warranty deed to the investor upon payment of the last installment. In accordance with state law, the farmland’s land-sale agreement was properly recorded.After making the first 10 installment payments, the investor discovered that there was an unrecorded mortgage on the farmland. The veterinarian, who is the mortgagor, has regularly made her mortgage payments and is not in default. This jurisdiction has the following recording statute in effect:“No conveyance or instrument is good as against any subsequent purchaser for value and without notice, unless the same be recorded prior to subsequent purchase.”After the investor learned of the outstanding mortgage, he discontinued making further installment payments to the veterinarian. The investor alleged that the veterinarian was in breach of the land-sale agreement due to the existence of the unrecorded mortgage.The investor sues the veterinarian for breach of contract. Ifjudgment is rendered in favor of the veterinarian, it will be because(A) the installment land-sale agreement is a security device.(B) although the land-sale agreement is actually a mortgage, it does not impair the investor’s right of redemption.(C) the prior mortgage has no legal effect on the investor’s rights under the installment land-sale agreement.(D) the time for the veterinarian, as seller, to deliver marketable title has not yet arrived.
A
  1. (D) The vendor has an affirmative obligation to convey “good” and “marketable” title to the vendee at closing. The veterinarian agreed to deliver the warranty deed to the investor upon the final payment of 30 monthly installments. Thus, the veterinarian is not under an obligation to tender “good” and “marketable” title until the date the last $10,000 installment payment is due, so the veterinarian’s own mortgage need not be resolved until the date of the last payment. Accordingly, the purchaser may not rescind a land-sale contract before the closing date or when performance is due. Choice (A) is incorrect. Certainly an installment land-sale agreement is a security device. However, this still does not explain why the investor is required to continue paying on that installment land agreement. The investor is required to continue paying because the veterinarian has not breached the land-sale agreement. Because the veterinarian is not required to tender good and marketable title until the time the Last payment is due, she has not breached the contract. Choice (0) explains that the reason the veterinarian is not in breach is because the time for performance has not yet arrived. That makes choice (D) a better answer than (A). Choice (B) is incorrect. The right of redemption allows a mortgagor in default to pay off the amount owed to the mortgagee and any interest prior to foreclosure. Because the issue here is when the veterinarian is required to pay off her own mortgage and when she is required to deliver marketable title, the investor’s right of redemption is not in issue. Therefore, this choice is incorrect. Choice (C) is wrong because the mortgage will have the legal effect of rendering title unmarketable if it is not “removed” or “cured” before the date of closing.
120
Q
  1. A landowner was the record title owner of a three- acre tract of land. In order to finance the purchase of the property in 2005, the landowner borrowed $100,000 from the bank, which secured the loan with a mortgage that amortized principal and interest payments over a 15-year period. The bank promptly recorded the mortgage. This jurisdiction has the following recording statute in effect:“Any unrecorded conveyance or mortgage is invalid as against a subsequent bonafide purchaser for value without notice who records first.”In 2006, the landowner subdivided the property into three one-acre lots. He sold lot ito his friend for $75,000. The following year, the landowner sold lot 2 to his brother for $60,000. The landowner continued to reside on lot 3. When the landowner sold lots 1 and 2 to the friend and the brother, the deeds did not make any reference to the original mortgage between the landowner and the bank.In 2009, the landowner was laid off from his job and went into default on his mortgage payments. The bank is now about to institute foreclosure proceedings.Which of the following most accurately states the rights and obligations of the parties?(A) The bank can foreclose only on lot 3 because ownership to that parcel is retained by the landowner, the original mortgagor.(B) The bank has the option of foreclosing on parcels 1, 2, or 3 because the mortgage covered the entire three-acre tract.(C) The bank must first foreclose on lot 3, and if the proceeds are insufficient, then the mortgagee may foreclose against lots 1 and 2 in the inverse order of their alienation.(D) The bank can foreclose on lot 3, but not on lots I and 2 unless the friend and the brother assumed the mortgage when they purchased their land from the landowner.
A
  1. (C) The “inverse order of alienation” rule applies when a mortgaged tract of land is “sold off” or conveyed in parcels and the various grantees pay full value to the mortgagor without getting a release from the mortgagee. Upon default, if the mortgagee forecloses, the “inverse order” rule will require the mortgagee to proceed first against the lands still owned by the mortgagor, and then proceed against the other parcels in the inverse order in which they were sold until the mortgage is fully satisfied. The rationale is that the buyer of the first parcel acquired the most equity, and likewise down the line, until the land still held by the mortgagor that has the least equity. As between the mortgagor and the grantees, the mortgagor should pay the debt and his land should be sold flst for that purpose. Law of Property, Boyer, pg. 512. If the mortgagee—the bank—forecloses, it must proceed first against lot 3, the land still owned by the landowner, the mortgagor. If the proceeds are insufficient to discharge the mortgage debt, then the mortgagee may foreclose against lots 1 and 2 in the “inverse order of the alienation” (i.e., lot 2 first, then lot 1) until the mortgage is satisfied. Choice (C) states the correct rule of law. Choice (A) is incorrect. Under the “inverse order of alienation” rule, the bank is required proceed first against the lands still owned by the mortgagor, the landowner. Then the bank can proceed against the other parcels in the inverse order in which they were sold. Because this choice states that the bank can foreclose only against lot 3, it is incorrect. Choice (B) is incorrect. Under the “inverse order of alienation” rule, the bank is required to proceed first against the lands still owned by the mortgagor, the landowner. Then the bank can proceed against the other parcels in the inverse order in which they were sold. Because this choice states that the bank has a choice as to which parcel they can foreclose on, it is incorrect. Choice (D) is incorrect. The friend’s and the brother’s deeds were silent as to the mortgage between the landowner and the bank. These two grantees took their parcels “subject to” the mortgage. Therefore, they are not personally liable. However, the bank can still foreclose on lots land 2 to satisfy the mortgage if necessary, even though the friend and the brother took “subject to” the mortgage. The bank, however, cannot sue the friend and the brother personally. Because this choice states that the bank can foreclose on the lots only if the friend and the brother assumed the mortgage, choice (D) is incorrect.
121
Q
  1. A retiree owned a 100-acre farm. For many years, the retiree grew tobacco on a 10-acre strip located in the northeast section of the property. In March, the retiree planted his annual tobacco crop, which he usually harvested in early October. In September, the retiree sold his farm to a tobacco grower for $100,000. At the time the retiree conveyed the property to the grower, the tobacco crop was well developed and quite mature. When the retiree and the grower entered into their land-sale agreement, there was no mention of the status or ownership of the tobacco crop.In early October, after the grower took possession of the property, the retiree contacted him and requested permission to harvest and remove the tobacco crop. The grower refused to allow the retiree to re-enter the property.The retiree brings suit against the grower seeking to re-enter the property and remove the tobacco crop that he had planted. Which of the following is correct regarding the respective rights of the parties?(A) The retiree is entitled to remove the tobacco crop, but he must pay the grower a reasonable value to enter the property, thus gaining access to the crop.(B) The retiree is entitled to remove the tobacco crop and is not required to pay the grower for entering the property, thus gaining access to the crop.(C) The retiree is not entitled to remove the tobacco crop and, thus, is not entitled to re-enter the property.(D) The retiree and the grower each have a colorable title to the tobacco crop, and consequently, there should be an equitable division of the proceeds from the sale of the crop between both parties.
A

121.(C) This question deals with the area of emblements, or fructus industriales. Growing crops are generally classified as personal property and will pass with a sale or mortgage of the land. Therefore, upon the retiree’s sale of his farm to the grower, the growing tobacco crop will pass as personal property, and the retiree will not be entitled to remove it. Choice (A) is incorrect. Growing crops are generally classified as personal property and will pass with a sale or mortgage of the land. Therefore, upon the retiree’s sale of his farm to the grower, the growing tobacco crop will pass as personal property, and the retiree will not be entitled to remove it because no longer belongs to him. The retiree is not entitled to enter to remove the tobacco, so his paying the grower to enter is irrelevant. Choice (B) is incorrect. Growing crops are generally classified as personal property and will pass with a sale or mortgage of the land. Therefore, upon the retiree’s sale of his farm to the grower, the growing tobacco crop will pass as personal property, and the retiree will not be entitled to remove it because no longer belongs to him. Choice (D) is incorrect. Growing crops are generally classified as personal property and will pass with a sale or mortgage of the land. Therefore, upon the retiree’s sale of his farm to the grower, the growing tobacco crop will pass as personal property, and the retiree wiLl not be entitled to remove it because no longer belongs to him. There is no need to engage in an equitabLe division of the proceeds from the sale of the tobacco crop.

122
Q
  1. An attorney, a botanist, a chemist, and a dentist own adjoining properties. All four are enthusiastic badminton players. They have decided that instead of having each person construct his own badminton court, it would make much more sense if they all could get together to build a set of courts in one place with all parties having equal rights to use them. The parties envision a court layout as appears in the diagram below:(A) (B) (C) (D)Attorney | Botanist | Chemist | Dentist———————————————————Proposed| | |Proposed Access | | | Access——————————————————— |Proposed | |Badminton Courts |——————————————————–The parties want the courts physically to occupy part of the botanist’s and part of the chemist’s land, but they want the attorney and the dentist to have equal right of access to, and use of, the courts with the botanist and the chemist. Which of the following devices would most likely accomplish the implementation of their proposed badminton court layout and be most readily acceptable to the parties?(A) A covenant against partition.(B) An indenture granting cross-easements.(C) An equitable servitude.(D) A fee simple upon condition subsequent.
A
  1. (B) An indenture granting cross easements would most likely accomplish the pLan as well as be most acceptable to all of the parties involved. An indenture is a deed to which two or more persons are parties, in which the parties enter into reciprocaL and corresponding grants or obligations to each other. The cross-easements would grant equal rights of access and use to the attorney and the dentist who own the adjoining properties [identified as (A) and (D) on the diagram]. The botanist and the chemist are owners of the servient tenement [lots (B) and (C) on the diagram]. The attorney and the dentist are owners of the dominant tenement [Lots (A) and (D)], because they are benefited by the easements. Thus, easements appurtenant are created by the indenture. Choice (A) is incorrect. A covenant is a restriction on the use of land that is enforceable at law. It is attached to or connected with the estate because it may be enforced against, or by, someone who is not one of the original parties to the covenant. However, this does not explain how the attorney and the dentist will gain access to a badminton court that is on someone else’s property. Because a covenant is not an interest in land (but rather a restriction on the use of land), it is not going to give the attorney orthe dentist the right to use the property of another. Because an easement is an interest in land, it is a better answer. Choice (C) is incorrect. An equitable servitude is a restriction on the use of land that is enforceable in equity. However, this does not explain how the attorney and the dentist wilL gain access to a badminton court that is on someone else’s property. Because an easement would explain how these two neighbors will gain access to the courts, it is a better answer. Choice (D) is incorrect. A fee interest is not necessary here when the only goal is to give the attorney and the dentist a means to get onto the badminton court on the property of another. This granting of cross-easements will allow the attorney and the dentist to go onto the land of another to access the court, even when the botanist and/or the chemist have granted their properties to another. ALl that is required here is that the attorney and the dentist be given a right to use the land of another; they do not need to be given a fee interest in the land of another just to be able to use the land. Because this answer choice goes too far in giving actual fee ownership, it is incorrect.
123
Q
  1. Jacqueline, Jessica, Jonathan, and Julianne own adjoining properties. All four are enthusiastic cat lovers. The four neighbors are frustrated at only being able to find “dog parks,” without finding a single place for their cats to play. The four neighbors decided to get together to build a fenced-in “cat park” with four fenced sides and a fenced “ceiling” where each person could bring her or his cats to play out in the open with the cats of the other neighbors. The four neighbors decide to place this cat park in the middle of all four properties with all four parties having equal rights to use the park. The parties decided to construct the park in the manner that appears in the diagram below. After the park is constructed, the parties plan to grant each other cross-easements while granting Jacqueline and Julianne (the dominant tenants) the right to enter and use the property of Jessica and Jonathan (the servient tenants).(A) (B) (C) (D)Jacqueline | Jessica | Jonathan | Julianne————————————————————-Proposed | | Proposed Access | | Access————————————————————- |Proposed “Cat Park”|————————————————————If the parties want to set up some mechanism to insure that the construction and maintenance costs of the park will be shared fairly among all of the parties, the best way to arrange this is by(A) an affirmative covenant.(B) a cooperative organization.(C) co-ownership in tenancy in common.(D) implied reciprocal servitudes.
A
  1. (A) An affirmative covenant would provide the best legal arrangement under the circumstances. Promises respecting land are referred to as covenants. Here, the covenant would require an affirmative action, thus binding the parties to share in the construction and maintenance costs of the cat park. In effect, the obligation to pay money would be utilized for purposes relating to the land, e.g., it “touches and concerns” the land. Furthermore, the payment reLates to the Land in which aLL four of the parties have an interest. Therefore, such a covenant to pay money could be enforceable at law as a co venant running with the land. There are four pre requisites for the creation of a covenant at law, as follows: (1) a writing in compliance with Statute of Frauds; (2) an intent that the covenant run with the land; (3) the covenant must “touch and concern” the land; and (4) privityof estate must exist between the covenantor and covenantee. Choice (B) is incorrect. A cooperative organization (or co-op) is a form of ownership whereby a corporation will hold title to land and/or buildings and will then lease the units to those who own shares in that corporation. No one in this fact pattern desires that there be any ownership interest in the land that the park is built upon. At most, the four neighbors simply desire that the parties promise to pay the costs of construction and maintenance of the cat park. The best way to do this would be through affirmative covenants. Choice (C) is incorrect. To create a tenancy in common would create an ownership interest in the land that the park is built upon. No one in this fact pattern desires that there be any ownership interest in the land that the park is built upon. At most, the four neighbors simply desire that the parties promise to pay the costs of construction and maintenance of the cat park and then use the cat park. The best way to do this would be through affirmative covenants. Choice (D) is incorrect. The issue of implied reciprocal servitudes arises when there is a common development scheme and one of the owners fails to comply with the covenants that the other owners must follow because those covenants do not appear in his specific deed. That is not necessary here because the four parties involved will make affirmative covenants to pay for the construction and maintenance of the cat park. Moreover, this fact pattern does not deal with conformance to a common development scheme, making choice CD) incorrect.
124
Q
  1. A citrus grower was the owner in fee of two adjacent parcels of land in a city: an orange grove and a lemon grove. The grower’s title to the lemon grove was subject to an unrecorded 20-year mortgage given to the mortgagee, a bank, in 1990 to secure repayment of a loan for $100,000.Beginning in November 1993, the following events occurred:November 1993: The grower died, leaving all of her real property to her husband. The grower’s will was admitted to probate. No mention was made of the mortgage given to the bank.December 1993: Having heard about the grower’s death, the bank recorded its mortgage to the lemon grove.August 1994: The husband executed and delivered to his sister a mortgage deed on the orange grove, which the sister immediately recorded. The mortgage instrument contained the following recitations: “This mortgage is secured by the orange grove and all other real estate that I may own in the city or have an interest in.” The husband defaulted on his mortgage obligation to the sister; the amount due on the debt was $100,000.February 2009: The bank brought suit against the husband to foreclose its mortgage on the lemon grove.The applicable recording statute provides in part:“No deed or other instrument in writing, not recorded in accordance with this statute, shall affect the title or rights to, in any real estate, of any devisee or purchaser in good faith, without knowledge of the existence of such unrecorded instruments.”Judgment should be for(A) the husband, if he was unaware of the existence of the mortgage at the time he acquired title to the lemon grove.(B) the husband, only if the lemon grove was not subject to the mortgage when he acquired title.(C) the bank, because the husband assumed the mortgage when he acquired title to the lemon grove.(D) the bank, because property once mortgaged remains mortgaged as against the mortgagor’s successors in interest.
A
  1. (A) When the grower died and her will was admitted to probate in November 1993, the bank’s mortgage on the lemon grove had not yet been recorded. Therefore, the husband’s fee simple title to the lemon grove would be unaffected by the prior unrecorded mortgage (in accordance with the relevant recording statute in effect). So, if the husband was fully unaware of the existence of the mortgage and had no actual notice of it, then he would prevail because there was no constructive notice of this mortgage when he acquired title. Choice (B) is incorrect because it falsely suggests that the only way that the husband can take the lemon grove free of the grower’s mortgage is if there were simply no mortgage on the property when he acquired title. Where the language of conveyance is silent, the grantee is considered to take “subject to” the mortgage. This means that the grantee is not personally liable for the mortgage debt. The husband can acquire the lemon grove free of any mortgage because it was not recorded early enough, and this is true even if there was a mortgage on it when he acquired it. Choice (C) is incorrect. Where the language of conveyance is silent, the grantee is considered to take “subject to” the mortgage. This means that the grantee is not personally liable for the mortgage debt. Becacise the husband never “assumed” the mortgage, this choice is incorrect. Choice (D) is incorrect. All this choice does is state that the mortgage would continue to bind the property for its duration, and that principle might serve to bind the successors in interest if they “assumed” the mortgage debt. However, the husband never assumed the mortgage, and the mortgage was never recorded anyway, so the husband is not personally liable.
125
Q
  1. A developer, owner of several hundred acres in a rural part of the county, drafted a general development plan for the area. The duly recorded plan imposed elaborate limitations and restrictions upon the land in the plan, which was to be developed as a residential district. The restrictions were to extend to all persons acquiring any of the lots and to their heirs, assigns, and lessees. It was further provided that all subsequent owners would be charged with due notice of the restrictions.Among those restrictions in the general plan were the following:(22) A franchise right is created in a strip of land 10 feet in width along the rear of each lot for the use of public utility companies with right of ingress and egress.(23) No house or structure of any kind shall be built on the aforementioned strip of land running through the said blocks.The franchise right created for public utilitycompanies would most likely be an example of a(an)(A) license.(B) equitable servitude.(C) easement appurtenant.(D) easement in gross.
A
  1. (D) The franchise right would be construed as an easement in gross. This non-possessory interest is created when the holder of the easement interest acquires his right of special use in the servient tenement independent of his ownership or possession of another tract of land. In an easement in gross, the easement holder is not benefited in his use and enjoyment of a possessory estate (i.e., there is no dominant tenement) by virtue of the acquisition of that privilege. Choice (A) is incorrect because a license is a revocable privilege to enter upon the land of another. The grant from the developer to the utility companies went beyond a mere privilege to enter; rather, this was an intent to grant a strip of land to be used for utility purposes. Choice (B) is incorrect because an equitable servitude is a restriction on the use to which an owner may make of his land. Choice (C) is wrong because the utility company did not possess an adjoining dominant tenement.
126
Q
  1. A developer, owner of several hundred acres in a rural county, drafted a general development plan for the area. The duly recorded plan imposed elaborate limitations and restrictions upon the land in the plan, which was to be developed as a residential district. The restrictions were to extend to all persons acquiring any of the lots and to their heirs, assigns, and lessees. It was further provided that all subsequent owners would be charged with due notice of the restrictions.Among those restrictions in the general plan were the following:(22) A franchise right is created in a strip of land 10 feet in width along the rear of each lot for the use of public utility companies with right of ingress and egress.(23) No house or structure of any kind shall be built on the aforementioned strip of land running through the said blocks.The court will most likely construe restriction (23) as a (an)(A) negative easement.(B) equitable servitude.(C) affirmative covenant.(D) fee simple absolute.
A
  1. (B) The proviso prohibiting lot owners to construct a house or structure on the strip of land would properly be construed as an equitable servitude. An equitable servitude is a restriction on the use to which an owner may make of his land. It is created (1) by a writing complying with the Statute of Frauds; (2) concerning a promise that “touches and concerns” the land; and (3) by indicating an intention that the servitude exists. The basis for enforcing an equitable servitude is that one who takes land with notice (actual or constructive) of a restriction on the land cannot in equity be allowed to violate that restriction. Choice (A) is incorrect. An easement is the right of one person to go onto the land in possession of another and make a limited use thereof. Restriction 23 is a restriction on the use of land, not the right to use the land of another. Because the restriction, by itself, does not create an easement, choice (A) is incorrect. Choice (C) is wrong because the lot owners are under a negative, not affirmative, obligation to refrain from constructing on the said strip. Choice (D) is incorrect because it is inapplicable inasmuch as Restriction (23) is a restriction on the use of land, not a fee interest in it.
127
Q
  1. On October 1, 1980, a developer, owner of several hundred acres in a rural county, drafted a general development plan for the area. The duly recorded plan imposed elaborate limitations and restrictions upon the land in the plan, which was to be developed as a residential district. The restrictions were to extend to all persons acquiring any of the lots and to their heirs, assigns, and lessees. It was further provided that all subsequent owners would be charged with due notice of the restrictions.Among those restrictions in the general plan were the following:(22) A franchise right is created in a strip of land 10 feet in width along the rear of each lot for the use of public utility companies with right of ingress and egress.(23) No house or structure of any kind shall be built on the aforementioned strip of land running through the said blocks.In 2000, a retiree purchased one of the lots, built a house, and erected a fence in the rear of his property within the restricted area. In 2004, a teacher purchased a lot adjacent to the retiree’s property and built a new house. Two years later, a librarian purchased the lot that adjoined the teacher’s property. The three deeds to those properties each contained references to the deed book where the general plan was recorded.In 2008, the librarian began the construction of a seven-foot post-and-rail fence along the line dividing his lot with the teacher’s, and along the center of the area subject to the franchise right. Although the teacher objected to its construction, the fence was completed. If the teacher seeks a mandatory injunction to compel removal of the librarian’s fence, the court will most likely(A) grant relief, because the fence was in violation of the easement restriction.(B) grant relief, because the encroachment of the fence violated the restriction in the original plan.(C) deny relief, because the teacher failed to enforce the restriction against the retiree.(D) deny relief, because the fence would not be construed as “a structure” within the terms of the restriction.
A
  1. (B) Because the encroachment of the fence violated the restriction (i.e., equitable servitude), the teacher would be entitled to injunctive relief. Because the plan was recorded and because the librarian is a grantee of the developer, the librarian will have notice of restrictions in the chain of title. Therefore, the teacher can compel the librarian to remove the fence because it violates Restriction (23). Choice (A) is incorrect because the restriction would be construed as an equitable servitude, rather than an easement. Choice (C) is incorrect. Choice (C) falsely suggests that the teacher must also sue the retiree just to be able to seek relief against the librarian. As long as the librarian has violated the covenant, the teacher can enjoin the violation by seeking a mandatory injunction. The fact that the teacher fails to seek an injunction against the retiree (for the retiree’s own fence) is irrelevant to his right to seek an injunction against the librarian. Choice (D) is incorrect. Clause (23) uses the language “of any kind” to suggest that there is no set list of things that would trigger the restriction if they were built. A fence would qualify as a structure “of any kind” because the intent was to grant an easement for utilities along that strip and to keep that area free of obstructions regardless of size or substance.
128
Q
  1. An orange grove and a grapefruit grove are two parcels abutting each other. A citrus grower owns the orange grove in fee simple and maintains both his dwelling house and the business he operates on the orange grove. He has a right of way, granted by a written agreement, across the grapefruit grove for crossing the grapefruit grove on foot, by bicycle, or automobile. A farmer is the owner of the grapefruit grove.The citrus grower’s property interest in the use of his right of way across the grapefruit grove may best be described as a (an)(A) license.(B) easement in gross.(C) easement appurtenant.(D) prescriptive easement.
A
  1. (C) An easement is deemed appurtenant when the right of special use benefits the holder of the easement in his physical use or enjoyment of another tract of land. For an easement appurtenant to exist, there must be two tracts of land; the dominant tenement (which has the benefit of the easement), and the servient tenement (which is subjectto the easement right). The citrus grower’s property interest in the use of his right-of-way across the grapefruit grove would be construed as an easement appurtenant. Choice (A) is incorrect because a license is a revocable privilege to enter upon the Lands of the licensor. The call of the question asks what the citrus grower’s property interest is; a License is not a property interest. Choice (B) is incorrect. An easement is in gross when it is intended to benefit the holder of easement personally, rather than in connection with any land the holder owns. In otherwords, every easement in gross requires only one piece of land (i.e., the servient tenement) that is owned by a person other than the owner of the easement in gross. This easement enhances the enjoyment of the orange grove because it allows for a method of ingress and egress to the orange grove. Thus, the easement is appurtenant, not in gross. Choice (D) is incorrect. An easement may be acquired by prescription in a manner similar to that by which ownership of a possessory estate may be acquired by adverse possession. However, the farmer granted the citrus grower an express easement by written agreement, so it is not necessary to evaluate the elements required for a prescriptive easement.
129
Q
  1. A western parcel, a central parcel, and an eastern parcel are three business lots abutting each other, with the central parcel between the other two lots. A business owner owns the central parcel in fee simple and maintains his dwelling house thereon. He has a right-of-way, granted in a written agreement, across the eastern parcel for crossing the eastern parcel on foot, by bicycle, or automobile. A landscaper is the owner of the eastern parcel. An investor owns the western parcel and conveys it to the business owner in fee simple. The business owner then builds a 15-story office building, covering the western parcel and the central parcel, which houses 6,000 persons during working hours each day. All of these persons use the right-of-way over the eastern parcel, and the business owner uses the way for delivering as many as 30 loads of supplies per day to the office building.In an action by the landscaper to enjoin the business owner and the office workers from using the right- of-way across the eastern parcel, the court will most likely hold that(A) the business owner’s right-of-way would be extinguished due to excessive use by the office workers.(B) the business owner’s right-of-way would be forfeited due to the unauthorized use by the office workers.(C) the business owner, by making use of the right-of-way beyond the scope of the original privilege, would permanently be enjoined from using the servient tenement.(D) Although the business owner may continue to use the right-of-way, the office workers would be enjoined from making such use.
A
  1. (D) Mere excessive use of an easement does not forfeit or extinguish the easement. In the case of excessive use, the owner of the easement is simply making use of the servient tenement beyond the scope or extent of the use permitted by the easement in its creation. The easement (as it was originally created) still exists and within its scope can be used. However, the user in excess of the scope of the easement can be enjoined and damages may be had for injury caused by the excess user. Thus, although the business owner may continue to use the right of way, the office workers’ use of the right of way is not merely excessive, it is wholly unauthorized. Choice (A) is incorrect. The landscaper can obtain an injunction to stop the excessive and unauthorized use of the easement; but excessive use will not be enough to extinguish an easement. At most, the landscaper can stop the usage that is in excess of what he originally granted to the business owner. Choice (B) is incorrect. The landscaper can obtain an injunction to stop the excessive and unauthorized use of the easement; but excessive use will not be enough to create a forfeiture of the easement. At most, the landscaper can stop the usage that is in excess of what he originally granted to the business owner. Choice (C) is incorrect. The Landscaper can obtain an injunction to stop the excessive and unauthorized use of the easement; but excessive use will not be enough to cause the business owner from being permanently enjoined from using the easement, because the fact remains that the Landscaper gave the business owner an express easement for ingress and egress across the eastern parcel and to make reasonable use of that easement. At most, the landscaper can stop the usage that is in excess of what he originally granted to the business owner.
130
Q
  1. A wedding planner owned a summer cottage on the lake. In order to facilitate the access to the cottage, which is located on a knoll above the lake, the wedding planner entered into an agreement with a neighbor, an adjoining land owner, whereby the neighbor, in writing, granted the wedding planner a right-of-way over a strip of land 30 feet in width and a quarter of a mile in length along the eastern margin of the neighbor’s property. Without notif’ing the neighbor, the wedding planner proceeded with his plan to improve the roadbed by having the road asphalted in order to make it more accessible for motor vehicle traffic. Several years later, the neighbor started a sand business, which required him to do heavy hauling that subsequently destroyed the asphalted surface on the road.Ten years after the neighbor started his sand business, the wedding planner sold his lakefront property to an artist. Shortly after the artist took possession of the cottage and property, the neighbor erected wooden barriers across the roadway, thus obstructing the access to the artist’s property.The interest that the wedding planner acquired in the access road may best be described as(A) an easement in gross.(B) an easement implied by prior use.(C) an easement by necessity.(D) an express easement.
A
  1. (D) Choice (D) is correct because an express easement appurtenant was created by the written agreement between the neighbor (owner of the servient tenement) and the wedding planner (owner of the dominant tenement). This granted the wedding planner the right to use the strip of land across the neighbor’s property for ingress and egress. Choice (A) is incorrect because an easement in gross is personal to the owner and the use is not connected in any way with the enjoyment of any other land. Choice (B) is incorrect. Choice (B) is an implied easement viewed by the courts as arising by reason of prior use. It is not necessary to evaluate the requirements for an easement by implication because the easement was given by express grant. Choice (C) is incorrect because it is an implied easement viewed by the courts as arising by reason of necessity. It is not necessary to evaluate the requirements for an easement by necessity since the easement was given by express grant.
131
Q
  1. A caterer owned a summer cottage on the lake. In order to facilitate the access to the cottage, which is located on a knoll above the lake, the caterer entered into an agreement with a neighbor, an adjoining land owner, whereby the neighbor, in writing, granted the caterer a right-of-way over a strip of land 30 feet in width and a quarter of a mile in length along the eastern margin of the neighbor’s property. Without notifying the neighbor, the caterer proceeded with his plan to improve the roadbed by having the road asphalted in order to make it more accessible for motor vehicle traffic. Several years later, the neighbor started a sand business, which required him to do heavy hauling that subsequently destroyed the asphalted surface on the road.In an action by the caterer to enjoin the neighbor’s use of the road in transporting sand, the court will most likely(A) issue an injunction against the neighbor to prevent the further use of the road to haul sand.(B) refuse to grant the caterer’s prayer for relief because the servient owner continues to have the right to use his own land.(C) issue an injunction against the neighbor unless it can be shown that the neighbor’s use did not unreasonably interfere with the caterer’s right of access.(D) dismiss the cause of action because the caterer’s only remedy would be monetary damages.
A
  1. (C) Choice (C) is correct because although the owner of the servient tenement remains the owner of the land subject to the easement he has granted, he may only make such use of the land as long as he does not unduly or unreasonably interfere with the rights created for the dominant owner. Choice (A) is incorrect. This choice goes too far in restricting the neighbor’s use of his own land. The neighbor owns the servient tenement and will not be completely enjoined from using the road at all because that would restrict the neighbor too severely. ALL that is required here is that the neighbor not behave in such a way as to damage or restrict usage of the easement that he granted to the caterer. Because this choice enjoins all use by the neighbor, it is incorrect. Choice (B) is incorrect. Although the owner of the servient tenement remains the owner of the land subject to the easement he has granted, he may only make such use of the land as long as he does not unduLy or unreasonably interfere with the rights created for the dominant owner. However, if the use by the servient tenant does unduly interfere with the rights of the dominant owner, then that use can be enjoined. Because this choice falsely suggests that the servient owner can do what s/he wants to the exclusion of the rights of the dominant tenant, it is incorrect. Choice (D) is incorrect. The caterer, as an easement holder, has rights in this strip of land. One of those rights would be the right to enjoin those uses that interfere with his access to that easement. There is no requirement that the caterer only has a remedy of money damages. The caterer has a right to enjoin those uses that interfere with his use because money damages may not be what the caterer is seeking.
132
Q
  1. A chef owned a summer cottage on the lake. In order to facilitate the access to the cottage, the chef entered into an agreement with a neighbor, an adjoining land owner, whereby the neighbor, in writing, granted the chef a right-of-way over a strip of land 30 feet in width and a quarter of a mile in length along the eastern margin of the neighbor’s property. Without notifying the neighbor, the chef proceeded to improve the roadbed by having the road asphalted in order to make it more accessible for motor vehicle traffic. Several years later, the neighbor started a sand business, which required him to do heavy hauling that subsequently destroyed the asphalted surface on the road.Ten years after the neighbor started his sand business, the chef sold his lakefront property to his assistant. Shortly after the assistant took possession of the cottage and property, the neighbor erected wooden barriers across the roadway, thus obstructing the access to the assistant’s property.The assistant’s strongest argument in an action against the neighbor for removal of the barriers, which are obstructing his access to the property, would be that(A) an easement appurtenant is alienable in that any conveyance transferring possession of the dominant tenement also passes the easement privilege.(B) the assistant, as a bonafide purchaser, is protected from the neighbor’s obstruction, even though the easement was unrecorded.(C) because the easement was created by implication, the chef’s prior and continuous use gave rise to a prescriptive easement.(D) the chef and the assistant, being in privity of estate, could “tack on” their successive use periods.
A
  1. (A) Choice (A) is the correct choice because today the statutory presumption is in favor of creating easements appurtenant for perpetual duration unless the grant specifically limits the interest. On the other hand, one should note that an easement in gross is generally inalienable. Choice (B) is incorrect because it incorrectly focuses on recording statutes and chain-of-title problems. The concept of bona fide purchaser (BFP) comes up when the applicable recording statute will protect a subsequent BFP who takes without notice of prior conveyances. This concept is relevant when competing grantees are vying for the same piece of land. However, there is no such competition taking place here between competing grantees. The assistant has a property interest in the easement because he is a grantee of the chef, the original dominant tenant. This choice provides the assistant with a very weak argument because his status of BFP is irrelevant to helping him explain why he owns an easement across the neighbor’s property, even though he was not an original party to the original grant of easement. Choice (A) will explain why the assistant still has a right to cross the neighbor’s property and why he has the right to enjoin further use of barriers across the road. Choice (C) is incorrect. The easement was originally created by an express grant. This choice is not accurate on the facts. Choice (D) is incorrect. The concept of tacking arises in adverse possession situations where the current adverse possessor will “tack on” and use a prior adverse possessor’s time to meet the statutory period. Not only was no one adversely possessing the property, there was an express grant of easement, so there is no need to consider how long the easement was being used by each party.
133
Q
  1. A sculptor owned a summer cottage on a small lake. In order to facilitate the access to the cottage, the sculptor entered into an agreement with a neighbor, an adjoining land owner, whereby the neighbor, in writing, granted the sculptor a right-of-way over a strip of land 30 feet in width and a quarter of a mile in length along the eastern margin of the neighbor’s property.After using the roadway for a year, the sculptor and his family moved to Europe for business reasons. The sculptor and his family have not used the cottage for 11 years because of his employment commitment in Europe. During the sculptor’s absence, the neighbor constructed an access ramp to his dock, which obstructed the road to the sculptor’s cottage. Upon the sculptor’s return from Europe, he goes to his lakefront cottage and discovers the obstructing ramp.The sculptor initiates suit against the neighbor to compel him to remove the ramp. The court would most likely(A) hold in the neighbor’s favor, because the sculptor’s absence for 11 years constituted an abandonment of the easement.(B) hold in the neighbor’s favor, because the dominant owner is under an affirmative duty to notifS’ the servient tenement holder of non-use.(C) hold in the sculptor’s favor, because mere non- use, however long continued, will not as a rule effectuate an abandonment.(D) hold in the sculptor’s favor, because of the validity of the agreement for the express grant of the easement.
A
  1. (C) Choice (C) is correct because mere non-use, however long continued, will notterminate an express easement. In addition, there must be evidence of some affirmative or overt act on the part of the easement holder, i.e., dominant owner manifesting his intent to abandon the easement. Choice (A) is incorrect. Non-use of an easement, no matter how long continued, will not be sufficient to terminate an easement. However, non-use coupled with an intentto abandon is sufficient to constitute an abandonment. The sculptor’s absence for 11 years will not, alone, be enough to constitute an abandonment of the easement. Choice (B) is incorrect because it falsely suggests that the neighbor will prevail because the sculptor failed to notify the neighbor of his non-use (due to an affirmative duty to notify that the easement will not be used). However, non-use alone will still not be enough to constitute an abandonment anyway. So, even if the sculptor had notified the neighborthat he will not be using the easement due to business reasons in Europe, that will still not be enough to constitute an abandonment, and the neighbor will not prevail. Choice (D) is incorrect because it focuses on the creation of the easement. Certainly there is a valid agreement for an express easement. There was never any doubt about that. However, this choice does not address why the sculptor still retains his interest in the easement despite an 11-year absence. The reason the sculptor still has an easement (and the right to compel the neighbor to remove the ramp) is because his non- use was not enough to constitute an abandonment. Because choice (C) addresses this reason, it is a better choice.
134
Q
  1. A musician owned a summer cottage on the lake. A neighbor, an adjoining land owner, started a sand business, which required him to do heavy hauling of sand, rocks, dirt, and other materials on his property.The neighbor’s excessive excavating in the course of conducting his sand removal business causes the collapse of a large storage building on the musician’s property. If the musician brings an action to recover damages for the collapse of his storage building, the musician will probably(A) be successful if he can prove that the neighbor was negligent in his excavations.(B) be successful, because the neighbor would be strictly liable for his removal of lateral support.(C) not be successful, because the neighbor could not be held liable for damage to an artificial structure such as a storage building.(D) not be successful, because an adjacent land owner is under no affirmative duty to laterally support his neighbor’s land.
A
  1. (A) If there is negligence on the part of the wrongdoer who removes lateral support, then the wrongdoer is liable for all damages that naturally and proximately flow from his negligence, including damages to both land and artificial structures. Choice (B) is incorrect. One who by excavation or otherwise withdraws lateral support from his neighbor’s land is absolutely and strictlyliable (regardless of negligence) for land that is in its natural condition. When land has artificial structures on it and those structures are damaged due to a removal of lateral support, the plaintiff must then show negligence on the part of the defendant. Choice (C) is incorrect because it is incomplete. The neighbor could be liable for damage to an artificial structure if there were negligence on his part. The way choice (C) is written forecloses any possibility of there being any liability. Because liability could possibly attach to the neighbor’s activities if negligence were found to exist, this choice is incorrect. Choice (D) is incorrect. An adjacent landowner has a duty not to remove lateral support to the extent that strict liability is created if removal causes damage to land in its natural condition. If there is negligence on the part of the wrongdoer who removes lateral support, then the wrongdoer is liable for all damages that naturally and proximately flow from his negligence, including damages to both land and artificial structures. Because this choice states that no duty is owed, it is incorrect.
135
Q
  1. On March 1, 1999 a widower, the sole owner and occupant of a piece of property, died and devised the property to both his co-worker and his boss “as their community property.” The co-worker and the boss were also siblings, and neither was married. The property consisted of a single-family house with a yard, garage, and driveway.The boss died intestate on May 1, 2001, leaving her daughter as her sole heir. During her occupancy, the boss paid $3,500 each year in insurance and property taxes. In addition, the premises on the property had a fair market value during this period of $1,500 each year. Following her mother’s death, the boss’s daughter moved into the house on May 2, 2001. As the administratrix of the boss’s estate, the daughter sought to collect from the co-worker one- half of the cost of insurance and property taxes that the boss paid.Conversely, the co-worker claimed that he was not liable for any of the expenses. Furthermore, the coworker’s attorney advised him that the daughter did not own any interest in the property and that since the boss’s death, he (the co-worker) owned the entire property. The attorney also informed the co-worker that the daughter owed him rent for the entire period of her occupancy, and if she continued to occupy the premises, then she would be liable for insurance and property taxes as well.In an appropriate action to determine the title of the property, what, if any, interest does the daughter have in the said property?(A) None, because the co-worker acquired title to the whole of the property by right of survivorship.(B) An undivided one-half interest, because upon the death intestate of a tenant-in-common, the latter’s interest descends to his heirs.(C) An undivided one-half interest, because in a tenancy by the entirety, either sibling can make a testamentary disposition.(D) An undivided whole interest, because in a joint tenancy, every joint tenant is part and parcel of the unit group that owns the whole.
A
  1. (B) A tenancy in common was created by the widower’s testamentary devise to the co-worker and the boss in spite of the language “as their community property” because they were siblings, not husband and wife. Upon the death intestate of a tenant in common, his interest descends to his heirs. The daughter (as the boss’s sole heir) would have an undivided one-half interest in the property. Choice (A) is incorrect. The widower did not convey the property to the co-worker and the boss as joint tenancy with right of survivorship. Joint tenancies are disfavored and require a clear expression of intent to create this estate; otherwise it will not be recognized and there will be tenancy in common. Because this was at most a tenancy in common, there was no right of survivorship and the co-worker will not take the whole of the property under a right of survivorship theory. Choice (C) is incorrect. A tenancy by the entirety requires the unity of husband and wife in order to be effective. Because the co-worker and the boss were siblings, there can be no tenancy by the entirety. Choice (D) is incorrect. The widower did not convey the property to the coworker and the boss as joint tenancy with right of survivorship. Joint tenancies are disfavored and require a clear expression of intent to create this estate; otherwise it will not be recognized and there will be tenancy in common. It is becaus.e this was a tenancy in common that the boss’s interest passed to the daughter. This choice is incorrect because it states that the daughter received an undivided whole interest because it was a joint tenancy. If anything, the opposite result would be reached; if the property were truly owned by the co-worker and the boss as joint tenants, the daughter would receive nothing upon the boss’s death because of the co-worker’s right of survivorship. For these reasons, this choice is incorrect.
136
Q
  1. | Driveway————————————————————–House | House |————————————————————– | Garage | House————————————————————–LOT3 | LOT2 | LOT1 ————————————————————–(TEACHER) | (NEIGHBOR) | (CO-WORKER | | & BOSS)————————————————————-On March 1, 1999, a landowner, the sole owner and occupant of lot 1, died and devised lot ito both his co-worker and his boss “as their community property.” The co-worker and boss were siblings, and neither was married. Lot 1 consisted of a single- family house with a yard, garage, and driveway.On May 1, 1999, the boss moved into the house on lot 1. One year later, the co-worker and the boss executed and delivered the following deed instrument to a neighbor “… hereby grant to (the neighbor) the northerly 30 feet of lot 1, consisting of the paved driveway now existing, to be used for the ingress and egress of motor vehicles, but should (the neighbor) or his heirs and assigns use said property for any other purpose, all the rights, privileges, and immunities herein granted shall cease and determine.” In consideration for the said deed, the neighbor paid the co-worker and the boss $2,000 (which they divided equally). The deed was never recorded by the neighbor. Because the boss didn’t own a car, she never used the driveway. Similarly, the neighbor never used the driveway because he unexpectedly had his driver’s license suspended shortly after executing the above instrument.The boss died intestate on May 1, 2001, leaving her daughter as her sole heir. Following her mother’s death, the daughter moved into the house on May 2, 2001. On June 1, 2001 the neighbor sold lot 2 to a professor by a deed that contained no mention of the driveway located on lot 1. The neighbor and the professor assumed that the latter had the right to use the driveway, so they didn’t insert any recitations in their deed instrument regarding the driveway. Immediately upon her taking possession of the premises, the daughter began to use the driveway on lot 1. Consequently, she objected to the professor’s use of the driveway.After the daughter refused to permit the professor to use the driveway, he brought suit to determine his right to continue use of the driveway. The professor should(A) win, because he acquired an implied easement to use the driveway as owner of the dominant tenement.(B) win, because the neighbor’s easement to use the driveway was conveyed to the professor.(C) lose, because the Statute of Frauds was not satisfied.(D) lose, because the neighbor’s non-use of the driveway effectuated an abandonment of the easement.
A
  1. (B) Choice (B) is the best alternative because a conveyance of the dominant tenement (lot 2) carries with it all easements and profits appurtenant thereto as incidents unless it is otherwise expressly provided. When the neighbor conveyed to the professor, the professor received not only the estate of lot 2, but also the easement appurtenant to it that benefits Lot 2. Choice (A) is incorrect because an easement by implication arises from the circumstances surrounding the dividing by the owner of a piece of land into two pieces and conveying one of the pieces to another. From such surrounding circumstances, an inference is drawn that the parties intend the creation of an easement. In the present case, the co-worker and the boss created an express easement in the deed instrument permitting the neighbor to use the driveway. Choice (C) is wrong because the written deed instrument does satisfy the Statute of Frauds. Choice (D) is incorrect because mere non-use does not constitute an abandonment of the easement.
137
Q
  1. In 1960, a widower, advancing into old age, realizes that he is no longer able to farm his 1,000 acres; therefore, he decides to sell some of the farmland in parcels of 250 acres. The president of a development company is interested in purchasing three of the parcels. The president buys the three parcels from the widower and begins formulating plans for constructing single-family dwelling units on the land, which is located in an upper-middle-class area of the county. The original deed between thewidower and the development company contains a provision expressly binding “upon all subsequent grantees, their heirs, and assigns,” stipulating that any further subdivisions by any such persons shall be restricted to minimum two-acre lots to be used for single-family dwelling units only.In the original deed between the widower and the development company, the stipulation that restricted the size and residential character of any subsequent subdivision of the parcels is an example of a (an)a. easement.b. affirmative covenant. c. covenant for quiet enjoyment. d. negative covenant.
A
  1. (D) The original deed provision between the widower and the president restricting subdivision of the parcels to minimum two-acre lots for residential use is an example of a restrictive, or negative, covenant running with the land. In order for a covenant to run with the Land, the following four requirements must be met: (1) the covenant must be in writing; (2) it must “touch and concern” the land; (3) privity of estate must exist between covenantor and covenantee; and (4) there must be intent that the covenant run with the land. Choice (A) is incorrect. An easement is the right of one person to go onto the land in possession of another and make a limited use thereof. It is an interest in the land itself. The stipuLation restricting the size and character is a covenant or a restriction on the use of land and not an interest in the land itself. Choice (B) is incorrect because an affirmative covenant imposes an obligation to perform a duty on the part of the covenantee, i.e., obligation of condominium owner to pay an annual fee for maintenance of common areas. Because this covenant restricts the use of the land, it is more appropriately termed a negative covenant. Choice (C) is incorrect. The covenant of quiet enjoyment is a deed covenant by the grantor that the grantee will not be disturbed in his possession or enjoyment of the property by a third party’s lawful claim of title. Ths issue is inapplicable to these facts.
138
Q
  1. A retiree, advancing in age, realizes that he is no longer able to farm his 1,000 acres and therefore decides to sell some of the farmland in parcels of 250 acres. The president of a development company is interested in purchasing three of the four parcels. The president buys the three parcels from the retiree and begins formulating plans for constructing single- family dwelling units. The original deed between the retiree and the development company contains a provision expressly binding “upon all subsequent grantees, their heirs, and assigns,” stipulating that any further subdivisions by any such persons shall be restricted to minimum two-acre lots to be used for single-family dwelling units only.The development company immediately subdivided two of the parcels into lots of three, four, and five acres, and began construction of homes thereon. The original deed restrictions were enumerated within the special warranty deeds and were given to the purchasers of the homes in the new development, called tract I.Two years later, the president sold the remaining 250-acre parcel, which had not been included in the tract 1 subdivision plan, to a contractor. The deed between the president and the contractor included the same restriction as was in the original deed between the retiree and the president. The contractor, in turn, drafted a subdivision plan for the last 250-acre parcel, dividing it into one-acre lots. The contractor then commenced construction of single- family dwelling units in the new development, to be known as tract 2. There was no mention of the restriction for two-acre minimum lots in the deeds to the purchasers of the new homes in tract 2.In a subsequent action to enjoin the contractor from subdividing the parcel into one-acre lots by any of the present owners of lots in the tract 1 development, the most probable judicial determination would be(A) that the action should be dismissed because the owners lack standing to sue.(B) that the action should be dismissed because there is no privity of estate between the owners of the lots in tract 1 and the contractor.(C) that the action would be successful if tract 2 were considered by the court as part of a common development scheme.(D) that the action would be successful because the restrictions in the original deed between the retiree and the president will be enforceable.
A
  1. (D) Choice (D) is correct because a covenant running with the land will be enforceable against successors of the original parties, where, again, the following four requirements are met: (1) there is privity of estate between the original parties and their successors; (2) the covenant “touches and concerns” the land; (3) the parties intend the covenant to run with the land; and (4) the covenant is in writing. Thus, the subsequent grantees, i.e., owners of tract 1 lots, may enforce the original restrictive covenant between the retiree and the president. Choice (A) is incorrect. A covenant running with the land will be enforceable against successors of the original parties, where, again, the following four requirements are met: (1) there is privity of estate between the original parties and their successors; (2) the covenant “touches and concerns” the land; (3) the parties intend the covenant to run with the land; and (4) the covenant is in writing. Thus, the subsequent grantees, i.e., owners of tract 1 lots, may enforce the original restrictive covenant between the retiree and the president. Because the current owners will have standing to sue, even though they were not the original covenanting parties, this answer choice is incorrect. Choice (B) is incorrect. A covenant running with the land will be enforceable against successors of the original parties. The covenant made by the grantor (the retiree) and grantee (the contractor) will be binding upon successive grantees of the original covenanting parties. The action will be successful and this choice is incorrect. Choice (C) is incorrect. The issue to be resolved in this question is whether or not grantees in the tract 2 development can be bound by the restrictive covenant even though the deeds they received made no mention of it. The restriction can be enforced because the covenant runs with the land and can bind remote grantees of the original covenanting parties. The issue of common development scheme is not the central issue here because it seems clear from the original deed from the retiree to the president that all 750 acres were to have this common feature of minimum lot sizes. This question falsely suggests that the issue of a common development scheme is the only issue that a court would have to decide. However, the enforceability of the restrictions is also in issue, and it is the issue that is most in need of being decided.
139
Q
  1. In 1998, a farmer, advancing in age, realizes that he is no longer able to farm his 1,000 acres and therefore decides to sell some of the farmland in parcels of 250 acres. The president of a development company is interested in purchasing three of the four parcels. The president buys the three parcels from the farmer and begins formulating plans for constructing single-family dwelling units on the land. The original deed between the farmer and the development company contains a provision expressly binding “upon all subsequent grantees, their heirs, and assigns,” stipulating that any further subdivisions by any such persons shall be restricted to minimum two-acre lots to be used for single- family dwelling units only.The development company immediately subdivided two of the parcels into lots of three, four, and five acres, and began construction of homes thereon. The original deed restrictions were enumerated within the special warranty deeds and were given to the purchasers of the homes in the new development, called phase 1.Two years later, the president sold the remaining parcel, which had not been included in the phase 1 subdivision plan, to a contractor. The contractor, in turn, drafted a subdivision plan for the last 250-acre parcel, dividing it into one-acre lots. The contractor then commenced construction of single-family dwelling units in the new development, to be known as phase 2. There was no mention of the restriction for two-acre minimum lots in the deeds to the purchasers of the new homes in phase 2.Meanwhile, after the farmer’s death, his estate isrequired to sell the remaining 250-acre parcel of his farmland. The buyer is an investor, who proposes to construct a two-level shopping center and parking lot on the property.The area surrounding phase 1 and phase 2 was rezoned for commercial and industrial uses in 2010. The investor’s shopping center has grown to include 150 stores. Now, one of the lot owners in phase 1 contracts to sell his property to two physicians who plan to start a suburban medical practice.In an action by the homeowners in phase 1 to prevent such commercial use by the physicians, the court will most likely hold that(A) the restrictions are still enforceable, thus preventing such commercial use.(B) the restrictions would no longer be enforceable, because the offering of personal services (i.e., medical) would be a conforming use.(C) the restrictions would no longer be enforceable, because of the change in the character of the neighborhood.(D) the restrictions would no longer be enforceable, because the opening of a physician’s office in a private home would not be construed as a commercial enterprise.
A
  1. (A) Choice (A) is correct because the change in the character of the neighborhood was not of such a magnitude as to render the restriction unenforceable. As a result, the restrictions must continue to be enforced. Choice (B) is incorrect. The restriction was to single-family dwelling units only. This medical practice will likely not be considered by the court to be a conforming use. Choice (C) is incorrect. While it is true that the area has been rezoned, those zoning ordinances will not serve to invalidate the covenants unless neighborhood conditions have changed sufficiently to make the servitude meaningless. However, the facts only state that the area has been rezoned. The facts do not say that the whole area immediately surrounding phase 2 and phase I has changed so much and to such an extent that the restriction renders the property unusable. Moreover, a change in the character of the neighborhood will never be enough to cause the restrictions to be unenforceable. The change in the neighborhood must be so fundamental that the servitude is rendered meaningless because the servitude can no longer survive in this changed neighborhood. Choice (D) is incorrect. This probably would be construed as a commercial enterprise because the doctors are running a for-profit medical clinic.
140
Q
  1. A landowner, advancing in age, realizes that he is no longer able to farm his 1,000 acres and therefore decides to sell some of the farmland in parcels of 250 acres. The president of a development company is interested in purchasing three of the four parcels. The president buys the three parcels from the landowner and begins formulating plans for constructing single-family dwelling units on the land. The original deed between the landowner and the development company contains a provision expressly binding “upon all subsequent grantees, their heirs, and assigns,” stipulating that any further subdivisions by any such persons shall be restricted to minimum two-acre lots to be used for single family dwelling units only.The development company immediately subdivided two of the parcels into lots of three, four, and five acres, and began construction of homes thereon. The original deed restrictions were enumerated within the special warranty deeds and were given to the purchasers of the homes in the new development, called phase 1.Two years later, the president sold the remaining parcel, which had not been included in the phase 1 subdivision plan, to a contractor. The contractor, in turn, drafted a subdivision plan for the last 250-acre parcel dividing it into one-acre lots. The contractor then commenced construction of single-family dwelling units in the new development, to be known as phase 2. There was no mention of the restriction for two-acre minimum lots in the deeds to the purchasers of the new homes in phase 2.Meanwhile, after the landowner’s death, his estate is required to sell the remaining 250-acre parcel of his farmland. The buyer is an investor, who proposes to construct a two-level shopping center and parking lot on the property. The deed for the remaining 250-acre parcel contained the identical restrictions as the deeds to the other three parcels. The investor, ignoring these restrictions, was able to use his political influence to persuade the county zoning board to rezone the said parcel for commercial use.The residents of phase I seek to enjoin the construction of the shopping center. The residents will have(A) little chance of preventing the projected commercial development.(B) no standing to enjoin the construction of the shopping center.(C) no chance of succeeding, because the zoning ordinance takes precedence over the covenant running with the land.(D) success in their cause of action in accordance with the common development scheme.
A
  1. (D) A subsequent zoning ordinance will not destroy the restrictive covenant as long as there has not been such a change in the neighborhood as to make the physical use and enjoyment of the restriction meaningless. Quite so, the restrictive covenants in the deeds of the owners of phase 1 will be enforceable in accordance with the common development scheme preserving the residential character of the property. Choice (A) is incorrect. This is the opposite of choice (D), the correct choice. A subsequent zoning ordinance will not destroy the restrictive covenant as long as there has not been such a change in the neighborhood as to make the physical use and enjoyment of the restriction meaningless. Just because the area has been rezoned is not to say the restriction cannot co-exist. Nothing in the facts suggests that the neighborhood has changed so much as to render the covenant now unenforceable; all that has changed is that there has been a rezoning to allow for commercial use. That does not necessarily mean that the neighborhood itself has changed to such an extent as to make the physical use and enjoyment of the restriction meaningless. Choice (B) is incorrect. A covenant running with the land will be enforceable against successors of the original parties, where the following four requirements are met: (1) there is privity of estate between the original parties and their successors; (2) the covenant “touches and concerns” the land; (3) the parties intend the covenant to run with the land; and (4) the covenant is in writing. Thus, the subsequent grantees, i.e., owners of phase 1 lots, may enforce the original restrictive covenant between the landowner’s estate and the investor. Because the current owners will have standing to sue, even though they were not the original covenanting parties, this answer choice is incorrect. Choice (C) is incorrect. A subsequent zoning ordinance will not destroy the restrictive covenant as long as there has not been such a change in the neighborhood as to make the physical use and enjoyment of the restriction meaningIess.As long as that does not occur, there is no reason to presume that the zoning ordinance necessarily “trumps” the covenant just because the covenant was arranged through a private agreement.
141
Q
  1. A woman inherited her father’s farm when he died testate. Prior to her father’s death, the woman had already taken over the operations on the father’s farm due to his declining health.Recently, the woman had granted a coal company rights to strip-mine coal from underneath the farm. Their agreement stipulated that the coal company would pay the woman a per-ton royalty for the coal extracted. In addition, the coal company agreed to fill in the excavated area and replace top soil on the surface of the land.After the coal company commenced its strip-mining operations, the woman noticed that the company was not filling in the excavated area as previously agreed. However, because the company paid the woman all the coal royalties from the strip mining, she did not voice any objection regarding its failure to replace the top soil. Two years later, the coal company had now completed its strip-mining operation under its arrangement with the woman.The coal company’s right to strip-mine coal from the woman’s property would be an example of a (an)(A) profit-a-prendre.(B) license.(C) easement in gross.(D) voluntary waste.
A
  1. (A) A profit (sometimes called a “profit-a-prendre”) is a non-possessory interest in land and consists of a right to take the soil or a substance of the soil. If the exercise of the right is restricted in the sense that it may be exercised only in connection with the use and enjoyment of a dominant estate, the right is a profit appurtenant. If the right is not so restricted, it is a profit in gross. Choice (B) is incorrect. A license simply permits one person to come onto the land in the possession of another without being a trespasser. A license is not an interest in land. It is merely a privilege, revocable at the will of the licensor. Because the coal company received the right to take coal from the property of another, their interest is a profit, not a license. Choice (C) is incorrect. An easement is an interest in land that allows the holder of the easement to use the property of another. An easement is in gross when it is intended to benefit the owner or possessor personally, rather than in connection with any land the holder owns. The coal company received more than just the right to use the land of another; it received the right to take substances and things from the property. The company’s interest is a profit, not an easement in gross. Choice (D) is incorrect. Voluntary waste consists of injury to the premises or land caused by an intentional or negligent affirmative act of the tenant (either life tenant or tenant for years). However, the term voluntary waste refers to the conduct of a tenant, not the right granted. Because the call of the question asks for the name of the interest given, this choice is incorrect.
142
Q
  1. A man, a teenager, and a woman are siblings who inherited their father’s farm when he died testate, leaving his farm to his children as joint tenants. Soon after the father’s death, the teenager died and the woman moved to another part of the country. The man has not heard from the woman in many years.Prior to their father’s death, the man and the teenager operated their father’s farm. They continued doing so after their father died, sharing all expenses equally and dividing the profits between them. Following the teenager’s death, the man has continued to operate the farm for his sole benefit. Recently, the man has granted a coal company rights to strip-mine coal from underneath the farm. Their agreement stipulated that the coal company would pay the man a per-ton royalty for the coal extracted. In addition, the coal company agreed to fill in the excavated area and replace top soil on the surface of the land.During the coal company’s strip-mining operation, the woman returns to the farm and demands a proportionate share of the royalties paid to her brother (the man).Which of the following is the LEAST accurate statement regarding the woman’s right to share in the royalties?(A) As a joint tenant, the woman would be entitled to contribution for a proportionate share of the royalties.(B) It would be inequitable for one concurrent owner to receive an unapportionate share of the royalties.(C) The woman’s redomiciling constituted a severance of the joint tenancy, which resulted in a destruction of her interest in the mining royalties.(D) Because the woman’s redomiciling did not effectuate an ouster, she retained her right to contribution as a joint tenant.
A
  1. (C) Choice (C) is the LEAST accurate statement because the redomiciling (of a joint tenant) does not effectuate a severance. Generally, a severance of the joint tenancy can be made by (1) conveyance inter vivos, or (2) by partition only. Be aware that a severance can never be made by will because survivorship is prior to and defeats the effect of the will. Choice (A) is incorrect. A co-tenant out of possession has a right to share in rents and profits received from third parties. Because the call of the question asks for the least accurate statement, this choice will be discarded because it is an accurate statement regarding the rights and duties of joint tenants. Choice (B) is incorrect. A co-tenant out of possession has a right to share in rents and profits received from third parties. While it may be true that the woman should be allowed to receive a proportionate share of the royalties (to do equity), this is not the strongest statement of the four because this is not really expressing a specific rule of law. However, it is still not the least accurate and must therefore be discarded. Choice (D) is incorrect. A tenant in possession need not share such profits with a co-tenant out of possession, unless there has been an ouster. An ouster occurs where one co-tenant manages to wrongfully exclude her co-tenants from possession of the property. Because the woman’s redomiciling alone does not constitute an ouster, the woman still retains the right to share in the proceeds. This choice is an accurate statement and is incorrect.
143
Q
  1. In 1980, an attorney purchased an 80-acre tract in a rural county. The 80-acre tract included the family home, an exquisite mansion built in 1929 by a wealthy industrialist. For many years, the closest town to the tract had been in the grips of economic decline. However, in 1988 and 1989, several large corporations built plant facilities in the local area.By the early 1 990s, prosperity had burst upon the area. To take advantage of the real estate boom, the attorney divided the 80-acre tract into 160 lots. By 1995, the attorney had sold 90 lots to various individual buyers. Each deed contained the following provisions:“It is an express covenant and condition that the property hereby conveyed shall not be used for other than single-family residences.”“The grantor hereby covenants for himself, his heirs, successors, and assigns to insert a similar restriction in all deeds to lots now owned by grantor in the 80- acre tract.”In 1996, the county board of supervisors purchased from the attorney 30 lots within the 80-acre tract. The board, which had the power of eminent domain, planned to construct a new county pest-control office on the property. The deed of conveyance from the attorney to the county board of supervisors made no mention of the aforesaid restrictions contained in the deeds to other lot owners in the 80-acre tract. The attorney—county deed, however, did contain a provision, which stated:“The grantee hereby covenants for itself, its successors, and assigns that this conveyance is made in lieu of the exercise of the power of eminent domain.”Thereafter, the county enacted a zoning ordinance whereby the 30 lots were rezoned to permit the construction of a pest-control office building. When the county started construction, all the other lot owners in the 80-acre tract brought suit to enjoin the building plan or, in the alternative, to recover damages.If the county board of supervisors, the named defendant, prevails on both counts, it will be because(A) the power of eminent domain is the equivalent of the power to zone.(B) the purchase of the 80-acre tract property by a public body with the power of eminent domain is an act of inverse condemnation.(C) the abrogation of the right to enforce the restrictive covenant is not a compensable taking.(D) restrictions on the use of land can be enforced only against a buyer whose deed contains the restrictions.
A
  1. (C) The attorney—county deed contained a provision that abrogated the right to enforce the restrictive covenant. If the Board can argue that this is not a compensable taking because the attorney himself abrogated the restriction, the county will not have to pay just compensation because no taking was required. Thus, eminent domain would not be in issue. If the board can successfully argue that there was no taking, then the defendant will prevail. Choice (A) is wrong because the question wants you to presume that the lot owners will not prevail in their action to recover damages. In eminent domain proceedings, just compensation must always be paid to the landowner. Thus, if the lot owners fail in their suit to recover damages against the county, then eminent domain was not the means by which the county was able to use the land for an office building because if eminent domain were the controlling issue, the property owners would receive just compensation. Choice (B) is wrong because the question wants you to presume that the lot owners will not prevail in their action to recover damages. In an inverse condemnation proceeding, justcompensation must always be paid to the landowner. Thus, if the lot owners fail in their suit to recover damages against the county, then inverse condemnation was not the means by which the county was able to use the land for an office building because if that were the controlling issue, the property owners would receive just compensation. Choice CD) is wrong because an equitable servitude can be enforced against a buyer in a subdivision scheme, even though his/her deed does not necessarily contain the restriction. Whether or not a particular person can enforce an equitable servitude is determined by the intention of the parties.
144
Q
  1. A woman owns a tract of land located in a state in the Midwest. On June 1, 2005, the woman sells to a husband and wife an undivided one-half interest in this land for $100,000 (the entire consideration being paid by the husband). The deed to the husband and wife reads as follows: “To (husband) and (wife) and their heirs as tenants by the entirety, and not as joint tenants, and not as tenants in common.”On June 1, 2006, the woman sells her remaining interest in the tract of land to the husband and his brother for $125,000 (the husband pays $80,000 of the purchase price, and the brother pays the balance). The deed to the husband and the brother provides: “To (the husband) and (the brother) and their heirs as joint tenants and not as tenants in common.”The husband conveys to his cousin all his right, title, and interest under the two deeds from the woman. The husband then dies. The brother then dies. The cousin is thus the owner of(A) an undivided one-third interest in the land.(B) an undivided one-quarter interest in the land.(C) an undivided one-half interest in the land.(D) an undivided three-quarters interest in the land.
A
  1. (B) At common law, a tenancy by the entirety was created by a deed or will conveying to a husband and wife. The incident of survivorship attached such that neitherspouse acting alone while the marriage existed could sever the tenancy or defeat the right of survivorship. See Smith and Boyer, Law of Property, pg. 65—66. Therefore, when the husband conveyed all his interest under the June 1, 2005 deed to the cousin, the tenancy by the entirety remained intact and the cousin received no interest with regard to that one-half interest in the Land. Ajoint tenancy, on the other hand, can be severed by conveyance of the interest of one of the joint tenants. Such a conveyance destroys the unities of time, title, and interest and results in the creation of a tenancy in common (which contains only a unity of possession). Therefore, when the husband conveyed all his interest under the June 1, 2006 deed to the cousin, the joint tenancy was severed with the cousin and the brother each holding an undivided one-half interest as tenants in common. Upon the brother’s death, no right of survivorship inured to the cousin, so the cousin retains only his one-half interest. Because the two deeds from the woman to the husband each comprised one-half of the entire tract, the cousin’s interest is an undivided one-quarter interest in the land. Choice (B) is thus correct. The fact that the husband paid more of the purchase price than the brother under the 2006 deed ($80,000 of the $125,000 purchase price) is irrelevant because the husband and the brother took as joint tenants—the language of the deed so expressed this intent—and therefore, by definition, they had to take the same interest (i.e., unity of interest). Had they taken as tenants in common, then the husband would have owned a greater interest than the brother, even though both co-tenants would have shared an equal right of possession. Choice (A) is incorrect. The property interests in this fact pattern were never parsed out in thirds. The parcels were conveyed in one-half interests and some interests became quarters (such as when the husband conveyed to the cousin, destroying the joint tenancy). Choice (C) is incorrect..The cousin cannot own a one-half interest in the land because when the husband conveyed to the cousin, that fact severed the joint tenancy that once existed between the husband and the brother. That meant that the brother and the cousin owned as tenants in common, so the most the cousin could have ever owned was an undivided one-quarter interest in the land. Choice (D) is incorrect. The cousin cannot own the one-half interest that the husband held with the wife as tenants by the entirety. Because the husband, acting alone, could not unilaterally convey his interest, that transfer to the cousin had no effect. At most, the cousin could own an undivided one-quarter interest in the land.
145
Q
  1. A farmer owned an olive grove. The farmer gavea friend a mortgage on the olive grove to secure aloan from the friend to the farmer in the amountof $90,000. The farmer then gave a co-workera mortgage on the olive grove to secure a loanfrom the co-worker to the farmer in the amount of$120,000.The farmer then gave an investor a mortgage on the olive grove to secure a loan from the investor to the farmer in the amount of $110,000. The co-worker then records. The investor then records. Thereafter, the friend records. In a foreclosure proceeding where the friend, the co-worker, and the investor were parties, the olive grove sold for $220,000.The recording statute in this jurisdiction provided “any written instrument affecting title to land that is not recorded is void against a subsequent purchaser in good faith for valuable consideration, whose conveyance shall be first duly recorded.”If the olive grove sold for $220,000, what dollar amount, if any, should go to the investor?(A) $0.(B) $90,000.(C) $100,000.(D) $110,000.
A
  1. (C) In this race-notice jurisdiction, note that when both the co-worker and the investor took their mortgages, no one else had recorded, so both had taken without notice of any of the other parties. If this were a pure notice jurisdiction, then the investor would prevail here because notice is all that would matter. However, because this is a race-notice jurisdiction, we also require someone to be the first to record. Because both the co-worker and the investor are in the same situation (inasmuch as they both took without notice of any prior mortgagee), we now must see who was the first to record of those two. Because the co-worker recorded becore the investor, her security interest will trump the investor’s. As a result, the co-worker is able to satisfy her $120,000 claim from the foreclosure proceeds. The investor will be entitled to satisfy as much of her claim as possible and wilL receive the remaining $100,000. The friend receives nothing from the proceeds of the foreclosure sale because after the investor takes, there is nothing left. Choice (A) is incorrect. Because the investor was the second party to record, she will take some portion from the sale proceeds because the property did at least sell for more than the value of the co-worker’s $120,000 claim. Thus, the investor can take the remaining $100,000 from the $220,000 purchase price. Choice (B) is incorrect. This is the amount of the friend’s mortgage and is thus an incorrect dollar figure. Moreover, there is nothing to limit the investor’s recovery to only $90,000; she is entitled to take the remaining $100,000 from the sale proceeds after the co-worker has been paid. Choice CD) is incorrect. The investor is not entitled to the full amount of her original mortgage in the olive grove. Because the investor was not the first to record (but rather the second), she must take after the co-worker is paid and runs the risk that what is left over will not be enough to satisfy her debt and that is what occurred here. At most, she will be entitled to the remaining $100,000.
146
Q
  1. Ann owns a tract of land. On August 1,2005, Ann sells to Hunter and Willa, a husband and wife, an undivided one-half interest in this land for $100,000 (the entire consideration being paid by Hunter). The deed to Hunter and Willa reads as follows: “To Hunger and Willa and their heirs as tenants by the entirety, and not as joint tenants, and not as tenants in common.”On August 1, 2006, Ann sells her remaining interest in the tract of land to Hunter and Brandon, his brother, for $125,000 (Hunter pays $80,000 of the purchase price, and Brandon pays the balance). The deed to Hunter and Brandon provides: “To Hunter and Brandon and their heirs as joint tenants and not as tenants in common.”On August 1, 2007, Hunter, Willa, and Brandon die in a common disaster, and as a consequence, the order of their deaths cannot be established by proof. Hunter’s will and Willa’s will devise whatever interest in the land in question that is subject to disposition by their wills to their son.Thus, the son owns(A) an undivided one-third interest in the land.(B) an undivided one-quarter interest in the land.(C) an undivided one-half interest in the land.(D) an undivided three-quarters interest in the land.
A
  1. (D) A tenancy by the entirety is destroyed by the death of both spouses because the entirety interest applies only to a husband and wife during marriage. Death, just as divorce, eliminates the unity of person, and thus the need for the attribute of survivorship is also eliminated. Disposition of property upon the death of both spouses would then follow just as if the tenancy by the entirety were severed into a tenancy in common. In other words, the heirs would inherit the undivided interest of the deceased cotenant. Similarly, the disposition of a joint tenancy would follow as if a tenancy in common had existed. Therefore, even though neither a tenancy by the entirety nor a joint tenancy can be disposed of by will, a tenancy in common can be so devised. Thus, the son would obtain Hunter and Willa’s one-half interest in the land under the 2005 deed as weLl as Hunter’s one-quarter interest under the 2006 deed. Choice (D) is, therefore, the correct answer. See Smith and Boyer, Law of Property, pp. 61—65. Choice (A) is incorrect. The property interests in this fact pattern were never parsed out in thirds. The parcels were conveyed in one-half interests, and some interests became quarters (such as when the death of both Hunter and Brandon severed the joint tenancy passing a one-quarter interest to the son). Choice (B) is incorrect. After the deaths of all the parties, the joint tenancy and the tenancy by the entirety became severed. As a resuLt of all of this, the son would obtain Hunter and Willa’s one-half interest in the land under the 2005 deed as well as Hunter’s one-quarter interest under the 2006 deed. Because the son now will own an undivided three-quarters interest in the land, this choice is incorrect. Choice (C) is incorrect. After the deaths of alL the parties, the joint tenancy and the tenancy by the entirety became severed. As a result of all of this, the son would obtain Hunter and WiLla’s one-half interest in the land under the 2005 deed as well as Hunter’s one-quarter interest under the 2006 deed. Because the son now will own an undivided three-quarters interest in the land, this choice is incorrect.
147
Q
  1. On May 10, 1955, a rancher, owner of a 300-acre cattle ranch in Texas, conveyed a 20-acre strip across the property as follows:“To the railroad, its successors and assigns, to have and to hold so long as the same shall be used for railroad purposes.”In 1972, the rancher made the followingconveyance:“To my daughter and her husband and their heirs, as much of the 300-acre cattle ranch as is not described in my deed to the railroad dated May, 10, 1955.”The following year, the rancher, a widower, died intestate, survived by the daughter and a son. In 2000, the railroad decided to discontinue operating its trains over the strip conveyed in 1955.By 2004, the growth of a nearby town had made the 300-acre cattle ranch valuable as a potential site for homes or for an industrial park. However, as of January 1, 2005, the governing body of the county (in which the ranch sits) took appropriate action, in accordance with Texas statutes, to zone the 300-acre cattle ranch as single-family residential property with a minimum lot size of one acre. The ordinance provided that the exclusion of ranching operations and industrial development was necessary to protect the health of county residents by limiting the extent of waste disposal and preventing pollution of air and the nearby river, the county’s major water supply.The state’s power authority has now taken appropriate action to condemn half of the 300-acre cattle ranch, which will be flooded as a result of construction of a dam for a proposed hydroelectric facility. The 150 acres taken includes the 20-acre strip described in the deed to the railroad.Is the railroad entitled to any compensation from the state’s power authority?(A) Yes, because if all the leased land is condemned for the full balance of the lease term, the lessee is entitled to compensation for the taking of the leasehold estate.(B) Yes, because the holder of an easement benefit is entitled to compensation for the value lost.(C) No, because by discontinuing its operation of trains, the railroad’s determinable, feeterminated.(D) No, because the holder of an easement is not entitled to compensation when the servient tenement is extinguished by condemnation.
A
  1. (C) First, it is necessary to determine what interest the railroad has across the 300-acre cattle ranch property. The answer is a fee simple determinabLe, which is a fee simple created to continue until the happening of a stated event. Because the duration of the estate is correlated to the happening of a named event—here, the ceasing to use it for railroad purposes—the estate terminates automatically by operation of law upon the happening of that event. A very important characteristic of this type of estate is that the instant it is no longer used for railroad purposes, it “reverts” back to the grantor or his heirs. The following words are usually used to create a determinable fee: “until” or “as long as.” Choice (A) is wrong because nothing in the deed instrument would indicate the creation of a leasehold interest. The railroad received a fee interest in this strip that could have continued indefinitely as long as the strip was used for railroad purposes. Choice (B) is incorrect because the railroad does not have an easement. The railroad received a fee interest in this strip that could have continued indefinitely as long as the strip was used for railroad purposes. Choice (D) is incorrect because the railroad does not have an easement. The railroad received a fee interest in this strip that could have continued indefinitely as long as the strip was used for railroad purposes.
148
Q
  1. A buyer signed a contract to purchase a tract of land from a seller. The contract was signed on May 1, 2006, and the closing is set for June 15, 2006. The land was located in a jurisdiction that has in force the following statute:Statute of Limitations—”an action to recover the possession of land shall be brought within twenty- one (21) years after the cause thereof accrued, but if a person who is entitled to bring such action at the time the cause accrued is within the age of minority (under 21) or of unsound mind, such action may be brought within ten (10) years after such disability is removed.”This statute was enacted in the jurisdiction in 1930. The land in question had been owned by a rancher in 1960. On September 1, 1960, the rancher died intestate, leaving his nephew as his only heir. The nephew had been born on October 1, 1954. In addition, there is a deed from the rancher’s administrator to the seller dated November 1, 1960, which the latter recorded on the same date.During his title search, the buyer learned that the administrator had never obtained any license to sell the land in question; and also he (the administrator) never formally received the approval of any court with respect to the administration of the rancher’s estate. Moreover, the buyer ascertained that the seller entered into possession of the land in question on November 1, 1960.On the assumption that there are no additional facts, the buyer should be advised that the seller became or will become the owner of the land in question(A) on November 1, 1960.(B) on November 12, 1981.(C) on October 1, 1985.(D) when the nephew dies.
A
  1. (C) Based on the present factual situation, the nephew, as the rancher’s only heir, would have until October 1, 1975 (when he reached majority) plus 10 years in which to bring his ejectment action. Therefore, the seller became the owner on October 1, 1985, which, in accordance with the Statute of Limitations, was the last date on which the nephew could initiate an action to recover possession of the land in question. Choice (A) is incorrect. This would have represented the date that the administrator fraudulently gave to the seller to record. This is incorrect because the nephew was a child at the time of this transfer and had until 10 years after reaching the age of 21 (age 31) in which to bring an ejectment action against an adverse possessor. Choice (B) is incorrect. This date does not appear to have any significance to these facts because it is not the date of a significant event nor is this the anniversary of a significant event. Choice (D) is incorrect. The statute allows an ejectment action to be brought, at the very latest, within 10 years of reaching the age of majority or when the disability of unsound mind is removed. This choice would allow the nephew to bring an ejectment action at any point before his own death, which could be many more than 10 years after reaching the age of majority. Because this statute allows at most 10 years in which to bring the ejectment action, this choice is incorrect.
149
Q
  1. A buyer signed a contract to purchase a tract of land from a developer. The contract was signed on May 1, 2008, and the closing was set for June 15, 2008. The land was located in a jurisdiction that has in force the following statute:Statute of Limitations—”an action to recover the possession of land shall be brought within twenty (20) years after the cause thereof accrued, but if a person who is entitled to bring such action at the time the cause accrued is within the age of minority (under 21) or of unsound mind, such action may be brought within ten (10) years after such disability is removed.”This statute was enacted in the jurisdiction in 1932. The land in question had been owned by a doctor in 1962. On September 1, 1962, the doctor died intestate. The doctor’s administratrix conveyed the tract to the developer on November 1, 1962, which the latter recorded on the same date.During her title search, the buyer learned that the administratrix had never obtained any license to sell the land in question. The buyer also learned that the administratrix never formally received the approval of any court with respect to the administration of the doctor’s estate. Moreover, the buyer ascertained that the developer entered into possession of the land in question on November 1, 1962.The developer was ousted from the land in question by a woman on October 1, 1982. The developer then recovered possession of the land from the woman in an action of ejectment on December 31, 1982. The buyer should be advised that the developer became the owner of the land in question on(A) November 1, 1962.(B) November 1, 1982.(C) December 31, 1982.(D) December 31, 2002.
A
  1. (D) Under the doctrine of adverse possession, for one to hold real property adversely, that person’s possession must be continuous and unbroken for the entire length of the statutory period. Because the developer was ousted from the land, her possession was disrupted due to the ouster and would not be continuous. Therefore, the developer will not become owner of the property (through adverse possession) until December 31, 2002. Because the administratrix was not in a position to convey legal title (because her administration of the doctor’s estate was never formally approved by the court) of the property to the developer, the latter will not acquire Legal title to the property until the expiration of the period of adverse possession. Choice (A) is incorrect. Because the manner in which the administratrix disposed of the doctor’s property was never approved by the court, the manner in which the developer took title was not proper. The administratrix was not in a position to convey title, so the deed to the developer has no effect. So, even though there was a conveyance on November 1, 1962, that is not the date that the deveLoper took title. Choice (B) is incorrect. The developer was ousted from the land by the woman on October 1, 1982. This date was just one month before the 20-year statutory limitations period would have expired. This act by the woman served to put the developer out of possession and stopped her own possession of the property. Because the developer had been dispossessed of the property, November 1, 1982 will no longer be of any meaning to the developer; she must start all over again. Choice (C) is incorrect. This is the date that the developer, at most, recovered the property from the woman and began to repossess it adversely. If anything this is the beginning of her period of adverse possession.
150
Q
  1. Osgood is the record owner of Desertacre, a 100- acre tract of farmland in California’s Coachella Valley. Osgood inherited the property from his father and farmed the land until 2006, when he reached the age of 70. He then decided to retire and move to Scottsdale, Arizona. At that time, Osgood conveyed Desertacre to Cutler “for his use during his natural life.”After taking possession of the property, Cutler executed a mortgage on Desertacre in the amount of $100,000. In 2009, Osgood died and in his will, the testator devised Desertacre to his son, Desmond, with remainder to Deirdre. The will provided that Desmond and Deirdre took Desertacre “subject to Cutler’s mortgage.”At the time of Osgood’s death, Desertacre had a market value of $300,000. When Desmond learned of the devise, he was hesitant about taking title to Desertacre because he did not want to incur any indebtedness. Assume that Culter is still alive and in possession of the tract.With respect to the mortgage, Desmond will be(A) liable for the mortgage, because he is the devisee under the terms of Osgood’s will.(B) liable for the mortgage, because the property was devised subject to the mortgage.(C) not liable, because there was no assumption of the mortgage.(D) not liable, because he did not personally grant the mortgage.
A
  1. (C) A life tenant has the right to use the property during her/his lifetime. This includes the right to mortgage, to create liens, easements, or leases. However, none of these dispositions can extend beyond the period of the life estate. Boyer, Law of Property, pg. 13. Even though Osgood’s will provided that Desmond took the property “subject to Cutler’s mortgage,” this provision would be invalid because the mortgage on Cutler’s life estate could never extend past the end of Cutler’s life. Because Desmond never assumed the mortgage, he is not personally liable for it. Choice (A) is incorrect. Cutler’s mortgage was only as good as the estate attached to it; when Cutler dies, the mortgage will end with the life estate. For this reason, Desmond will not be liable. Moreover, being a devisee does not automatically create liability for Cutler’s mortgage. Unless Desmond took the property and expressly assumed the mortgage, Desmond is not personally liable for the mortgage. Choice (B) is incorrect. Cutler’s mortgage was only as good as the estate attached to it; when Cutler dies, the mortgage will end with the life estate. For this reason, Desmond will not be liable. So, even though Osgood’s will provided that Desmond took the property “subject to Cutler’s mortgage,” this provision would be invalid because the mortgage on Cutler’s life estate could never extend past the end of Cutler’s life. Choice (D) is incorrect because it falsely suggests that only the person giving the mortgage can be the Liable party. Had Desmond assumed the mortgage when he received the property, he would have been liable even though it was Cutler who initially gave a mortgage on Desertacre. The better reason why Desmond is not liable is because he did not assume the mortgage, as stated in choice (C).
151
Q
  1. In 1973, a woman was the actual and record owner of 20 acres of certain undeveloped timberland. In September 1973, the woman mortgaged the 20 acres of timberland to the bank by a mortgage deed (in the traditional form of a conveyance in fee simple subject to a condition subsequent), which was not recorded until January 1974. The mortgage deed contained the following clause immediately after the legal description of the 20 acres of timberland:“Together with all the real property now owned by (the woman) or which shall be owned by (the woman) during the continuance of the indebtedness secured by this mortgage.” This mortgage was given, as the recorded instrument revealed, to secure a note for $100,000 repayable over a 40-year term.In March 2004, the woman, using money loaned by a finance company, purchased a 50-acre mountainous estate situated outside of a major city. This deed was recorded immediately. In April 2004, the woman executed and delivered to the finance company a mortgage to secure its loan. This mortgage was promptly and duly recorded. The finance company had no actual notice of the prior mortgage to the bank.In February 2007, the woman defaulted on both mortgages. The bank then initiated foreclosure proceedings against the woman and the finance company as joint defendants. In its foreclosure action, the bank averred that its mortgage was a first mortgage on both the 20 acres of timberland and the 50-acre estate. The finance company filed a cross-complaint, alleging that its mortgage was a first mortgage on the 50-acre estate and praying for foreclosure on that parcel of property.In order to decide this case in favor of the bank, the court does not need to resolve which of the following issues?(A) Whether the finance company is charged with record notice of the bank’s mortgage.(B) Whether the quoted clause in the mortgage instrument to the 20 acres of timberland covers the 50-acre estate.(C) Whether the finance company’s mortgage is a purchase money mortgage.(D) Whether the finance company can rely on the doctrine of implied purchase money liens.
A
  1. (D) Questions covering mortgages are frequently tested on the MBE. By definition, a mortgage is an interest in land created by a written instrument providing security for the performance of a duty or the payment of a debt. Mortgages and assignments thereof should be recorded. Failure to record the mortgage may make it possible for the mortgagor to convey to a bona fide purchaser who would take free of the mortgage under the recording act. As such, choice (A) is a correct statement and thus an incorrect choice because the court would have to resolve the issue of whether or not the finance company had record notice of the bank’s mortgage. Even though the facts state that the finance company had no actual notice of the mortgage to the bank, if the court finds there is record (or constructive notice), then the finance company will not prevail. Choice (B) is also a correct statement because the deed instrument stated that the mortgage would apply to property “now owned by (the woman) or which shall be owned” in the future. Consequently, when the woman subsequently purchased the 50-acre estate, the bank would argue that its mortgage interest covers that property as welt as the 20 acres of timberland. Note that such “after-acquired” property clauses in mortgage instruments are enforceable as long as the after-acquired property is sufficiently identified to put third parties on notice. Because this issue of whether or not the quoted clause applies to the 50-acre estate is one that the court will, in fact, have to address, choice (B) is incorrect. In addition, choice (C) is aLso a correct statement because if the finance company’s mortgage is viewed as a purchase money mortgage, it would clearly take priority over the “after-acquired” property clause contained in the bank’s mortgage instrument. A purchase money mortgage takes priority over other prior mortgages regardless of recording statutes. The purchase money mortgage itseLf, however, must be recorded. Under these facts, it appears that the finance company’s mortgage was a purchase money mortgage, but the court would still have to decide this issue to come to this conclusion, making (C) an incorrect choice. Choice (D) is an issue that the court will not have to resolve, thus making it the correct choice. There is no reason for the finance company to resort to an impLied purchase money Lien because the court will find that the finance company received a purchase money mortgage.
152
Q
  1. A seller sold the eastern two-thirds of his land to a buyer, who soon thereafter constructed a house there. One year later, the buyer cleared a path from her home across the seller’s retained land to a road that abuts the western boundary of the seller’s land. The seller stood by and watched the buyer clear the path, but made no objection. This path is very convenient to the buyer’s use of her land, and the buyer used it daily for several months. The path is readily apparent to anyone. Recently, the seller put a barrier across the path. The buyer now has brought an action to have the barrier removed.The theory giving the buyer her best chance of success would be that(A) the buyer has an easement by necessity. This would depend on the strength of the court’s feeling about the use of the laid.(B) the buyer has an easement by implication. This would depend on whether the convenience was sufficiently strong to amount to “reasonably necessary.”(C) the buyer has an easement by prescription. This would depend on the length of time she used the path prior to the erection of the barrier.(D) the buyer has an express easement.
A
  1. (B) An easement by implication generally arises when the owner of two or more adjacent parcels sells one or more of them and it is clear (although no easement was mentioned in the instrument of conveyance) that one was intended. In order to establish an easement by implication, one part of the land must have been used for the benefit of the other part at the time of the conveyance, the use must be apparent and continuous, as well as necessary to the enjoyment of the quasi-dominant tract. In alL Likelihood, the buyer’s best chance of success would be to show that she acquired an easement by implication, rather than an easement by necessity. This is so because a greater degree of necessity is required by the latter easement. For an easement by necessity, the buyer must prove that the easement is “necessary.” The facts tell us that the buyer used the path regularly, but also that she cleared the path a year after building the house, suggesting that it was convenient—but not necessary. Choice (B) is preferred over choice (A) because an easement by implication does not require as great a burden of proof as an easement by necessity. The question prompt asks for the BEST CHANCE of success. The establishment of an easement by prescription, much Like the establishment of ownership by adverse possession requires that the use be (1) adverse as distinct from permissive, (2) open and notorious, (3) continuous and without interruption, and (4) for the statutory period. Because no facts suggest that these elements have been met, answer choice (C) is wrong. Choice (D) is incorrect because no easement was granted expressly through a writing.
153
Q
  1. A landowner, being fee simple owner of an apartment building, devised it “to both my assistant and my friend as tenants in common.” Thereafter, the assistant died intestate, leaving his daughter as his only surviving heir. While the assistant’s estate was in administration, the friend agreed to convey his interest in the apartment building to the daughter.A valid conveyance by the friend would be(A) to the estate of the assistant and its successors and assigns.(B) to the heirs and assigns of the assistant.(C) to the daughter and her heirs and assigns.(D) to the assistant as a former tenant in common with remainder to the daughter.
A
  1. (C) Many students do poorly on the Multistate because they “out-psych” themselves into thinking that every MBE question involves some “trick.” This is not true. Sometimes the simple, obvious answer is correct. Here, for example, the facts state that “the friend agreed to convey his interest in the apartment building to the daughter.” The question then asked is, “Which of the following would be a valid conveyance by the friend?” The correct answer is “to the daughter and her heirs and assigns,” as this language would transfer the friend’s interest to the daughter in fee simple. Choices (A) and (B) are wrong because they do not carry out the friend’s intention, whkh was to convey his interest in the apartment building to the daughter and not to the estate of the assistant. Furthermore, the language in choice (A) suggests that the assistant’s estate can have heirs and assigns. Only a person has heirs and assigns. Choke (‘D) is incorrect because it is not possible to convey an interest in land to a dead person. Because the assistant is dead she cannot take, so there is no “remainder” for the daughter.
154
Q
  1. A doctor owned a two-acre tract just outside the city.She subdivided the parcel into 12 lots, numbered1—12, 11 of which she sold to 11 different buyers.The doctor retained lot 12 to live on. Each deed tothe 11 lots sold contained the following restriction:“It is an express covenant and condition that the property hereby conveyed shall not be used for other than single-family residences.”Two years after the doctor subdivided and sold off the parcels, the purchasers of lots 1—11 had each built residences on their property. In December of that year, the doctor dug a well on her property. From that well, the doctor supplied water to all of the lot owners. One year later, a veterinarian, who lived on the lot adjacent to the doctor’s, dug a well in her backyard. Her well caused water to be diverted from the doctor’s well, and the doctor is no longer able to get any water.In an appropriate action by the doctor against the veterinarian, what remedy, if any, is available?(A) An injunction should be decreed enjoining the veterinarian from using her well.(B) The doctor should be entitled to recover money damages from the veterinarian.(C) The veterinarian should be required to supply water to the doctor.(D) There is no remedy.
A
  1. (D) Percolating water is water beneath the surface of the earth that is not confined to a known and well-defined channel or bed. The common law rule is one of absolute ownership. A landowner is not restricted in the withdrawal of percolating water located beneath the surface of her land, even if this causes drainage of water from adjoining land to the damage of other landowners. An increasing number of states now apply the “reasonable use” rule. Under this rule, all of the landowners have a correlative right with respect to percolating water located beneath the surface of their land. Even under the reasonable use rule, choice (D) is correct; no facts suggest that the veterinarian’s use of the water is unreasonable. Because neither money damages, injunction, nor other equitable relief is available, choices (A), (B), and (C) are wrong. As discussed above, percolating water under the surface is subject to the absolute control and ownership of the surface owner, and if the withdrawal affects the neighboring landowner, it is damnun absque injuria; that is, an injury without remedy.
155
Q
  1. A professor was the record owner in fee simple absolute of a 30-acre tract of land located in a small town. The professor made a conveyance thereof in these words, “I hereby convey my 30-acre tract to both my friend and my co-worker as joint tenants with right of survivorship.”Two years after making the conveyance to the friend and the co-worker, the friend executed a mortgage on the 30-acre tract of land to a bank to secure a loan. One year after executing the mortgage, the friend died intestate, leaving his uncle as his only heir. At the time of the friend’s death, the indebtedness had not been paid to the bank. The jurisdiction in which the 30-acre tract of land is located recognizes a title theory of mortgages.In an appropriate action, the court should determine that title to the 30-acre tract of land is vested(A) in the co-worker, with the entire interest subject to the mortgage.(B) in the co-worker, free and clear of the mortgage.(C) half in the co-worker and half in the uncle, with both subject to the mortgage.(D) half in the co-worker, free of the mortgage, and half in the uncle, subject to the mortgage.
A
  1. (D) The friend’s execution of the mortgage severed the joint tenancy, leaving the coworker and the friend as tenants in common in the 30-acre tract of land, with the friend’s half-interest encumbered by the mortgage. At common law, and still in about 20 states, the mortgage operates as a conveyance of the legal title to the mortgagee. However, such title is subject to defeasance on payment of the mortgage debt. Here, there are no facts that speak to a default. Because the friend severed the joint tenancy in the 30-acre tract of land, by encumbering his interest in it, the uncle may inherit the friend’s half-interest in the 30-acre tract of land. If the uncle fails to pay the mortgage, the bank may take ownership of a half-interest in the 30-acre tract of land. Because there has been no default, the bank has no right to do so, and therefore the co-worker owns half and the uncLe owns half as tenants in common. Answer choices (A) and (B) are wrong because the mortgage severed the joint tenancy, thus destroying the right of survivorship. Thus, on these facts, title to the 30-acre tract of land cannot vest in the co-worker alone. Choice (C) is incorrect because the friend only had the right to encumber his interest in the 30-acre tract of land. The co-worker’s interest cannot be subject to the mortgage.
156
Q
  1. A shopkeeper is the owner of a vacant lot in fee simple absolute (the record title is also in the shopkeeper). In 1960, the shopkeeper conveyed the property by general warranty deed as follows: “The grantor hereby conveys the vacant lot to my friend, but if the property shall ever be used for church purposes, then to the children of my sister who reach the age of 25.” At the time of the conveyance, the sister was single. Thereafter, the sister married and had two sons. Subsequently, one son died in an automobile accident. Three years after that son’s death, the sister had a daughter.In an appropriate action to determine the ownership rights to the vacant lot with all minors represented, title is in(A) the friend only.(B) the friend, the son, and the daughter.(C) the friend, the son, the daughter, and any additional children of the sister born during her lifetime.(D) the friend, the son, the daughter, and any additional children of the sister born within 21 years after the death of the deceased son.
A
  1. (A) In brief, the RuleAgainstPerpetuities provides that “no interest (usually meaning, contingent remainders or executory interests) is good unless it must vest, if at all, not later than 21 years after some life in being at the creation for the interest.” Remember that the rule is measured from the time the instrument takes effect, and the time for vesting is calculated accordingly. In the case of a deed, the period is measured from the date of delivery. The contingent remainder in favor of the sister’s children who reach the age of 25 violates the Rule against Perpetuities because it might not vest within 21 years of the sister’s death. To illustrate, assume that one year after the conveyance the sister gave birth to the son. The sister then dies two years later. Now, 23 years after the sister’s death, the son reaches the age of 25. Because this is more than 21 years afterthe sister’s death, itviolates the ruLe (which requires the vesting not later than 21 years after “some life in being”). Hence, no other lives (of the sister’s children) can vest; therefore, choice (A) is correct. (B) is incorrect because where a conveyance violates the Rule against Perpetuities, the offending language will be stricken. Here, the language “but if the property shall ever be used for church purposes, then to the children of my sister who reach the age of 25” will be removed, leaving only the language “to my friend.” Although the shopkeeper’s intent was to give the friend a fee simple subject to an executory interest, the friend will take in fee simple because the second gift violates the Rule Against Perpetuities. Choice (C) is wrong because the son and the daughterwill take nothing, because the language that conveyed the vacant lot to them as members of the group “the sister’s children who reach the age of 25” will be struck. Choice (D) is incorrect because, as discussed above, the son and the daughter wilL take nothing. Note that the deceased son need not be the measuring life. Furthermore, artificially closing the class within 21 years of the death of the deceased son—or any other measuring Life—cannot bring the conveyance into compliance with the Rule Against Perpetuities, and so, to the extent that it violates the rule, the conveyance will fail.
157
Q
  1. The city installed a sewer line across a residential subdivision. The city acquired a valid easement for construction of the sewer from the development company that owned the subdivision. One year later a buyer purchased a house within the subdivision. Unknown to the buyer, the sewer line ran across his property, approximately 10 feet beneath the foundation of his home. The deed that the buyer received from the grantor, the development company, made no mention of the easement. Ten years after the buyer purchased his house, a crack in the sewer line caused water to leak into the buyer’s basement. The flooding resulted in extensive damage to his home. The city has abolished governmental immunity.In an appropriate action by the buyer against the city to recover damages, the plaintiff will probably(A) not prevail, because the sewer line was installed before the buyer purchased the property.(B) not prevail, because the city had acquired a valid easement for the sewer line.(C) prevail, only if the sewer line was negligently maintained.(D) prevail, because under the circumstances, the sewer line constituted a public nuisance.
A
  1. (C) A holder of an easement in gross may make reasonable use of the servient tenement as long as her conduct does not unreasonably interfere with the use of the servient tenement owner. Choice (A) is wrong because it falsely suggests that if the city took an easement before the buyer purchased, that circumstance forecloses any chance of recovery. The fact remains that if the city had unreasonably used the servient tenement or negligently maintained the sewer line, then the buyer could still recover, even if the easement for a sewer was granted before the buyer purchased. Similarly, choice (B) is wrong because even though the easement was granted validly, the city could still be liable if they had unreasonably interfered with the buyer’s use of the servient tenement or negligently maintained the sewer Line. The city may not make unreasonable use of the sewer and will not necessarily be liable for damage arising from reasonable use. The holder of an easement in gross will be liable for unreasonable interference with the use of the servient tenement, or for damage arising from unreasonable use of the easement. Note that negLigent maintenance would constitute unreasonable use. Public nuisance is defined as an unreasonable interference with a right common to the general public. Choice (D) is incorrect. The issue of public nuisance is meant to distract here.
158
Q
  1. In 1985, a landowner, the undisputed owner of an apartment building, leased it to a tenant for a term of seven years. Rent was to be paid in installments due on the first day of each month. One year later, in 1986, the landowner died leaving no heirs. In her will, the landowner left her entire estate to the Girl Scouts of America. The apartment building was not specifically mentioned in the will.One month after the landowner died, the tenant, having learned of the landowner’s death, decided to remain on the property, but stopped paying rent. The Girl Scouts of America organization was unaware of the landowner’s gift to the organization until 2009, when attorneys for the organization conducted an inventory of testamentary gifts to the Girl Scouts of America, which revealed that they had a claim to ownership of the apartment building. The statutory period for adverse possession in this jurisdiction is 20 years.In an ejection action by the Girl Scouts of America against the tenant, the organization will prevail(A) because the tenant discontinued paying rent following the landowner’s death.(B) if the tenant leased the apartment building to a teacher for three years while he (the tenant) was called into military service.(C) if the tenant believed that his lease with the landowner was still in effect after the latter died.(D) because the tenant never paid taxes on the apartment building.
A
  1. (C) The tenant’s failure to pay rent does not help the Girl Scouts in their action for ejection because it tends to show that the tenant was acting in derogation of the rights of the true owner who would be someone other than himself (the tenant). This would make the tenant’s behavior like that of an adverse possessor. For this reason, choice (A) is wrong. Answer choice (B) speaks to the issue of a continuous possession and tacking, but this does not answer the question of whether or not the tenant did so under the belief that he was acting in derogation of the rights of the true owner. In other words, choice (B) does not explain whether or not the tenant was leasing to the teacher to keep the lease alive (so that the tenant would not be in breach of the lease) or whether the tenant did this to keep up a chain of unbroken occupation to meet the elements of adverse possession. As a result, choice (B) is wrong because it does not help the Girl Scouts. The fact that the tenant never paid taxes is immaterial to the Girl Scouts’ action because it does not tend to negate the elements of adverse possession. For this reason, choice (D) is incorrect. Answer choice (C) is the best answer because if the tenant’s remaining on the land was permissive (due to his belief that the lease was still in effect) then there has been no adverse possession because the tenant would have believed he was only on there as a tenant. This would help the Girl Scouts to prevail.
159
Q
  1. In 1985, a widow, the undisputed owner of a cottage, leased it to a tenant for a term of seven years. Rent was to be paid in installments due on the first day of each month. One year later, in 1986, the widow died leaving no heirs. In her will, the widow left her entire estate to the Boy Scouts of America. The cottage was not specifically mentioned in the will.One month after the widow died, the tenant, having learned of her death, decided to remain on the property, but stopped paying rent. The Boy Scouts of America organization was unaware that the widow made a gift to the organization until 2009, when attorneys for the organization conducted an inventory of testamentary gifts to the Boy Scouts of America, which revealed that they had a claim to ownership of the cottage. The statutory period for adverse possession in this jurisdiction is 20 years.The tenant’s statutory period for adverse possession began to run when(A) the widow died.(B) the tenant discontinued paying rent.(C) the tenant’s lease with the widow expired.(D) the tenant subjectively believed that he no longer had permission to possess the cottage.
A
  1. (D) Adverse possession must be actual and exclusive, hostile but peaceable, (meaning against the interest of the “true” owner, without physical eviction or eviction by court action), open and notorious, and continuous for the statutory period. The “hostile” element is at issue in this question. There are two views that have interpreted “hostile”: the “objective view” and the “subjective view.” Under the objective view, the possession must simply be adverse to the owner’s rights; the possession must be without the true owner’s permission and done in a manner that is inconsistent with true owner’s rights. Under the subjective view, “hostility” requires bad faith or intentional trespass, so a mistaken possession in some jurisdictions does not constitute hostility. The former is the default view on the MBE, unless the facts of the question state otherwise. Choice (D) is the best of the lot under either view of what constitutes “hostile.” Choice (D) presents an unmistakably hostile possession under either view. The statutory period did not necessarily begin when the widow died. The tenant had a seven-year lease, and arguably had a right to remain on the property even after the widow’s death; so, the possession did not necessarily become adverse just because the widow died, and, thus, choice (A) is wrong. This makes choice (C) attractive, but the termination of the lease term is non-determinative of when the adverse possession began. Choice (C) still doesn’t tell us whether any further occupation would be without permission (under the objective view) or what the tenant believed when the lease was over (under the subjective view). In other words, the end of the lease does not necessarily mean that the elements of adverse possession necessarily begin. Under that same reasoning, choice (B) is incorrect because stopping rent payments (much like the end of the lease) doesn’t tell us whether any further occupation would be without permission (to meet the test under the objective view) or what the tenant believed when he stopped paying rent (under the subjective view). The tenant’s presence there may have been permissive for some time after he stopped paying rent. So, choice (D) is a better answer than choice (B).
160
Q
  1. In his will, a jockey devised his home “to my friend to whom I am everlastingly grateful for the devoted care he has lavished on my horses, but if ever my horses who survive me shall fail to receive proper care, then to my beloved daughter and her heirs, if she be living and own any horses, otherwise to the Equestrian Society.”In an appropriate action to construe the will, the court will determine the friend’s interest to be a(A) fee simple determinable.(B) fee simple subject to condition subsequent.(C) fee simple subject to an executory interest.(D) contingent remainder.
A
  1. (C) A fee simple subject to an executory interest is a fee simple estate, whereupon the happening of a named event, ownership is to pass from the grantee to one other than the grantor. The future interest created in the third party is an executory interest. Here, the daughter has an executory devise. The characteristics of this interest are exactly the same as attach to springing and shifting uses, but the devise, of course, is created by will and not by deed. Choice (A) is incorrect. A fee simple determinable is a fee simple estate created to continue until some specified event occurs, at which point the estate automatkally terminates and reverts to the grantor or his estate. The friend’s estate will go to either the daughter or the Equestrian Society, notrevertto the jockey. Choke (B) is wrong because in the fee simple subject to condition subsequent, the estate is terminated by the entry or exercise of the reserved power by the grantor (or her heirs) if that power is exercised. However, because the grantor has chosen not to reserve any future interest for himself, but rather devise it to the daughter or the Equestrian Society (should the horses fail to receive proper care), this is not a fee simple subject to condition subsequent. Choice (D) is incorrect. The friend’s interest could not be a contingent remainder because a contingent remainder is a future interest. The friend received a present possessory estate (a fee simple subject to an executory interest), which takes effect immediately, as long as he abides by the conditions placed on the estate.
161
Q
  1. In his will, a veterinarian devised his home “to my friend to whom I am everlastingly grateful for the devoted care she has lavished on my prize Pomeranians, but if ever my dogs who survive me shall fail to receive proper care, then to my beloved son and his heirs, if he be living and own any dogs, otherwise to the American Society for the Protection of Animals.”Assuming that the Rule against Perpetuities is in effect in this jurisdiction, which of the following statements is correct regarding the son’s interest under his father’s will?(A) It is a contingent remainder.(B) It is an executory interest of a shifting type.(C) It is an executory interest of a shifting type that is void under the Rule against Perpetuities.(D) It is an executory interest of a springing type that is void under the Rule against Perpetuities.
A
  1. (C) Executory interests and devises are subject to the Rule against Perpetuities. In the present case, the Rule is violated because it is possible that the son’s interest will neither vest nor fail within the lifetime of the friend. This is so because the Pomeranians may fail to receive proper care after the friend’s death. Had the son’s interest been limited to take effect “if the dogs fail to receive proper care during the friend’s lifetime” (instead of “if ever the dogs fail to receive proper care”), then the interest would not have been void under the Rule against Perpetuities. An executory interest of a shifting type is one where the right to possession shifts from one grantee to another grantee. Because the interest would shift from the friend to the son, this is of the shifting type. However, the future interest would fail because the Rule against Perpetuities would void it. Thus, choice (C) is the more complete choice because it addresses both that this is a springing executory interest and that it will be void. Choice (B) is incorrect because it fails to take into account the Rule against Perpetuities. A remainder is a future interest created in a third person that is intended to take after the natural termination of a preceding estate. A contingent remainder is a remainder subject to a condition precedent or created in favor of an unborn person or an existing but unascertained person. The son’s future interest is not a remainder because the son would not take upon the natural termination of any preceding estate; rather, the son would take (if at all) when a contingency occurs (the failure of the Pomeranians to receive proper care). The son never received a remainder, so choice (A) is incorrect. A springing executory interest is a future interest that cuts short an estate held by the grantor. Because the veterinarian, the grantor, held no present possessory estate to be divested of (the veterinarian conveyed his home to the friend), the executory interest was not of the springing variety. Thus, choice (D) is incorrect.
162
Q
  1. A housing corporation owned farmland and prepared a development plan to divide the land into 100 lots and create a residential community on the farmland tract. The Declaration of Covenants created the community association, as an administrative entity, to which the residential community lot owners would pay dues, and which would administer and enforce the regulations and restrictions recited among the covenants. One of the restrictions set forth in the Declaration of Covenants provides that the lots be used only for single-family residences, and that no trade, commerce, or business may be conducted in any single-family residence.The Declaration of Covenants gives the community association the right to assign all of its rights, powers, titles, easements, and estates granted to it under the Declaration of Covenants. The community association assigned “all the rights, powers, titles, easements, and estates granted to or conferred upon it by the Declaration of Covenants” to a municipal corporation, the city. The community association was then terminated. A chef, the owner of lot 18 in the residential development, proposes to convert his single-family dwelling into a massage parlor. The city asserts an action against the chef to recover money damages.Which of the following is the best argument for the chef?(A) The restraint on alienation of his land is invalid.(B) The city is not in privity of estate with community association.(C) The benefit is in gross; hence, the burden cannot run.(D) The burden is in gross; hence, the benefit cannot run.
A
  1. (B) In order for a covenant to run with the land, the following four requirements must be met: (1) there must be a covenant; (2) there must be an intention that the covenant shall run with the land; (3) the covenant must be of a type that “touches and concerns” the land; and (4) there must be privity of estate. With respect to the privity requirements, Smith and Boyer state that “where the running of covenants is allowed the cases seem to agree that ifthere is (i) privity of estate between the original covenanting parties and (ii) simultaneous ownership by the covenanting parties in the same Land and (iii) privity of estate between either of the covenanting parties and his assignee, the covenant will run with the land.” In our case, under the terms of the Declaration of Covenants, there is privity of estate between the developer (the housing corporation) and the purchasers of the lots (e.g., the cheO. Choice (B) is the best answer because there is no privity of estate between the housing corporation and the community association. The housing corporation simply owns the farmland, and the community association is an administrative entity created by the housing corporation. It has no interest in the farmland, either as a co-owner or as a conveyee of an interest therein. Between the housing corporation and the community association, there is simply the covenant (i.e., Declaration), which constitutes privity of contract. Therefore, there is no successive privity of estate between the community association and the city, because the latter only succeeds to the interest or estate that the association has in the land: none. Answer choice (A) is incorrect because there is no alienation (sale oO land attempted here. The chef doesn’t want to sell his lot; he wants to convert it into a massage parlor. Choices (C) and (D) are both incorrect because both misuse language that is used in a discussion of easements, which can be in gross or appurtenant. Indeed an easement appurtenant (but not an easement in gross) will run with the land. However, covenants are being tested here.
163
Q
  1. A housing corporation owned a tract of land. The housing corporation prepared a development plan to divide the land into 100 lots and create a residential community on the tract. The Declaration of Covenants created the homeowners’ association, an administrative entity that would administer and enforce the regulations and restrictions recited among the covenants. One of the restrictions set forth in the Declaration of Covenants reads:“There shall never at any time be erected, permitted, or maintained upon any part of the property any structure designed for or used as a saloon or place for the sale or manufacture of malt, vinous, or spirituous liquors.”The Declaration of Covenants was duly recorded and was included in the deed taken by a teacher when he purchased lot 62 in the development. The teacher immediately recorded his deed. The teacher leased his home to a librarian for a term of one year. The lease included the same restrictions as those in the Declaration of Covenants and in the teacher’s deed. The librarian immediately began to sell liquor on the premises during weekly “after hours” parties. The homeowners’ association sues the librarian in an action for damages.Which of the following is the best argument for the librarian?(A) The rule in Spencer’s Case prevents the librarian from being liable.(B) The librarian is not in privity of contract with the homeowners’ association.(C) The librarian is not in privity of estate with the teacher.(D) Other lots in the immediate vicinity are used for commercial purposes.
A
  1. (B) In order for the burden to run, we need both horizontal and vertical privity. Vertical privity is lacking, so the burdendoes NOT run. Absent being bound by a covenant that runs with the land, contract liability would be the only thing to hold the librarian liable to the homeowners’ association for damages. Choice (A) is wrong because the Rule in Spencer’s Case provides that an assignee of either the reversion or of the leasehold estate cannot be held liable for breach of covenant if the covenant is of a type that “does not touch and concern the land.” Because the librarian is a tenant, not an assignee, the Rule in Spencer’s Case is inapplicable. A tenant is in privity of estate with her landlord. The librarian is the teacher’s tenant, and is thus in privity of estate with him. Therefore, lacking privity of estate is simply not the best argument for the librarian. Thus, answer choice (C) is wrong. Answer choice (D) is incorrect because a plaintiff seeking to enforce an equitable servitude may be denied relief on equitable grounds if the purpose of the servitude in a development scheme is impossible to attain because of changed conditions. In this question, the ability to distinguish between legal and equitabLe remedies makes it easy to eliminate choice (D); the homeowners’ association is suing the librarian in an action for money damages (for breach of a covenant), so legal, rather than equitable, remedies are in issue. Because choice (D) presents an argument that is appropriate to refute the enforcement of an equitable servitude, it is misplaced and therefore incorrect. Given that privity of contract is required in order for the burden to run, the librarian’s best argument is that he is not in privity of contract with the homeowners’ association, and never had a contract with the teacher, or any of the other individuals who purchased lots from the housing corporation. For these reasons, choice (B) is correct.
164
Q
  1. A housing corporation owned a tract of land and prepared a development plan to divide the land into 100 lots and create a residential community on the tract of land. The Declaration of Covenants created the community association, an administrative entity that would administer and enforce the regulations and restrictions recited among the covenants. One of the regulations set forth in the Declaration of Covenants reads:“Each purchaser, by the acceptance of a deed therefore, promises to pay the community association an annual assessment or charge to be determined on the basis of the valuation of each individual lot and the improvements thereon. Nonpayment of any annual assessment or charge when due shall result in a lien upon the parcel of the property.”A gardener, the owner of lot 29 in the development, sold his land to a landscaper with a deed containing no restrictions. The community association, pursuant the Declaration of Covenants, sues the landscaper to collect the annual assessment for lot 29.Which of the following is the best argument for the landscaper?(A) There is not privity of contract between the housing corporation and the community association.(B) Because the charge constitutes a lien, there is no personal obligation on the landscaper’s part.(C) There is no privity of contract between the gardener and the landscaper.(D) There is no privity of estate between the gardener and the landscaper.
A
  1. (B) The provision, which provides forthe annual assessment payable to the community association, is enforceable as a covenant running with the land. The effect of such a covenant is that the benefit, burden, or both pass to succeeding holders of the estate of the original covenanting parties. Here, the landscaper is the gardener’s successor in interest and would ordinarily be obligated to perform under the contract. However, choice (B) is the best answer because the Declaration of Covenants stipulates that “nonpayment of any annual assessment or charge when due shall result in a lien upon the parcel of the property.” Thus, the landscaper’s best argument is that the landscaper is not personally liable for the payment, because failure to pay simply results in a lien on the parcel. Choice (A) is incorrect. Here, privity of contract between the housing corporation and the community association is not necessary. It is sufficient that the community association is the intended recipient of the annual payments. Choices (C) and (D) are incorrect because the conveyance of the property from the gardener to the landscaper created both privity of estate and privity of contract between the parties.
165
Q
  1. A housing corporation owned a tract of land and prepared a development plan to divide the land into 100 lots and create a residential community on the property. The Declaration of Covenants created the community association, an administrative entity that would administer and enforce the regulations’ and restrictions recited in the Declaration of Covenants. One of the restrictions reads:“There shall never at any time be erected, permitted, or maintained upon any part of the property any structure designed for or used as a saloon or place for the sale or manufacture of malt, vinous, or spirituous liquors.”The Declaration of Covenants was duly recorded and was included in the deed taken by a psychologist when he purchased lot 24 in the housing development. The psychologist recorded his deed. The psychologist gave his lot to his son. The deed conveying lot 24 to the son contained no reference to the Declaration of Covenants or any of its provisions. The deed provided that “these premises are conveyed to (the son), his heirs, and assigns, as long as they are used for residential purposes only.” The son did not record his deed. The son was unaware of the Declaration of Covenants. The son started a home business selling imported wine from his home. A geologist, the owner of lot 26, which was situated next to the son’s lot, brought an action of ejectment against the son.Which of the following is the best argument for the son?(A) The deed to the son created a fee simple determinable with a possibility of reverter, giving the psychologist, but not the geologist, the right to sue the son.(B) Not having been recorded, the condition cannot be enforced against the son.(C) The geologist is entitled only to an injunction against the son.(D) The law prohibits a fee simple determinable.
A
  1. (C) Under the “collateral document rule,” where the developer intends a common scheme for the entire parcel of land, including all of the plots, a land owner whose deed does not contain the restriction may be bound by the restriction if the other deeds of the adjacent properties contain the restriction. The purchaser of a plot whose deed does not contain the restriction is deemed to be on constructive notice of the contents of the deeds of adjacent properties. This notice is, of course, constructive notice. However, the purchaser of a plot does not have to be on actual notice of the restriction to be bound by it. Here, the son has constructive notice (record notice) of the common scheme for the housing development. Therefore, choice (C) is the best answer because the geologist may seek to enforce the restriction against the sale of alcohol on the premises as an equitable servitude. The geologist may obtain an injunction to enforce the restriction, but he is limited to injunctive relief and may not eject the son. Choice (A) is incorrect. Although the conveyance did create a fee simple determinable, which would give the grantor (the psychologist) a possibility of reverter, the geologist need not rely on the provision in the psychologist—son deed; he may enforce the restrictions set forth in the Declaration of Covenants. Answer choice (B) is wrong because, despite the fact that the deed was not recorded, the restriction would nevertheless be enforceable because the son would have constructive notice of the restriction. Choice (D) is incorrect. Among the rights attendant to property ownership is the right to freely alienate it. Practically all American states recognize the fee simple determinable.
166
Q
  1. A retiree lived in a single-family dwelling in the city. Adjacent to his home was a vacant lot that measured 100 feet by 175 feet. The lot, which the retiree owned, was situated on the corner of Davis Street and University Way. The tract measured 100 feet along Davis Street and 175 feet along University Way.The retiree executed a deed purporting to convey the vacant lot to his lifelong friend for the consideration of $28,000. After escrow, the deed was delivered to the friend, who immediately filed it with the county recorder’s office. The recorded instrument described the property conveyed as “all the tract of land beginning at the northwest corner of Davis Street and University Way; thence west along Davis Street 100 feet; thence north 175 feet; thence west 100 feet; thence south 175 feet along University Way to the place of beginning.” Three months after his conveyance to the friend, the retiree died intestate. His heirs have now filed an appropriate action contesting the friend’s title to the vacant lot.Which of the following statements is most accurate concerning the outcome of this suit?(A) The retiree’s heirs will prevail because metes and bounds, rather than streets, are appropriate boundary descriptions.(B) The friend will prevail because equity will not permit forfeiture for a mere technicality.(C) The outcome will depend on whether the tract of land was plotted as a lot in a subdivision.(D) The outcome will depend on whether the last call (175 feet along University Way) prevails over the third call (west 100 feet).
A
  1. (D) As a general rule, no conveyance is valid unless the description of the land sought to be conveyed is sufficient to identify the land. In this question, the deed from the retiree to the friend purports to convey a 100-foot-by-175-foot lot, but the recorded instrument describing the property fails to describe an encLosed area, because the third call goes in the wrong direction. In the heirs’ action contesting the friend’s title to the vacant lot, the outcome will depend on whether the last call—which establishes an enclosure by stating that the boundary line returns “to the place of beginning”—prevails over the erroneous third call (west 100 feet). Choice CD) is correct. Note: When a deed describes the boundaries of the Land to be conveyed by reference to monuments, natural or artificial, the intention of the parties is the controlling factor and all rules of construction are mere aids in determining such intention. Choice (A) is incorrect because street descriptions (i.e., monuments) are appropriate boundary descriptions. Choice (B) is incorrect because an improper boundary description is never considered a “mere” technicality, and if no land is enclosed, the result may be an invaLid conveyance. Choice (C) is incorrect because if the tract of land had been platted in a subdivision, it would have been described by a specific parcel number or lot number, not by a boundary description.
167
Q
  1. A college student and her boyfriend lived together in a one-bedroom apartment in Los Angeles. They were engaged to be married when they purchased an undeveloped parcel of land on a hillside overlooking the Pacific. The deed, which was properly executed and recorded, named the student and the boyfriend as grantees, “not as tenants in common but as joint tenants with right of survivorship.”Thereafter, the boyfriend, who was experiencing financial difficulties, offered to sell the property to his co-worker. Without the student’s knowledge, the boyfriend executed a deed that purported to convey the hillside property to the co-worker in fee simple. The boyfriend signed his name and forged the student’s name. He then delivered the deed of conveyance to the co-worker, who paid the boyfriend $150,000, which was the fair market value of the property. The co-worker immediately recorded the deed received from the boyfriend. The common law joint tenancy is unmodified by statute.Title to the property in question is now in(A) the boyfriend and the student as joint tenants with right of survivorship.(B) the co-worker and the student as joint tenants with right of survivorship.(C) the co-worker and the student as tenants in common.(D) the co-worker as owner in fee simple.
A
  1. (C) The traditional rule is that a forgery voids the instrument. Placing such a deed on record does not add any legal efficacy to such a forged or undelivered instrument. The recording statutes are not intended for the purpose of assisting wrongdoers, tortfeasors, criminals, and forgers in depriving innocent owners of their real prop. erty. It is settled that anything forged is null. However, there is a jurisprudential spLit as to the issue of “partial validity” under circumstances like these where the boyfriend’s signature was real, but the student’s was forged. Partial validity of a conveyance has been upheLd. More and more states have held that in such situations where the person doing the forging had something to convey, s/he conveys the interest she did have in the property, and the deed is a nullity to the extent that it is forged. The other party’s property, in which the perpetrator of the forgery had no interest, is not conveyed. Therefore, the boyfriend successfulLy severed the joint tenancy and transferred a half-interest in the property to the co-worker, Leaving the co-worker and the student as tenants in common. Choice (A) is attractive under the traditional rule that forgery voids the instrument, but it is incorrect because, as discussed above, modernly, a partial forgery can result in a partial conveyance. Choice (B) is wrong because the joint tenancy is severed by the sale, and the coworker and the student must hold it as tenants in common. Answer choice (0) is incorrect because the forged instrument cannot transfer the student’s interest in the property.
168
Q
  1. Walter, a widower, died in 1997 leaving $1,000,000 to Trend Trust Company in trust to pay the income to his son, Stan, for life. Stan was married to Morgana and had two children, Andrew and Beverly. Walter’s will provided in part:“The net income from this trust shall be paid to my son, Stan, for his life. Upon Stan’s death, the net income is to be paid to any widow of Stan. Upon the death of Stan’s widow, Trend Trust Company shall then pay the income (from said trust) to the living children of my sister, Harriet, in equal shares.”Harriet’s only surviving child, Grace, was born in 2001. Both Stan and Morgana died in an airplane crash in 2009. There is no statute modifying the common law in this jurisdiction.Harriet, on behalf of Grace, brings an appropriate action against the Trend Trust Company and Walter’s estate to allow the distribution of the income from said trust to be paid to Grace. Is Harriet likely to prevail in this action?(A) No, because Grace was not a life-in-being at the time of Walter’s death.(B) No, because the provisions under which Grace was intended to take violate the Rule against Perpetuities.(C) Yes, because that was Walter’s intent.(D) Yes, because all other persons who would have had any claim to the income from the trust corpus are deceased.
A
  1. (B) The Rule against Perpetuities provides: “No interest is good unless it must vest, if at all, not later than 21 years after some Life in being at the creation of the interest.” Choice (B) is correct because postponing the vesting of an interest until after the death of the “wife” or “widow” of a Living person violates the Rule against Perpetuities. This is the classic “unborn widow” scenario. Walter’s son may eventually marry someone who has not yet been born when the interest was created, which was when Walter died. Note that this is a testamentary trust, so the interest was created when Walter died. A will speaks at death of the testator, not when s/he wrote the will. Answer choice (A) is incorrect because Grace need not be alive when Walter died. What is required under the Rule against Perpetuities is that the chiLd’s interest will definitely vest or fail within 21 years of the death of someone who WAS alive when Walterdied. Choice (C) is wrong because the intent of the testator is not determinative here. In fact, the purpose of the Rule against Perpetuities is to prevent the grantor from (accidentally or intentionally) controlling the property indefinitely. The common law tradition of the Rule against Perpetuities assures that the “dead hand cannot reach beyond the grave to control the living.” Gray’s Rule against Perp etuitles (1886). Choice CD) is an attractive answer because it is factually true. Stan has died, so he can have no additional widows, and it would seem that Grace, as the only living child of Harriet, would be allowed to take due to the death of the Stan and Morgana. However this is immaterial. Answer choice (D) is wrong because the Rule against Perpetuities requires that it be clear and certain at the time of conveyance that the interest will vest (or not vest) within 21 years of a life in being. The fact that it might vest, or could vest, during that 21-year period is insufficient to bring the conveyance into compliance with the Rule. Note that some jurisdictions take a wait-and-see approach; the facts say that in this jurisdiction, there is no statute modifying the common law rule.
169
Q
  1. A landlord, the owner of a large, high-rise apartment building in the city, leased a three-bedroom apartment in the building to a husband and wife. The written lease agreement specified that rent should be paid at a rate of $2,000 per month and provided that the tenants were required to pay two months rent initially, of which one half, or $2,000, should be held by the landlord as a security deposit. In addition, the lease contained the following provision:“The duration of this lease shall be month-to-month and either party may have the right to terminate on thirty (30) days notice.”The lease agreement did not specify whether notice of termination must be in writing, but it did provide that the deposit could be applied, at the landlord’s option, to unpaid rent and to reimbursement for damage done by the tenant(s) to the apartment or the fixtures attached thereto.On January 1, the husband and wife paid the landlord $4,000 and moved in. The tenants then paid the $2,000 rent on the first of each of the months of February, March, and April, and all rents were duly accepted by the landlord.On February 1, the city enacted a housing code applicable to all multiple dwellings in the city. A provision of that code required that in “any building having a main or central entrance that is open and accessible to the public directly from a public street, a doorkeeper shall be maintained on duty at all times.” Although the landlord’s building has such an entrance, the landlord failed to comply with the requirement.On March 15 and again on April 1, the husband and wife lodged complaints with the city’s housing authority relating to the landlord’s failure to hire a doorkeeper in violation of the code regulation. On April 5, the landlord orally gave notice to the tenants that he was terminating the lease and demanded that they vacate the premises by May 5. No damage was done to the apartment, furnishings, or fixtures. There is no statute applicable to cover any landlord—tenant dispute except those related to housing and building codes.In the landlord’s action to evict, the husband and wife have contested the landlord’s right to terminate on May 5. If the tenants prevail, it will probably be because(A) there was no damage to the apartment, furnishings, or fixtures.(B) the statute of frauds requires that the notice of termination be in writing.(C) the jurisdiction recognizes that retaliatory actions are a defense.(D) the husband and wife have the option to apply their $2,000 deposit to cover May’s rental.
A
  1. (C) The landlord accepted the rental payment of $2,000 on April 1. Four days later he gave them notice to terminate the premises in 30 days, despite the fact that no damage had been done to the premises nor had payment of rent ever been delinquent. When a tenant contests termination of the tenancy based on a landlord’s retaliatory actions, the court will place the burden of proof on the landlord to show that his actions were not retaliatory. Under these facts, the landlord’s actions certainly do seem retaliatory based on the husband and wife’s complaints to the city housing authority. Furthermore, if the landlord’s actions were not motivated by retaliation, he should not have accepted the April 1 rent. Choice (C) is the correct answer. (A) is incorrect because lack of damage to the apartment is not relevant basis to avoid termination of a periodic tenancy under which either party can term inate. Choice (B) is incorrect because the Statute of Frauds is a defense to contract formation, and therefore would not be the reason a writing might be required to terminate a periodic tenancy. Note that the general rule is that absent a lease term that provides fortermination, propertermination of a periodic tenancy requires that written notice be delivered a full period in advance, and that the tenancy terminate at the “natural” end of the period (so the landlord’s notice on April 4 would have the effect of terminating the tenancy on May 31). Here, however, the lease defines proper termination as 30 days notice by either party. No writing is required and the tenancy may terminate on May 5. Although choice (D) presents a true statement, this will not help the tenants defend against the landlord’s right to terminate on May 5.
170
Q
  1. A writer owned a building in a city. This was the only piece of real estate that the writer owned. The three-story building had a store on the ground floor and apartments on the other two floors. The writer entered into a leasehold agreement with a shopkeeper, who would lease the first floor, where she planned to open a sporting goods store. After identifying the parties, the operative words of the lease were as follows: “Landlord hereby agrees to lease for the three years the first floor of his building in the city to tenant, reserving unto said landlord annual rental in the sum of $12,000 payable in advance in monthly installments of $1,000.”At the moment of signing, the leasehold agreement entered into between the writer and the shopkeeper(A) created a periodic tenancy.(B) could be terminated at will by either party.(C) did not convey to the shopkeeper a term of years at law.(D) was void, invalid, and had no legal effect.
A
  1. (C) At the moment of signing, the leasehold agreement did not convey to the shopkeeper a term of years at law. Here, you are being tested on the following (rather obscure) common law rule: At the moment a leasehold contract is entered into (but before the tenant actually takes possession), he is deemed to have an interesse termin! (or equitable) interest in the leased premises. Once, however, the tenant takes possession and pays part of the rental, then his equitable interest vests into a term of years at law. Choice (C) is correct because at the moment of signing, the leasehold contract entered into between the writer and the shopkeeper did not conveya term of years at law. Rather, it created a term of years in equity, or an interesse term/ni interest. Choice (A) is wrong because we know from the facts that a term of years, not a periodic tenancy, was formed. A tenancy for a term of years has a fixed duration that is set forth in the lease, e.g., “for six months” or “for two years.” It ends automatically at the expiration of the term unless the parties agree to renew it. Here, the lease clearly states that the lease tenancy will last for three years. Answer choice (B) is incorrect because a tenancy for a term of years is properly terminated only upon the expiration of the term or upon properly tendered and accepted surrender. The only leasehold estate that can be terminated at will by either party is a tenancy at will. A tenancy at will is an estate in land that is terminable at the will of either the landlord or the tenant. At common law, this estate could be terminated by either party without advance notice. Choice (D) is an attractive answer because the language of the lease is vague with respect to the description of the property. However, choice (D) is incorrect because the description of the property in the lease is adequate to identify it. Here, the facts indicate the building was the only piece of real estate owned by the writer. Thus, the description of the building in the leasehold agreement is sufficient to identify the first floor of that building as the property that the shopkeeper would lease.
171
Q
  1. A widow owned an apartment building. Several years ago, a student, in a signed writing, entered a three-year lease agreement with the widow. The student agreed to pay $800 monthly rent for the third-floor apartment in the widow’s building. The original three-year term had long since expired, but the student had continued to pay his rent, and the widow continued to accept it.If the widow had chosen to evict the student immediately upon the end of the three-year period, and before she had accepted the next rent check, the student’s strongest argument to prevent eviction, if true, would have been that(A) the leasehold contract was void ab initio. (B) the widow had not given the student requisitenotice for terminating a periodic tenancy. (C) the widow had not given the student requisitenotice for terminating a term of years.(D) the terms of the leasehold contract were binding on both parties.
A
  1. (B) The student’s strongest argument is that the widow had not given him notice forterminating a periodic tenancy. Remember a periodic tenancy continues indefinitelyin the absence of either party’s giving notice of termination. Answer choice (C) is wrong because the stuaent does not have a tenancy for a term of years. If a tenant continues in possession after his right to possession has ended, the landlord may (1) evict him, or (2) bind him over to a new periodic tenancy. At common law, the new tenancy was usually from year to year when the term of the original Lease was one year or more. However, under the modern view, most states today regard the new tenancy as running from month to month. Here, the student stayed in the apartment after his tenancy for a term of years ended. By accepting his continued rent payments, the widow has created a periodic tenancy. Answer choice (D) is incorrect because even though the terms are binding on both parties, a periodic tenancy may be terminated by either party upon proper notice. By process of elimination, we are left with choice (B). The only way the student wiLl successfully avoid the widow’s attempt to terminate his periodic tenancy is if she did not provide the requisite notice. Choice (A) is incorrect because it states that the original leasehold contract was void from the beginning. Nowhere in the facts does it state that the lease was void or that there were any circumstances that would render it void. Moreover, the original three-year term had long since expired, so this would be a very weak argument for the student
172
Q
  1. A retiree owned a building in a city. Several years ago, an artist, in a signed writing, entered a three- year lease agreement with the retiree. The artist agreed to pay $800 monthly rent for the third-floor apartment in the retiree’s building. The original three-year term had long since expired, but the artist has continued to pay his rent, and the retiree continued to accept it. The building became infested with rats after a restaurant was opened in the adjacent building. The artist could not tolerate the rats and served notice on the retiree that he was moving to another city and has not been heard from since.At common law, what remedy was available to the retiree before she accepted the first rental payment after the end of the initial three-year period?(A) Eviction of the artist for non-payment of rent.(B) Recovery of the difference between the lease price and the fair market value of the leasehold for a year.(C) Recovery of double the rent prorated on a monthly basis for the number of months the artist was on the premises beyond the three- year period.(D) Require that the artist move out immediately or be held responsible for three years’ rent.
A
  1. (D) When a tenant who is rightfully in possession wrongfully remains in possession (holds over) after termination of the tenancy, he is known as a holdover tenant or a tenant at sufferance. If a tenant continues in possession after his right to possession has ended, the landlord may (1) evict him, or (2) bind him over to a new periodic tenancy. At common law, the new tenancy was usually from year to year when the term of the original lease was one year or more. However, under the modern view, most states today regard the new tenancy as running from month to month. Choice (D) is therefore correct because the hold-over doctrine permitted the landlord to hold the hold-over tenant to another term. Here, because the original lease between the retiree and the artist was for a term of three years, under the holdover doctrine the artist would be liable for another term under the common law rule. Choice (A) is incorrect because the artist had paid rent, so non-payment is an improper ground for eviction. Answer choices (B) and (C) are wrong because the retiree had only two options at the end of the lease—evict the artist or bind him over to a new tenancy. There is no rule at common law that provides such formulas for recovery.
173
Q
  1. A professor owned a building in a city. The building had a commercial space on the first floor, with two apartments above it; one on the second, and another on the third floor. Several years ago, a teacher, in a signed writing, entered a three-year lease agreement with the professor. The teacher agreed to pay $800 monthly rent for the third-floor apartment in the professor’s building. The original three-year term had long since expired, but the teacher has continued to pay his rent, and the professor continued to accept it.The professor rented the first floor to a restaurateur, who opened a sushi restaurant there. Within months, there was a citywide hookworm epidemic that was traced to the consumption of raw fish. The city council, in an attempt to protect the public from the parasites, passed an ordinance prohibiting all sushi restaurants from operating in the city. The restaurateur was forced out of business. She locked the doors and left a sign in the window: “Closed until further notice.”Within weeks, rats were attracted to the odor from the fish that was left in the restaurant, and the building became infested with them. The teacher could not tolerate the rats and served notice on the professor that he was moving to another city and has not been heard from since.The teacher’s best defense is that the professor, by permitting the restaurateur to store the fish in the closed restaurant, caused a(A) partial constructive eviction.(B) partial actual eviction.(C) constructive eviction.(D) breach of implied covenant of habitability.
A
  1. (C) As a general rule, liability for rent is not affected by the conduct of a lessor (or third person) that merely interferes with the lessee’s enjoyment of the leased land. However, if the disturbance is substantial in nature, the lessee may give up possession of the Land and thereby avoid further liability under the lease. His defense to a rent action is constructive eviction. In this situation, if the lessee sees fit to give up possession of the leased land, he thereby avoids liability for rent that becomes payable after that date. Choice (A) is attractive but incorrect because it addresses the correct theory, and because the professor did not actuallyevict the teacher; he had already left. However, constructive eviction, by its definition cannot be “partial”. Constructive eviction is a material breach by the landlord that violates the tenant’s implied covenant of quiet enjoyment and constitutes a constructive eviction if it renders the premises uninhabitable. Choice (B) is wrong for a similar reason; an eviction, by definition is not partial. Answer choice (D) is not the best answer as the breach of implied warranty of habitability would require that some basic necessity (like running water) is missing, and the landlord, having had the opportunity to cure, did not, and that fact rendered the residence inhabitable. Here, the restaurateur’s storage arguably interferes with a property right, but does not make the apartment uninhabitable. Note that in addressing this “best of the lot” type of question, you may be asked to identify the “best of the worst.” Here, most of the arguments would likely fail, but choice (C) is the best of the lot.
174
Q
  1. An investor owned a building in a city. This was the only piece of real estate that the investor owned. The three-story building had a store on the ground floor and apartments on the other two floors. The investor entered into a leasehold agreement with a tenant, who would lease the first floor, where she planned to open a sporting goods store. After identifying the parties, the operative words of the lease were as follows: “Landlord hereby agrees to lease for the three years the first floor of his building in the city to tenant, reserving unto said landlord annual rental in the sum of $12,000 payable in advance in monthly installments of $1,000.”After the tenant took possession and the investor accepted her rent payment for the first floor of the building, which of the following most accurately describes the legal relationship between the parties?(A) A tenancy at will was created.(B) A periodic tenancy from month-to-month was created.(C) A periodic tenancy from year-to-year was created.(D) The tenant’s equitable three-year term became a legal three-year term.
A
  1. (D) An estate for years is an estate the maximum duration of which is measured by a fixed duration that is set forth in the lease, e.g., “for six months” or”fortwo years.” At the time of the signing of the lease, the estate for years was not enforceable at law because the agreement failed to specify the exact beginning or commencement of leasehold period. Be advised that the most important requisite of an estate for years is that it must have a definite beginning and a definite ending (however, the ending can be inferred from the beginning). Once the tenant took possession, her estate began a legal three-year tenancy because a commencement date had been established and, therefore, a definite termination date (three years from the commencement date) was established as well. Answer choice (A) is wrong because a tenancy for a term of years, not a tenancy at will, was created because the lease agreement specified the exact term. A tenancy at will is an estate in land that is terminable at the will of either the landlord or the tenant. At common law, this estate could be terminated by either party without advance notice and will continue indefinitely until one of the parties terminates it. Choice (B) is an attractive choice because the agreement does indicate that the rent is to be paid in monthly installments; however, the determinative fact is that the lease agreement specified a term of three years, so it is incorrect. Similarly, Choice (C) is attractive because the agreement does express the amount of rent to be paid in terms of an annual rate, but the key issue is that the tenant agreed to rent the first floor for three years at that rate. Thus, it is also incorrect.
175
Q
  1. A rancher is the owner of a ranch situated upon the top of a mountain. Located below the ranch is a 40-acre farm that is owned by a farmer. There is a stream, which is a non-navigable watercourse, that originates at the top of the mountain and runs all the way down into the valley below. Both the ranch and the farm are within the watershed of the stream.When the farmer purchased the farm in 1974, he began taking water from the stream and used it to irrigate the southern half of his property which he has used as a farm. Prior to 1974, the southern half of the farm had been cleared and placed in cultivation, while the northern half remained wooded and unused except for an occasional hike or gathering of timber for use as domestic fuel. The farmer continued this established pattern of use. Now (January 2010), he is still taking water from the stream and using it to irrigate the southern half of the farm.In 2008, the rancher built a home on the ranch and began taking water from the stream for domestic purposes. During that year there was heavy rainfall, and this caused the stream to run down the mountain at a high water level. The next year, however, there was a drought. As a result, the stream flowed at a very low level. Consequently, there was only enough water to irrigate the farmer’s farmland or, in the alternative, to supply all of the rancher’s domestic water needs and one quarter of the farmer’s irrigation requirements. The mountain is located in a jurisdiction where the period of prescription is 15 years.Inasmuch as the stream is still flowing at a very low level and the rancher is continuing to take water for his personal needs, there is insufficient water to irrigate the farm. As a consequence, the farmer brings an appropriate action to declare that his water rights to the stream are superior to those of the rancher. In addition, the farmer moves to have the full flow of the stream passed to him, notwithstanding the effect it might have on the rancher. If this state follows the doctrine of prior appropriation, judgment should be for whom?(A) The rancher, because as an upstream landowner, he would have superior rights to the water than a downstream owner.(B) The rancher, because domestic use is superior to and protected against an agricultural use.(C) The farmer, because he has obtained an easement by prescription to remove as much water as he may need.(D) The farmer, because he has put the water to a beneficial use prior to the rancher’s use and has continuously used the water.
A
  1. (D) This question directs us to follow the prior appropriation doctrine, under which “first in time is first in right. “Answer choice (A) is wrong because status as upstream landowner does not allow the rancher to materially affect the farmer’s rights under the prior appropriation doctrine, or under the majority rule, which provides that a lower riparian has a right of action against an upper riparian whenever the latter’s use of the water materially affects eitherthe quantity or quality of the lake or stream waters, even though such use results in no injury or damage to the lower riparian. Choice (B) is an attractive one because it alludes to the majority rule that appLies to non-navigable Lakes and streams: water for natural purposes is paramount and takes precedence over the use of water for artificial purposes. Thus, although upper riparians can take all the water they need for natural uses, they cannot take it for artificial purposes unless there is enough water for the domestic needs of the lower riparians. Answer (C) is meant to distract here, as it improperly suggests that the law of easements is applicable here. Because neither land owner enters the land of the other to secure water for his own use, the facts do not trigger easements as an issue.